You are on page 1of 306

Module in

Integrated Math 7
1

Based on Most Essential


Learning Competencies (MELC)
Sets and


 Set
  Operations
01234351607829
!"#$!%
~ ..8
 ..€ { |
 ..8Zbg..€
‚ ..8_^..€
ƒ ..8..€ }
„1.$  +# ..!…

z opqorstqupvwxpxypwo
&'()!* + ,-.! !$,!/ 7*!.*$
,.=!#
01"*,!!!234"3"5"622/ !!/
7.*#!! 89:4"3"5"6"22;
&!!*!*$<=8 +$"!.$#
#  !!,.!/
7.! *!# ! +!/01"!
+*! +#.!!23.*,#!
> 9:*! +3#.?23;  > 9:3"@"A"24;/
B1! +!!%
C : " " " ; !! +$!#!!6!!/
C :,*"$"D",#";!! +$.*!/
C :E"F"G"H"I;!! +<#! !,/
 ,-.! !$,!!.(J(K(L)' K(KM(N' +!/
OPQRSQTQUSR
C V!% WXYZ[\[]\^_à WXYZb\c\[\bd_à WXYXba W]\c_beYd_a
C Vf!% WXYb_dZ[\[]\^_à WXYb_dZb\c\[\bd_à WXYb_dXba 
Wg_\Yb_d]\c_bed_a/
C hi8j!% Wdk\bl[]\^_`\c\[\bdYXbY\dma/
01" + 89:4"3"5"6"22;  5V8" nf V8"  hi8j95/
0120 3456789
 
9WXYZHC [\]^_`abc
 998868888599998
8599867
99
9899 !6"#!
$8599866%8
&'
( 2
9
69
& ) &#* ) +*
)$ , & )$-
. /"  012  01$2'
 34  # 5 6&#&67 $34#5&7
( /69' & 867
99
8'
/69' &#85998'
2
9' +885998$'
, 2
9' & 85998$'
. 01238 46859
96
9899987
01$23+ 46859
96
9+9998$7
9:9;<=>9?@ABC
C D885999
859986 d9899fg6"Phe
 78E99E99
9856 + ijklmjnompqrik
fg6"Ph'
( 78E9""9
9899 F6"#!
. 7
99
9856&#
G 9E97
99

H 7
96%8
 &
I87955%56
99856F!
J %879&6"F5%56
99856+!
K L6
99
9899&!6" !!'
? M5%5859N6
98
9-
 5)47
99
9856#!7 ( +* )4879+7
. )47
99
7 G &F5)4""9
7
H 6 )*47
99
7 I OP )*4599
7
J Q)41N5E97
K #)4879#76"#)46%8
87
@ D88599998859N98'086898599
999896%598N
01RRRRRR2868
/
9S679 T 34879 5%56
99856F!7 859
T 34 &&##++76"01T23+'
 347
99
9856&F7
( T 3487959899&!6"8!7
. U349
5%5"E"99S6%8V8#&7
012 3453 6789
1
 3

  8  9
    9 ! 8  99"  
#$ 9% 8  9 
 &8"89   89'(
) * 8     8   !9%+  9 (
, -./
0.1 9.1! 00.1! 9.1% 000.1%  .2 1!
03 .1  .2 1! 3 .1 9.1% 30.1%  .2 14
5 3

 678#  '  9:(-9;9 . 8"8/


) .16 9.<&16 , .16 9.<=16
> .16 9.?&16(
58    9
  89; 
@ABCDE@FE    
9  9 +  (
5 9 GHIJK 8 L89; M9" L 8 9
   (
- M9

678#&':9N78&'#: 8 67N(

OPOQRSTOUVWXU Z  s 9t 9


Y Z678&   9: N78 & 9 .:967N r9 8  ; L   
.( s 9 + t
Juv ; L    t
U Z78 ;    9#:9 9 + s(
!78 
 #  9[:
7!\
V Z] 78 & . ^: _78[   &:9] 7_
89 "9 99 .9^\
 5   9 /
) Z67N 8 `a6b7`aNb(
, Z`a6b7`aNb 8 67N4
EBcE@FE
Zd 78# $ ': 9e 78 # &  $ ' :   " 89 ; L    d 
99    e(f 9L 89 d 9ghigjke(
3 69lImlGn o ; L    p9o(
Z69 N    6qN(


 

!"#$%&'"%())*(++,!(!)!,-.!)/
)0.1,-)23,4 5)6,)!!.0)/
7,8.-9 ),:--*,-!5.1)1*!;!6<+,5-61*(!)4/
=,(+.0)()-*0))51),:!0>(?!)/
0123422156 2472892
@()+5))5,:)).!)A
B .0)4())51),:2C9D3/
B !0)())51),:()-:/7,8.- 2E9;9<3F2E9;9<3/
IeUfgh\O ijklmnopq
G()--)51)),:2E9;9<3/
)51)),:2E9;9<3A 49.0)
2E392;392<3 2)51))+,!(!(!>,!-.!3
2E9;392;9<392E9<3 2)51))+,!(!(!>*,-.!)3
2E9;9<3 2!0)())51),:()-:3
H)51))(!--/
IJIKLMNIOPQRP ,!)*r^)u w vTs
*)t,5)-?)
S 5,:-)T r -.!),:u
-),-.!),:vTs
U 2E9;9<9V3F2E9;9<9V9W9X3
Y 2E9;9<9V3F2V9<9;9E3
Z 2E9;9<9V3F2;9<9V3
[ 2;9V9X3F2?!!5.1)3
\ ]F] :,!0)]
O ^:_ `2<9V9W39-())51)),:_ *(++,!(!8+-0A
U ,!-.! Y *,-.!) Z -.!)/
P ^:a `2E9;9<9V39-())51)),:a *(++,!(!8+-0A
U *,-.!) Y -.!)/
b
5,)] !6c *,))/,0!6+,.-A
]`c (:]Fc !6cdddddd/
012 3453 6789
1
x stTXusTVYTWsXvuvwYTsY
99    9 7 !94"9# ! 8  
!9
9$ 
9 %74"9&9$' 9# ! 8  "!!
9$ (
)  ! 89 794"9
9$* 8!   9 8 $* + * 8 ,-. %(
58  74"9#!9  8 /01234251/60 9%(
58 7-89:;9<87=- > ?9@! 8  9    >8!89  * 8
 ?,-.@(
58 !   ! ?9@!> ! ?A@(
Tygz{|}[ ~€‚ƒ„…†
BCDEF#9GHICJFK#! 8  AG(
C9F9 !* 8 9GAGCF#
Tygz{|}q ~€‚ƒ„…†
?9  LM#9@9  LN#!?A@(
)89  ?A@
  O
?LMPQRLM#9@LMPRSLN#
3?A@LMPR#
58!
  99  LM9LN(
TUTVWXYTZ[\
] ^! 8  ?A@  ?_`abc#9@_d`ef#
Z ^ h * 8  9   > ! 8 > ‡ˆ‡31‰2019i * 8  9
   > ! 8 > ˆ364ˆ25163(
g ^! 8  h 9i(
j ) ! > 8  h Aik
[ h  !
 Q  89LR#9l9  LR#m
g ! 8  h 9l
j ! 8  h Al 9!noh Alpk
q 
! *   89LN#9l9  Pr#m
g ! 8  9l
j ! 8  Al 9!noAlp(
0120 3456789
 
 8859989
98
  !"#$
% &68
&! &68
&$
' ()*87*9&85686
9+,, ()*87*9&85686
9+,,

k efMghQihijRMeR
259-./0.012342566785998&6**9*9(988556
95097:
1*9(98;855667<93/.=>-?3?859)&5667:
259)&9856678
8895@7:
259)&8898(669)7;A(;B8599*9(98&8598898859
9(CB
6A9*9(9885556C9;99*896889:
D
9E6(7*9)779 5F#G667F!":
2&65@789&
8*88599*9(98&;855 2599*9(98
667859
)88599*9(98*896889 8956C98
#G!" 
)8&
(
859/.43923=4/0.
H96859*8859)(;9

69
 &85998:
5@7F!"#G
I966C686B9&85(98568568859)(;9
89

)86
98599*9(98&859/.43923=4/0.&5
667:05 7F:
Mlmnop qrstuvwxy
J&KFG66LF!"#$*885998
 K@L %K L
H99C9
*86
 K@LFG!"#$ 76
8)*6
9*9(98
F!"#$G &698889:
% K LF

MNMOPQRMSTP
U D65@7 &

 5F!"667F"#$
% 5FVWXYZ667F[W\Y]
' 5F!"#$667FG
^ 5F_`ab667F`cbd
  
  !"#$%$&$'()*+!"#$,$-$%$.($/)*0
1 2+ 3 4+
5 6!",$%$7$.()*8!"9$&$7$##$#9($/)*0
1 628 3 648:
; <!"&$'$#=$#,()*>!"%$7$.$##$#9($/)*0
1 <2> 3 <4>:
8 cdeefghijhkl
))?@ABC*B/)DE/FF0
GHIJKLMNOMKP/FFE/QQ@)/)FBB@BC)FEQ@)F/*/)D:
R@SCB$Q@)F/*F !",$%$7($E/Q/FFTAF@F@BBF/)DB*/D/
)TCAF=$#$,$9$-$%$&$7$'$.:
)/FQFT)/UFBF/F V!"=$#$,$9$-$%$&$7$'$.(:
WXKHHYINZLN[F@EFB/@)F/AE)FF:T)/UFBF/FF)*A?
Q)DB)*@FFF)*A?Q/QBFE//)/:
R@ !",$%$7()*V!"=$#$,$9$-$%$&$7$'$.($
\))*/DC/F0
r q y v
s z w
{ x
p t u

01234325 46778309
0
B/Q]@)/Q@)@B@**C@)F/@)@\))*/DC: 86 m
,=@BF]*E?@E)Q@*@D@A@:
/)@C/@)/FF@*@)@XKHHYINZLN[E/QQ@)F/FF
@E@@UB/)DQ/QBFE//)Q)DB:
/QB^ F)F@BE@@E)*@D
)*Q/QB_ F)F@BE@@E)
Q:Q/QBF@UBAQTFF@C@B@E)
A@Q`ab*@D:
n o

p
0120 3456789
 

 086
885998
68
9 8!98995"96579
#
97969
88596$
6
% &98'("55
9$'859)96$
6
97
998797*9"5"+
, 6$ , 6 68 , 6$66 68
, 9859
6$
6 68 , 6$-!886 68 , 6 68-!886$
, 6$
6 68
_ijklmn opqrstuvw
49
859$*9$8!-9
./0/1/2/3/!784
5986787
9!-9
*9856496:78'68
';9
 08689859!<9
6*98
= 588599*998'66:
> ? @ 6A: @@ 6B:
 &**!8
689859986)96$
6
 C78./0/1/2/3/D/;/E/F/49
= 6781/2/D/E96:780/1/2/;9
> @6A:781/29 @@6B:780/1/2/D/;/E9
 ~
x| y &86$968
€ ‚ 7!89*998859
 ƒ 89
98859
{ } )96$
6'
8
z „

?
GHIJKLMN"9 6998568859OPQMRSMTQOUP' x y
8"98
97
9989-(85989
98
<9
*67
'8598" 
*9 xAy
z
259VPOUP'8598"985"8599 x y
)96$
66*$9 xBy
W9*998859! !*-9998
859
859

-8598&8!8*9"85XYZ[X\Y]^[ z
'859 
*9
_`_abcd_%ef
 49
85998C78./0/1/2//49/g782/D/;/E/49/6h780/3/;/F/49
 ?+ @ gAh @@ gBh
= &**!8
689859986)96$
6
012 3453 6789
21
 hl o i   8  9 98 8  
m   !"
k n p # $ % & '
e j q

( "*+.„…†/‡''''''.ˆ02 *+9 
 ,…8-89   89.‡09
)*+,9-  ,./0'
 1 8    ,2 9)'
  3 2 $) 33 2 &) 333 452 $)6 37 452 &)6
! 89 9 8  9 9 9'
: 39

 "*+;<=>?@ABCDE0 F*+<=?ACE0 9


G*+;<>?@AE0'
 1 8   3 F$G 33 F&G
 89 9 8  9 9 9'
VWXYZ%[ \]^_`abcd
H - I   8 89  
  !
r s r s r s

e e e
 58     J'  58    I 8 ! 58      8
 J9 K'  J  K'
L M
9 9  9 9N 8  9 9  f g
89 8 8-8-9  89 O99 
 "  F e
! G #  F
%  G PI 8F9G tuvwxyz{|ty}|~€
Q FI9  G R GI9  F |ww‚vy}uyƒ~
3  8 F G'
S M
9 9  9 989 8  h i

   8  I 
 T   U
! UI9  T # I 8U9T e
%   8 U T PU T'
0120 3456789

 9
9
859569
9
  
L M L M L M

K K K
 
L M L M L M

K K K
! " #
L M L M L M

K K K
<=>?@A BCDEFGHIJ
259$96
6%58597
87&6'9'
8598(986&6)1658
97
9989 Q R
79
)*
97
9988598(9857&6'
6+986&&6,
97
9988598(985
7&6'89)
 -%6'8(986
9859
9859&6. K
 -%6'8(987&6'
#6+986&& ##89
###6+986&&689 #/ 9859
6+986&&
89
/ 6+986&&(8889 /#6+986&&
89.
259(%9

6+986
9(80
965
Q R
9)
STU SVUSWU 2593 %968568N
K SXU 8(989
98
9859
98O
P)
 259
96
9 1232425678 8(98859&6)
 #259
96
9 12369 8(9857&6'6+986&&)
##259
96
9 3246:7 8(9857&6'89)
###259
96
938(9857&6'6+986&&689)
#/ 259
96
958(9857&6'9859
7
8)
/ 18(987&6'6+986&&(8889)
/#123246:;7&6'6+986&&
89)


  
  ! "#$%&'()'*'+','-'.'/&"0
12'3'*'4'5','6'7'/&8
9 :$;('1"0"<""0$#=8
> 0#$""0%;($"%8
? 0#$""0%;1$";@8
A B="!(C1D
E F"@0'@$"0%;"00#$"D
G <""0$#="H"!"H8 m n
I"@"0"0J"8
K"@"0"0L"H8 opqrossroptr
9 M@=";H"!"HD l ostr
> M@=";H"!"H"0"0N
OJ" OOL"H
OOO%J""0L"H OP J"%"L"H
P "$J""L"H POJ"L"HD
QR S ;H%$"T$U"VW==%8<""
0$#="#$0@@;%0=$""1 u v
"0%""$X8 owrospror
x
9 L$"0.8 l opr
> M@=";!H%U==%;N
O%0=$"" OO%""$ OOO%0=$""%"%""$
OP %! P "$!D
y1z4{0|5}51|3~635€9610
Y =" Y =; Y Z
Y $"H$" Y ==% Y 
Y % Y "$" Y "$[
Y <""0$#=
012314516789
Q \$"#"$"'$="!!N
9 =$!]@$H"^_ > $="=%%@"W^"0`_8
a b([""=%%@"W^"0V^&8
9 b$="!(8 > L$"0,c(d8
e (W'`'f'gg&"01V'f'h'g^&8
L$"0$%[!.!@$HN
9 .i( "0.i1 > .i( "0.jVi1
? .i1 "0.kWi(


 

 !"#"$"%"&'()* $"+"&"!",'-./*"/0()%()+-
1 /23
4 5202/,2(67'8 0099((5:2(67'
; 05(5:'8 ))(5:'
p
= !>6 """----">7'"W 0099((5:2(>7' ()
q /U/>7'-
4 c025(/F G W GG q GGG WHq GM WIq
; .22(C(()0D5-
<
= !">"6","?"#"$"&"@'"A >"6"#"&"@'()
B !"6"?"$"&'-
4 .22A"B()=(C(()0D5-
; E0()F G AHB GG AIB GGG JKAHBL GM JKAIBL-
N O(C(()0D5")D0( g h
(0(DF
4 0(P ; (0(Q
R 0(:P ()Q S 0((0P (Q- f
T C(()0D5(2()0(D i j
U(9(0((U205U(2-
V(22U0(0()W(22 klmnkon klln
(90(5(20- kmn
4 XY5(Z2U(9(0(3 f
; XY5(Z2U(9(0(F
GU0(0 GG(90(5(20
GGG:U0(0()(90(5(20
GM (0U0(0((90(5(20
M (90(5(20:(U0(03

012314516789
[ \0(D(0("2025(//22F
4 ](5::Y(!7()#7 ; /U/!&-
^ _0U//22Y0(D3
4 >6` 05(5:2(>7' ; 6#à ](5:'
R #6` 5202/$'
b cd  5202/62(,7'()e  5202/,2(,7'-
4 c0d ()e- ; E0()d He-
R Z()U52F 5202/6'H 5202/,' 5202/------'
012 3453 6789
1
^ _` 34-abc6: 9:: 8 89 ` 8585 9
  :    # 8 :   0
  8  9 98 K L
 !" # $" MNO MPO MQO
% !&$" ' !($" MRO
) *" J
+ 7 8
 ,   -./00000-.01 23495  -.69
734
 89 :  89-.60;
  8   9 9 90
< =89 
  9> 8  89  ?
 # %
S T S T S T

J J J
@ A
9 9  9 989 8  U V

  9>
 B # C 9B
% C  B J

D [ 5:89 8\] 9 8:9 8 9  0


3  9 9   
:9>: 9  W X

:9>89: 0 MYYOMO
Z MYQO
  8 ,9:8 E0 J MNO
# F9> 9 9   
:9>
G:  GG89: 
GGG9 8: 989:  GH : 98  89: 
H 9 : 9   8 ,   HG I95 :>  8 ,  ?
Real Numbers
Measurements
Scientific Notations
Integers
Rational Numbers
REAL NUMBER SYSTEM, MEASUREMENT AND SCIENTIFIC
NOTATION

The picture below shows the mathematics at work. operations in the


real number. Other photo shows a balance scale for measurement. Students
U
measure the length and width of a table.

I
Mark, Jomar, and Raul wanted to treat Christine to a Burger house.
Each of them gave P85.00 for a total of P255.00. They bought 3 or 4
breakfast value meals which cost P75.00 but a value-added tax (VAT) of
P22.50 was charged.

How much change was received by each of Mark, Jomar, and Raul?
T
Hence, each of them paid P75.50. Computing gives:
P75.50+ P75.50 + P75.50 + P22.50 = P249.00

Mark Jomar Raul VAT

What happened to the other P2.00?

After this unit, you will be able to answer this question. This unit deals
with the development of the Real Number System. Properties of real numbers
and the laws governing operations on real numbers are presented so that you
I
have the basic tools necessary to understand the concepts of real numbers.

Almost everyday, you are confronted with questions like “How far is
your house from the school? How big is the ball? How long is the edge of the
table? How many books do you have? To answer these questions you need to
possess some measurement skills.

2
Measurement of length, weight, volume and temperature are important to everyone’s
life. Other measurements that are of equal importance are related to water and electric
consumptions. These are measured with special units.

Measuring is never exact and you will need to decide how accurate the measurement
should be. You will make use of some standard unit of measurement. However you need to
learn not only to measure with the proper unit but also to estimate with some degree of
accuracy. It is true many people go through life without knowing much about measurement.
Would you want to be one of them?

Measurement is also discussed in this unit in order for you to acquire the
measurement skills and use them in your daily life.

Did you watch the boxing encounter between Manny Pacquiao and Eric Morales
where the National Anthem was being played live and also broadcast over a radio system?
You heard it over a radio sooner than you did in person. This is because radio waves travel
300,000,000 meters per second.

Large numbers like 300,000,000 can be written in a much easier and shorter way.
This is by expressing the number in scientific notation.

Scientific notation is very important to engineers and scientists as well as to


astronomers as it saves one from unconsciously omitting a zero or misplacing the decimal
point in writing large or small numbers.

3
Real Number System

Natural numbers or counting If you consider the corresponding


numbers are the numbers you have learned positive and negative numbers, you have the
first. These are the numbers 1, 2, 3, …. Set of Integers. Below is a diagram of the set
Later you learned how to add, subtract, of integers:
multiply, and divide these numbers.
However, dividing these numbers such as 20 Integers
÷ 4 or 42 ÷ 6 resulted in whole numbers as
quotients. To name the result when a
number is subtracted from itself, the whole
numbers, 0, 1, 2, 3, … were introduced. Negative Zero Positive
Integers Integers
In arithmetic, whole numbers and
zero are considered problems in this system Knowing the natural numbers, whole
of numbers. You cannot subtract a large numbers and integers, dividing numbers such
number from a smaller number to get an as 8 ÷ 3 or 15 ÷ 4 is still a problem in the
answer that is zero or greater. To solve this number system since these numbers will not
problem, mathematicians introduced result in whole numbers as quotients. Thus, a
negative numbers or numbers less than zero. new set of numbers known as fractions was
For every positive number there exists a introduced to give meaning to the result of
number that is the negative of the positive such division.
number, more popularly known as the
opposite. If you include the entire set of integers
and positive and negative fractions you have
Thus, to name the result when a the Set of Rational Numbers.
bigger number is subtracted from a smaller
number, the integers were introduced.

4
Below is a diagram of the set of rational Rational Numbers
numbers.
A number that can be written as a
Rational Numbers quotient of two integers where the
denominator is not zero is a rational number.

Example 1. Express each rational number as


a quotient of two integers.
Non-Integers Integers
a) 12
6
b) 3
11
c) 25%
Positive d) 0.7
Negative Zero
Integers Integers/
Counting Example 2. Express each rational number as a
Numbers quotient of two integers.
e) 4 apples to a dozen apples
A rational number is a number that can f) 80 minutes to 2 hours
be expressed as the quotient of two integers
Solution:
1
such as -3, , 0.5. a) Any whole number can be written
3 as a fraction with a denominator
The counterpart of the set of rational of 1.
numbers is the set of irrational numbers. 12
Irrational Numbers are those which cannot be 12 =
expressed as the ratio of two integers such as 1
b) Any mixed number can be written
π, 2 , 3.012…. as a fraction.
6 39
The rational numbers together with the set 3 =
of irrational numbers comprise the entire Set of 11 11
Real Numbers. Below is a diagram of a set of
real numbers. Multiply 11 to 3 to get 33 then add the
numerator 6.
Real Numbers
Recall
c) Percent means by the hundreds.
Any percent can be expressed as a
Irrational Rational fraction.
Numbers Numbers
To change percent to fraction, write the
given number over 100, then simplify
25
25% = write over 100 and
Integers 100
Non-Integers remove the % sign
1
= simplify by dividing both
4
Negative Zero Positive numerator and denominator
Integers Integers/ by 25
Counting
Numbers

5
d). A decimal number can be expressed as a Recall:
fraction. The number of decimal places in the Example 2:
given number indicates the number of zeros in d) One dozen = 12
the denominator 4apples 4 1
= =
12apples 12 3
7
0.7 = write 7 over 10 if the
10
given is tenth
e) 60 minutes = 1 hour
7 120 minute = 2 hours
0.07 = write 7 over 100 if the
100 80 8 2
= =
given is hundredth 120 12 3
7
0.007 = write 7 over 1000 if the
1000
given is thousandth

Let’s practice for mastery 1

I. Express each number as a quotient of two integers and determine if each is


rational. Explain your answer.
1. 42
1
2. -6
7
3. -2.6
4. 23%
5. 0.3

II. Express the following relationships as fractions.

6. 5 oranges to a dozen oranges


7. fifty centavos to one peso
8. 3 days to one week
9. 45 minutes to 1 hour
10. 3 months to 1 year

Let’s check your understanding 1

I. I. Express each number as a quotient of two integers and determine if each is


rational. Explain your answer.

1. 8
3
2. -8
4
3. 6.7
4. 67%
5. 0.18

6
II. Express the following relationships as fractions.

6. 12 mangoes to one dozen mangoes


7. 90 centavos to two pesos
8. 12 days to one week
9. 35 minutes to two hours
10. 12 months to 3 years

• After answering the test, check your answers with those on the answer key page.
• If your score is 7 or higher, you may proceed to the next lesson; otherwise, read the
lesson once for the missed items.

Irrational Numbers Numbers like 3 , 5 , and 7 are


irrational numbers because there is no number
Numbers that cannot be written as a when multiplied by itself equals 3, 5 and 7.
quotient of two integers are called irrational Hence, their exact values cannot be expressed
numbers. In decimal form, these numbers as either terminating or repeating decimals.
are nonterminating and nonrepeating. However, you can use a calculator or the table
of square roots to find their decimal
Examples: approximation.

1. 2 no exact value for 2 since Numbers whose roots cannot be


there is no number when multiplied by itself extracted are not the only irrational numbers.
For example, π is an irrational number which is
will give exactly 2. 2 is read as “the square
approximately 3.1415926.
root of two”.

2. 17 no exact value for 17


3. 3.666… non repeating /
4. 7.012… non terminating
decimals

Let’s practice for mastery 2

Determine if the following numbers are irrational or rational.


1. 48 4. 72
2. 20 5. 121
3. 64

7
Let’s check your understanding 2

Determine if the following numbers are irrational or rational.

1. 32 4. 80
1
2. 576 5.
4
3. 24

• After answering the Test, check your answers with those on the answer key page.
• If your score is 3 or higher, you may proceed to the next lesson; otherwise, read the
lesson once more for the missed items.

Decimal Form of Rational Numbers Example 2:


Express each rational number as a
Since every rational number can be decimal.
expressed as a/b or a ÷ b, where a and b are 7 12 23
a) b) c)
integers and b ≠ 0, we can divide a and b to 6 99 7
obtain a decimal number. If the remainder Solution:
is zero the decimal is called a terminating 7
decimal. a) 1.166
6
6 7.000
Example 1: 6
3 1.0
Let us write as a decimal.
8 6
Solution: 40
36
0.375 40
8 3.000 Move the decimal point 7
24 straight up add as many Therefore = 1.1666…
6
60 zeros as necessary. 12
56 b) 0.1212
40 99
40 99 12.0000
0 99
210
3 198
The division shows that can be 120
8 99
expressed as a terminating decimal 0.375. 210
198
If the remainder zero is not reached 120
when dividing the numerator by the
denominator, continue to divide until the Therefore, 12/99 = 0.1212…
remainders begin to repeat.

8
The dots shown in the quotient indicate
23 that the decimals continue without end.
c) = 7 23.
7 Hence, the decimal quotients are repeating or
non-terminating decimals because the same
digit or block of digits repeats indefinitely. A
bar is used to indicate the block of digits that
repeat.

Solution: 3.2857 7
a) = 1.1666…
6
3.28571 = 1.16
7 23.00000
21 12
b) = 0.121212…
99
20
14
= 0.12
60
56
23
40 c) = 3.285714285714…
35 7
50
49 = 3.285714
10

23
Therefore, = 3.285714285714…
7

Let’s practice for mastery 3

Write each rational number as a decimal and determine if it is a terminating or


repeating decimal.

1 2 3 7 5
1. 2. 3. 4. 5.
4 3 11 10 12

Let’s check your understanding 3

Write each rational number as a decimal and determine if it is a terminating or


repeating decimal.

89 4 9
1. 2. 3.
100 11 2
• After answering the Test, check your answers with those on the answer key page.
• If your score is 2 or higher, you may proceed to the next lesson; otherwise, read the
lesson once more for the missed items.

9
Decimal Form to Fractional Form Fractions should always be reduced to its
lowest term. To reduce fraction to its lowest
In the preceding lesson, we changed term, divide both numerator and denominator
fractions to decimals. How about the reverse by their greatest common factor (GCF).
process, that is changing decimals to fractions?
Does a terminating decimal represent a rational Example:
number? Can decimals be written as fractions? Write 12 in lowest term.
Find out. 18
Solution:
Example 1:
Write each terminating decimal as a Find the GCF of the numerator and
fraction. denominator.

1. 0.9 3. 8.2 The GCF of 12 and 18 is 6.


2. 0.536 4. 2.25 12 12 ÷ 6 2
= =
18 18 ÷ 6 3
Solution:
9 Greatest Common Factor
1. 0.9 =
10
536 67 If the factors of the numbers 30 and 42 are
2. 0.536 = or
100 125 listed, the numbers 1, 2, 3 and 6 appear in both
225 9 lists.
3. 2.25 = or Factors of 30: 1, 2, 3, 5, 6, 10, 15, 30
100 44
Factors of 42: 1, 2, 3, 6, 7, 14, 21, 42
If you notice from the examples, if a
terminating decimal is given, move the decimal These numbers are called common factors
point to the right to make it an integer. This of 30 and 42. The number 6 is the GCF and is
will be the numerator. The denominator will therefore called the greatest common factor of
be 10, 100, 1000, and so on, where the number the two numbers. We write, GCF (30, 42) = 6
of zeros depends upon the number of decimal to denote the greatest common factor of 30 and
places the decimal point is moved to the right. 42.

Example

Let’s practice for mastery 4

Express each terminating decimal as a fraction.


1. 0.4 2. 0.45 3. 0.225
4. 0.63 5. 0.984

Let’s check your understanding 4


Express each terminating decimal as a fraction.
1. 0.15 2. 0.6 3. 0.473
4. 0.2 5. 0.08

9
Set of Integers
Notice that Joan used the number
symbols +20, +15, -18, -12, and +25 which
-5 -4 -3 -2 -1 0 1 2 3 4 5 are called integers. She used +20, +15, +25 to
represent the gains she made, and -18, -12
indicate the losses she incurred.
Joan is a first year student. On weekends, She The set of integers consists of the
earns a little pocket money by selling ice positive numbers, the negative numbers, and
zero.
candies to her neighbor. On some days when it
Uses of Integers
is too warm, some ice candies melt. In order that
they will not be wasted completely she and her Integers can be associated with many
real-world problems and situations. How
family consume or eat the ice candies, thereby
long before 1992 A.D. were the pyramids of
incurring some losses. Egypt built? You can think of B.C. years as
negative and A.D. years as positive. The
following examples will help us represent
Below is the record of her sales of ice
concepts in many fields.
candies for three weekends Here are some examples.
1. deposit of 85 pesos as +85 or simply 85
2. withdrawal of 84 pesos as -84
Date Sales 3. rise of 5in temperature as +5 or simply
5
April 9 +20
4. drop of 5 in temperature as -5
April 10 +15 5. distance of 45 km traveled going east
as +45 or simply 45
April 16 -18
6. distance of 45 km traveled westward as
April 17 -12 -45
7. 40m above sea level as +40 or simply
April 23 +25
40
April 24 +20
8. 40m below sea level as -40

Let’s practice for Mastery 5

Represent the following in integers. Use the (+) sign and the (-) sign.

_____ 1. profit of P700.00


_____ 2. loss of P150.00
_____ 3. gain of 15 grams in weight
_____ 4. reduction of 50 kilowatts in the electric meter reading
_____ 5. overtime pay of P1,000.00

10
Give an answer for each.
_____ 6. If 32 represents a deposit of P32.00, what does –32 represent?
_____ 7. If 280 means 280 feet above sea level, what does -280 mean?
_____ 8. If -25 represents 25 km south, what does 25 represent?
_____ 9. If -20 represents 20 seconds before departure, what does 20 represent?
_____ 10. If -P5,000.00 represents liabilities, what does P5,000.00 represent?

Let’s check your understanding 5


Represent the following in integers. Use the (+) sign and the (-) sign.
_____ 1. A growth of 5 meters
_____ 2. A reduction of 2 kg in weight
_____ 3. The elevation is 1,125 m
_____ 4. It is 40 minutes after takeoff
_____ 5. The jet is flying at 33,750 feet
Give an answer for each.
_____ 6. If -30 represents 30 minutes late, what does 30 mean?
_____ 7. If 20 cm represents an increase in height, what does -20 represent?
_____ 8. If -12 is to the left of zero, where is 12 located with regards to 0?
_____ 9. If 5 km represents going upstream, what does -5 mean?
_____ 10. If P250.00 represents earning, what does –P250.00 represent?

11
Order of Real Numbers
5 5 5 5
A number line can be used to order 2. Consider the fractions , , , .
8 9 12 18
numbers. Note that as you move to the right a. What do you notice about their
of zero along the number line, the numbers numerators?
increase in value and as you move to the left b. Which fraction is the greatest?
of zero, the numbers decrease in value. One c. Which fraction is the smallest?
number is greater than the second if the first
lies to the right of the second on the number When fractions have the same
line. numerator, the greater the denominator the
Let us now study how to determine when a smaller is the value of the fraction
rational number is greater than another.
For example,
5 5 3 3
Comparing Fractions a) < b) >
12 15 5 7
The following models of fractions help
you determine which fractions are greater when Comparing fractions with different
two or more fractions are compared. numerators and different denominators is
5 1 7 easy if you change them first to equivalent
1. Consider the fractions, , , , fractions with the same denominators.
12 12 12
11 3 1 2 4
. (Pls. insert visuals of the fractions) Consider the fractions
, , ,
12 5 5 5 5
a. What do you notice about their What do you notice about their
denominators? denominators?
b. Which fraction is the greatest?
c. Which fraction is the smallest? Equivalent fractions are fractions
having the same denominators.
When fractions have the same
denominator, the greater the numerator, the For example,
greater the value of the fraction.
1 2
For example, Which is greater or ?
5 3 7 12 2 3
a) > b) < (Pls. insert visuals of equivalent fractions)
8 8 15 15 Multiply both numerator and
denominator of each fraction by a certain
number to make them equivalent
1 3 3 fractions.
x =
2 3 6 You can choose whatever method that
is convenient for you.
4 3
> More Examples:
6 6
3 5
Determine which is greater or .
4 6

12
2 2 4 Solution:
x =
3 2 6 3
= 0.75
2 1 4
Therefore >
3 2
Another way to compare them is to change them 5
= 0.833…
to decimal number. 6
1 5 3
= 0.5 Therefore > .
2 6 4
2 1. Write <, =, or > on the blank to
= 0.6666… make a true statement.
3
So, 0.6666… is greater than 0.5. 3 4 7 9
a) > b) <
2 1 5 7 4 4
Therefore > .
3 2
0.6 0.57142

Let’s practice for mastery 6


Which fraction in each pair is greater?
2 3 2 3 2 4
_____ 1. , _____ 3. , _____ 5. ,
5 5 3 4 3 7
5 5 1 4
_____ 2. , _____4. ,
7 9 2 5
Write <, =, or > on the blank to make a true statement.
3 2 7 6 8 9
1. 3. 5.
7 7 9 9 3 4
6 6 8 12
2. 4.
11 12 8 12
Problem Solving:

The UST High School basketball team won 10 out of its 13 games. The Araullo High School
basketball team won 9 out of 11 games. Which school has the better record?
Let’s check for your understanding 6
Which fraction in each pair is lesser?
5 3 7 8
_____ 1. , _____ 2. ,
12 5 9 9
Write <, =, or > on the blank to make a true statement.

8 3 5 7
3. ____ 4. ____
12 15 6 8

13
Problem Solving:
5
Adamson High School won of the baseball games that it played. Magsaysay High School
8
3
won of the total number of its baseball games. Which school had a better record? Why?
4

• After answering the Test, check your answers with those on the answer key page.
• If your score is 3 or higher, you may proceed to the next topic; otherwise, read the lesson
once more for the missed items.

Comparing and Ordering Integers From the number line you will notice
that (-3) is to the right of (-5), so
-5 -4 -3 -2 -1 0 1 2 3 4 5 -3 > -5.

Consider the given number line. Which is greater (-2) or (-4)?


a. Where can we locate 5 with
respect to 2? (5 is located at Of two positive or two negative
the right of 2.) integers, the one located to the right of the
b. Which is greater 5 or 2? (5 is other in the number line has the greater
greater than 2.) value.

In comparing two integers, we compare More Examples:


their positions on the number line. For any two
integers on a number line, the number farther to 1. Draw a number line to illustrate
the right is greater. the following.
a. -1 > -5 b. -6 < -3
Looking at the number line above, c. -2 > -4
5 > 2; 10 > 3.
2. List 8, -4, -12, and -1 in order
Consider the numbers 4 and -1 in the from least to greatest.
number line, which do you think is greater? (4
is greater than -1.)
Solution:
Any positive integer is greater than any
negative integer like 4 > -1; 1 > -4; and 3 > -5. Think of the given numbers on the
number line. The farther to the right the
Zero is greater than any negative integer greater the number. Thus, you have -12, -4, -
for example, 0 > -1; 0 > -4. 1, 8.
3. List -5, 4, 0, -2, 10, -6 from the
How can we compare two negative
greatest to the least.
integers? Which is greater (-3) or (-5)?

Answer:
10, 4, 0, -2, -5, -6

14
Let’s practice for mastery 7

Pick out the greater number in each pair.


_____ 1. (-12), (-18) _____ 2. (-4), (10) _____ 3. 0, 5
Compare these integers. Use < or > in the .
4. 8 -16 5. -9 -4 6. -72 -64
Arrange the following integers in increasing order.
7. -13, -15, -19, -12, -1 ____________________
8. -17, 18, 0, -8, -5 ____________________
Arrange the following integers in decreasing order.
9. -6, 3, -8, 6, -7, -9, 10 ____________________
10. Who is closer to the ground, a man who is on a hill 207 m above sea level or a man who is
in a mine shaft 210 m below sea level?
Let’s check your understanding 7

Pick out the greater number in each pair.


_____ 1. (-3), (8) _____ 2. (-9), (-15) _____ 3. (-5), (5)
Compare these integers. Use < or > in the .
4. -14 -34 5. -8 -13 6. -7 7
Arrange the following integers in increasing order.
7. 25, -10, -32, -41, 15____________________
8. -4, -2, -9, 4, 9, 6, -5 ____________________
Arrange the following integers in decreasing order.
9. -15, 15, 16, -16, -17, 17 ____________________
10. Who did better, a student whose score was 35 below the passing score or one whose score
was 20 above the passing score? What is the difference in their scores?
• After answering the Test, check your answers with those on the answer key page.
• If your score is 7 or higher, you may proceed to the next topic; otherwise, read the lesson
once more for the missed items.

15
Basic Properties of Real Numbers C. Associative Property

Recall several basic properties in Adding three or more real numbers will give
addition and multiplication of real numbers. the same sum no matter how the numbers are
grouped.
A. Closure Property
Examples: 1) (2 + 5) + 9 = 2 + (5 + 9 )
Each pair of real numbers has a unique
7 + 9 = 2 + 14
(one and only) sum which is also a real
16 = 16
number.

Examples: 1) 7 + 6 = 13 2) 4 + (8 + 3) = (4 + 8) + 3
2) 19 + 8 = 27 4 + 11 = 13 + 3
15 = 15
Each pair of real numbers has a unique
product which is also a real number. Multiplying three or more real numbers
will give the same product no matter how the
Examples: 1) 9 × 5 = 45 numbers are grouped.
2) 6 × 5 = 30
 1 1 
Examples: 1)  9 ×  × 18 = 9 ×  × 18 
B. Commutative Property  3 3 
3 × 18 = 9 × 6
Adding two real numbers will give the 54 = 54
same sum no matter in what order the
numbers are added. 2) 3 × (2 × 4 ) = (3 × 2 ) × 4
3× 8 = 6 × 4
Examples: 1) 6 + 8 = 8 + 6
24 = 24
14 = 14
2) 12.5 + 6.2 = 6.2 + 12.5 D. Identity Property
18.7 = 18.7
Any number added to 0 is equal to the
Multiplying two real numbers will give given number. Zero is called the additive
the same product no matter in what order the identity.
numbers are multiplied. Examples: 1) 0 + 9 = 9
2) 28 + 0 = 28
Examples: 1) 9 × 3 = 3 × 9
27 = 27 Any number multiplied by 1 is equal to
the given number. 1 is called the multiplicative
2) 6 × 8 = 8 × 6 identity.
48 = 48

Examples: 1) 1× 8 = 8 2) 3(5 + 8) = (3 × 5) + (3 × 8)
2) 12 × 1 = 12 3(13) = 15 + 24
39 = 39

16
E. Inverse Property Operations with Real Numbers
Fractions
The sum of a real number and its opposite is A fraction whose numerator is greater
0. The number opposite the given real number than or equal its denominator is called
is called the additive inverse. improper fraction. A proper fraction is a
fraction whose numerator is less than its
Examples 1: denominator
1) -8 is the additive inverse of 8. Examples:
Proper Fractions Improper Fractions
2) 12 is the additive inverse of -12. 1 2 5 10 17 5 8 12 18 17 5
, , , , , , , , ,
4 3 9 12 18 2 3 5 15 4 5
Examples 2:
1) 5 + (− 5) = 0 A number that is expressed as the sum
2) − 10 + (10 ) = 0 of a whole number and the fraction is
called a mixed number.
The product of a real number and its Examples:
reciprocal is 1. The reciprocal of the given 1 1 3
4 , 5 , 7
number is called the multiplicative inverse. 2 4 4
These are sometimes referred to as
Examples 1: numbers in mixed forms.
1
1) is the multiplicative inverse of 4. To change an improper fraction to
4
2) -8 is the multiplicative inverse of mixed number in simplest form, we divide
1 the numerator by the denominator and
− . expressed the remainder as a fraction.
8 In general, the result of dividing can be
written as follows:
Examples 2:
1 P R
1) 12 × =1 = Q+ , where P = Any real number
12 D D
1 except zero
2) − × (− 5) = 1 D = Divisor
5
Q = Quotient
R = Remainder
G. Distributive Property
Example:
14
Multiplication is distributive with Change to a mixed number.
respect to addition. 3
Solution:
Examples: 14
= 14 ÷ 3 = 4 remainder 2
1) 7(6 × 9 ) = (7 × 6 ) + (7 × 9 ) 3
7(15) = 42 + 63 Here it is written as 4
2
105 = 105 3
14 2
Therefore, = 4 in mixed number
3 3
To change mixed number to an
improper fraction , rewrite it as an

17
indicated sum as shown in the following
examples . To add/subtract fractions/mixed numbers
3 with different denominators, change them to
Example : Change 1 to an improper fractions having the same denominators by
8
fraction finding the least common denominator LCD,
Solution: then apply the rules in adding/subtracting
3 3 8 3 11 8 fractions with the same denominators.
1 = 1+ = + = since 1 =
8 8 8 8 8 8
Example:
Another solution is shown below, 5 2
1. + LCM of 8 and 3 = 24
8 3
3 (8)(1) + 3 8 + 3 11 5 2 15 + 16 31 7
1 = = = + = = or 1
8 8 8 8 8 3 24 24 24
Notice that this is done by multiplying the 24 ÷ 8 = 3 3 x 5 = 15
denominator by the whole number and adding 24 ÷ 3 = 8 8 x 2 = 16
to it the numerator , then copy the numerator

Addition/Subtraction
6 3 24 − 3 21 3
The diagram below illustrates that 2. - = = or
5 2 7 7 2 5 7 28 28 28 4
+ = , − = LCM = 28
9 9 9 9 9 9 28 ÷ 7 = 4 x 6 = 24
5
2 28 ÷ 28 = 1 x 3 = 3
9 9
3 7
3. 5 +7
10 15

Solution: LCM = 30
This example suggests the following rules for 30
= 3 ; 3 (3) = 9
adding and subtracting fractions with the 10
same denominator 30
= 2; 2(7 ) = 14
15
To add/subtract fractions/mixed number 9 + 14
3 7 23
with the same denominators, add/ subtract 5 +7 = 12 = 12
the numerators and write over the given 10 15 3 30
denominator. 3 7
5 +7
Example 10 15
3 1 4 4 7 11
a) + = c) 3 + 1 =4 or What is the difference between least
5 5 5 9 9 9
2 2 common denominator and least common
4+1 = 5 multiple?
9 9 Given:
a. 3, 6, 9
5 2 3
b) - =
8 8 8 The least common multiple of 3, 6,
and 9 is 18.

18
The least common multiple (LCM) of two or Then get the product of the numerators and
more numbers is the least or smallest write over the product of the denominators
number that is the a multiple of two or more 15 5 6
non-zero given number 2. x = =
24 24 18
1 1 1 The common factor of 6 and 24 is 6 and
b. , ,
3 6 9 the common factor of 15 and 18 is 3.
5
Multiplying Fractions
Therefore, the product is in simplest
24
To multiply fractions, write the form.
product of the numerators over the product of 3  2 23 14
the denominators then simplify 3. (5 ) 4  = X .
Example: 4  3  4 3
2 5 10 5
1. x = or 161 5
3 6 18 9 =1 or 26
When the numerator and 6 6
denominator of either fraction have a
common factor, you can simplify before Dividing Fractions
you multiply. To divide fractions, multiply the
dividend by the reciprocal of the divisor. You
For example, can use cancellation if you wish to.
12 7 1
1. x = Example:
14 36 6
The common factor of 12 and 36 is 12 so
3 2 3 3 9 1
divide both by 12. 1. ÷ = x = or 1
The common factor of 14 and 7 is 4 3 4 2 8 8
7 so divide both by 7.
2 2 8 32 8 3 1
2. 2 ÷ 10 = ÷ = × =
3 3 3 3 3 32 4

Let’s practice for mastery 8

A. Perform the indicated operation. Simplify your answers.

5 1 1 1
1. + = ______ 6. x = ______
12 12 2 4
5 1 3 4
2. - = ______ 7. x = ______
6 6 8 7
1 2 3
3. + = ______ 8. x 8 = ______
9 3 4
3 1 2 5
4. + = ______ 9. ÷ = ______
5 7 3 6
7 3 1 7
5. - = ______ 10. ÷ = ______
8 16 2 16

19
3 4 1 4
11. 3 + 1 = ______ 14. 3 × = ______
5 5 2 7
7 3 3 1
12. 5 − 1 = ______ 15. ÷ 2 = ______
8 8 4 2

5
13. 6
9
7
+
9
___________

B. :Perform the indicated operations:

1. 8 +3-9 x 2 ÷ 3, in this type of problem where expressions are written


without grouping symbols, to simplify this we use Rule of MDAS. Do all
multiplications and divisions in order from left to right then do all additions and
subtractions from left to right.
Solution Try the following:
8 + 3 -9 x 2 ÷ 3 2.) 72 – 24 ÷ 3
= 8 + 3 – 18 ÷ 3 3) 8 + 3 – 9 x 2 ÷ 3
= 8+3–6 4) 18 – 3 x 4 ÷ 3
= 11 – 6 5) 9 ÷ 3 x 2 + 8

= 5.
Let’s check your understanding 8

Perform the indicated operation. Simplify your answers.

6 1 5 4 1 2 1
1. + = 6. x = 11. 3 13. 6 + 1 =
18 18 8 8 5 5 5
12 5 6 12 4
2. - = 7. x = + 2
15 15 8 24 5
2 3 4 1 1
3. + = 8. x 7 = 14. 3 ×9 =
3 4 7 2 3
5 2 3 3 3
4. + = 9. ÷ = 12. 5
12 3 7 7 4
4 1 5 3 1 2 1
5. - = 10. ÷ = - 2 15. 1 ÷ =
9 6 6 5 4 3 6

• After answering the Test, check your answers with those on the answer key page.
• If your score is 7 or higher, you may proceed to the next lesson; otherwise, read the lesson
once more for the missed items.

20
Let’s practice for mastery 9

Problem Solving:

Solve the following problems using the operation of fractions. Read and analyze the
problem before solving to determine what operation on fractions should be applied.

1. Last year was a bad year for Mang Jojo. Typhoon Juaning ruined 3/8 of his cornfields and
1
a few days later another storm, Milenyo destroyed an additional of his cornfields.
10
What fractional part of Mang Jojo’s cornfields was destroyed by the two typhoons?

3 1
2. Last Monday, John was on duty for 9 hours. During that time he drove for 6 hours.
4 2
How long was he not driving while on duty?

2 1
3. Elsa found of a cake in the refrigerator and ate of it. What part of the whole cake
3 2
did she eat?

1
4. How many boxes will be needed to contain 3 kilograms of butter if a box holds
4
kilogram?

7
5. Ryan is making gold and silver earrings. The length of the silver wire hook is inch.
16
5
The gold earring is inch long. Find the total length of each earring.
8

Let’s check your understanding 9


Solve the problems.
5 1
1. From a piece of ribbon m long, a -meter piece was cut. How much ribbon was left?
6 5

2. Wilma is making curtains. She needs two curtains for each window. Each curtain
3
requires m of materials. How many curtains can she finish with 9m material?
4

21
Operations on Integers To subtract two integers, add the
Absolute Value opposite of the subtrahend.
(12) – (4) = 12 + (-4) = 8
(-15) – (-3) = -15 + (3) = -12
• • • • • • • • • • • (18) – (-6) = 18 + (6) = 24
-5 -4 -3 -2 -1 0 1 2 3 4 5 (-20) – (5) = -20 + (-5) = -25
The distance from 0 to the graph of a
number is called the absolute value of the Multiplication and Division of Integers
number thus 4 and -4 have the same absolute The product of two integers with the
value, 4. The symbol for the absolute value same sign is positive.
of a number n, is n . We write 4 = 4 or
a. (5) (6) = 30 c. (-2)(-3)(-4) = -24
− 4 =4. The absolute value of zero is zero
b.(-7) (-8) = 56 d. (5)(2)(3) = 30

The graph of -4 and 4 are the same The product of two integers with
distance from zero, but in opposite different sign is negative.
directions. We call such a pair of numbers
opposites. Thus -4 is the opposite of 4 and 4 a.(-6) (8) = -48 c. (-3)(4)(6) = -72
is the opposite of 4. b.(9) (-7) = -63 d. (2)(-4)(-3) = 24

Addition/Subtraction of Integers The quotient of two integers with same


To add two integers having the same sign is positive.
signs, get the sum of their absolute values
and prefix the common sign. (24) ÷ (8) = 3
(5) + (8) = 13 (-36) ÷ (-4) = 9
(-6) + (-4) = -10
The quotient of two integers with
To add two integers with different different sign is negative.
signs, get the difference of their absolute
values and prefix the sign of the number (-72) ÷ (8) = -9
having the greater absolute value. (35) ÷ (-7) = -5
(-10) + (3) = -7
(15) + (-6) = 9

Let’s practice for understanding 10

Perform the indicated operation.

_____ 1. 8 + (-12)

_____ 2. -20 + (-13)

_____ 3. Subtract (-10) from (-28)

_____ 4. From 49, subtract (-18)

22
What should be in the box to make the statement true?

5. (-3) + =8

6. (-6) (-4) =

7. (32) ÷ (-8) =

8. (-54) ÷ = -6

9. An elevator is on the ground floor. It goes up 8 floors, then 5 down, and then 4 up. What
is its final position from the ground floor?

10. Ryan weighed 65 kg. He got sick and lost 4 kg on the first week, then 2 kg more on the
second week. When he got well he gained 3 kg on the third week and 1 kg on the fourth
week. What was his weight after one month?

11. A deep-sea diver plunged 25 m under water, then 23 m deeper. Then the diver rose 15 m.
How many meters is the diver below water level?

12. How much greater is (-15) than (-25)?

13. Which is greater -19 or 6? How much is it greater?

Let’s check your understanding 10

Perform the indicated operation.

_____ 1. (-15) + (12)

_____ 2. (-16) + (-9)

_____ 3. Subtract (-6) from 14

_____ 4. From 24, take away (-6)

What should be in the box to make the statement true?


5. – 23 = 9

6. (-15) – =0

7. (-5) (9) =

23
8. An airplane flying at 4,260 m from the ground ascends 1,059 m to avoid a storm. Then it
drops 2,115 m and finally ascends 780 m. What is the final altitude?

9. Ma. Jo Ann deposited P2000 in a bank. She withdrew P1050 on the first month, P450 on
the second month and deposited P1380 on the third month. How much money did she have in
the bank after the third month?

10. At 5 p.m. the temperature in Baguio City is 18°C. The temperature in Manila is 27°C.
What is the difference in temperature in:
a) Baguio City to Manila?
b) Manila to Baguio City?

• After answering the Test, check your answers with those on the answer key page.
• If your score is 7 or higher, you may proceed to the next lesson; otherwise, read the lesson
once more for the missed items.

Unit Test I

Answer the following. Solve if necessary.


1. A number that can be expressed as a quotient of two integers is __________.
2. Express “18 oranges to a dozen” in fraction.
7
3. Express the rational number in decimal.
9
4. Express 0.012 in fraction.

Represent the following in integers. Use the (+) sign and (-) sign.
5. 5° below freezing point on the Celsius scale
6. An increase of 6 points in the PBA game
7. A 9-yard loss in football
8. A bank deposit of P500

Write <, =, or > on the blank to make a true statement.

9. 5 3 10. 7 9 11. -15 -18 12. -30 25


6 4 8 10

Perform the indicated operation.

3 2 5 3 5 3 8 2
13. + = 14. – = 15. x = 16. ÷ =
7 7 6 4 12 5 12 3

17. What is the sum of (-5) and (8)?

18. Is the product of (-8) (9) equal to (-6) (12)?

19. What must be added to -18 to get -3?

24
20. If -6 is subtracted from -15, the difference is __________?

21. The product of two numbers is -48. If one number is -12, what is the other number?

22. One number is 18 and the other is -5. What is the product?

23. Mario lost 37.5 lbs. If he lost 2.5 pounds each week, how long has he been dieting?
Represent and solve the problem using signed numbers.

24. For 5 consecutive months, Pedro withdraws P20,000 from his deposit of P150,000 in the
bank to finance his monthly bills. What signed number represents his final financial
condition?

25. In the city of Baguio, the temperature was 9°C above zero at dawn. At noon the
temperature rose by 11 degrees. What was the temperature at noon?

25
Measurements
MEASUREMENTS AND SCIENTIFIC NOTATION

U
N
I
The picture shows that students are learning to measure. Each student
measures a particular object by means of a different unit. What do they use to
measure the blackboard? Their heads and waists?
T
Measurement is very important in our daily life. We need to answer
questions that begin with how long, how far, how much, and so on. All these
questions require some kind of measurement before they can be answered.

The unit of measure used depends on the quantity or item to be measured.


You buy gasoline by the liter, mangoes by the dozen, and ice cream by the gallon.
You get the distance between two buildings in meters and that between two cities or
II
towns in kilometers.
Did you know that the British were The Metric System
the first to have a standardized system of
measurements? In 1970, the French Academy of
Science was asked to establish and develop a
However, this system dates back to the new standard and logical system of
time of the Babylonians and Egyptians when measurement and thus, was born the original
measurements were made in terms of body metric system.
parts. For example an arm was used as the
unit of length. A finger or digit was the A present, in our country, two systems
width of a finger. A palm was the width of of units are generally used, the English
four fingers. A span was the width of three system and the metric system. However, the
palms. A cubit was the distance from the tip metric system is the only system of
of an elbow to the tip of a finger. measurement prescribed by law through P.D.
#187 issued on May 10, 1973 which was
superseded P.D. #187. (Math Time Journal,
vol. VIII-No. 1, July, 2005).

Our present systems of measurements


are based on measurement standards which
make everything consistent. The real
advantage of using the metric system lies on
the ease and convenience of conversion from
one unit of measurement to another.

Approximation of Measurements
Types of Measurement

But the use of the body parts in Measures of Length


determining lengths was not very reliable
since people vary greatly in size. The Meter (m) is the basic unit of length.
English system of measurement was also We use the unit of length when
adopted in the United States. measuring the length, width and thickness of
objects. Also in getting the distance.
However, the English system of
measurement did not gain much acceptance
because there was no definite scheme for
remembering the relationships between the
units. Besides, the computations were often
difficult.
Let’s Do It

Measure the following.

1. Length of a small paper clip


2. Width of a room
3. Length of a ten-peso bill
4. Thickness of a Math book
5. Length of your pencil
6. Length of your pants
7. Width of a table
8. Thickness of your notebook
9. Distance between your bedroom and the living room
10. Distance from the kitchen to the comfort room.

Measures of Weight Measures of Capacity

Gram (g) is the basic metric unit of Liter (L) is the basic Metric unit for
weight. Weight is actually caused by the measuring capacity. Liquid measures are
pull of the Earth on an object. However, in measures of volume. The dry measure is
our daily life, it is weight that we measure. used for measuring grains, fruits, vegetables,
Before, rice used to be bought by the etc.
“salop”. But nowadays, it is bought by the
“kilo,” the popular name for kilogram (kg). You can use the unit of liter if you are
Thus, we buy 5 kilos of rice, one or two getting the capacity of the objects.
kilos of mangoes, etc.
Let’s Do It
Let’s Do It
Get the measures of capacity of the
Get the weight of the following following objects in your house.
objects in your house. _____ 1. amount of milk in a carton container
_____ 1. a glass _____ 2. amount of liquid in an eye dropper
_____ 2. a spoon _____ 3. a glass of water
_____ 3. a cup _____ 4. a cup of coffee
_____ 4. a pair of shoes _____ 5. a can of coke
_____ 5. a can of milk _____ 6. a bottle of softdrinks
_____ 7. a can of pineapple juice
Write the unit (mg, g, kg, t) one should use _____ 8. a can of root beer
to measure the weight of each of the _____ 9. a sachet of shampoo
following. _____ 10. a pack of tomato sauce
_____ 1. a car
_____ 2. a grain of salt Although the Philippines uses the
_____ 3. a loaf of bread Metric System as prescribed by P.D. 187,
_____ 4. a fork
_____ 5. an onion
there are still some commodities or goods You want to sew expensive lace around
which are measured in English units. It is the bottom of a skirt. You would estimate the
necessary therefore, for you to know how to amount of the lace you need to the nearest:
see the measures of the English System. a. 1 mm
b. 1 m
An inch (in) is the basic unit of c. 1 km
length. Answer: 1 m is the most appropriate measure

An ounce (oz) is the basic liquid and Let’s Do It


mass measure.
Measure the following.
Pretend that you want to estimate in
each situation below. Decide how close _____ 1. Your height in feet
your estimate should be. Encircle the best _____ 2. Your weight in pounds
answer. _____ 3. Weight of a can of paint
_____ 4. length of paint brush in inches
1. You want to mail a pair of tennis _____ 5. length of a ruler in feet
shoes in a shoe box. You would estimate
the weight to the nearest
a. 5 ounce
b. 0.5 ounce
c. 50 ounce
Answer: 5 ounce is the most appropriate
measure.

Let’s practice for mastery 11

Choose the most sensible measure.


_____ 1. Length of a small paper clip 31 mm 31 cm 31 m
_____ 2. Distance around a racetrack 2m 2 km 2 cm
_____ 3. Length of a tennis racket 68 cm 68 m 68 mm
_____ 4. weight of a dog 8 kg 8 mg 8g
_____ 5. a can of beans 453 mg 453 g 453 kg
_____ 6. an egg 54 mg 54 g 54 kg
_____ 7. a math book 1.8 mg 1.8 g 1.8 kg
_____ 8. a can of juice 165 mL 165 L 165 kL
_____ 9. a cup of coffee 200 mL 200 L 200 cL
_____10. a bathtub 400 mL 400 L 400 kL

Let’s check your understanding 11

Choose the most sensible measure.

_____ 1. The thickness of a 25 centavo coin 1 mm 1 cm 1m


_____ 2. Length of a safety pin 26 mm 26 cm 26 m
_____ 3. Long-distance run 10,000 cm 10,000 m 10,000 km
_____ 4. a can of soup 305 mg 305 g 305 kg
_____ 5. a sewing needle 380 mg 380 g 380 kg
_____ 6. a pencil 5 mg 5g 5 kg
_____ 7. a paper clip 515 mg 515 g 515 kg
_____ 8. a glass of milk 250 mL 250 L 250 cL
_____ 9. a bottle of orange 500 mL 500 L 500 cL
_____10. a coffeepot 2 mL 2L 2 cL

• After answering the Test, check your answers with those on the answer key page.
• If your score is 7 or higher, you may proceed to the next lesson; otherwise, read the lesson
once more for the missed items.

Conversion of Units of Measure 1. Multiply when changing from a


larger unit to a smaller unit.
To change from one metric unit to
another, we multiply or divide by a power of 2. Divide when changing from a
ten. smaller unit to a larger unit.

Unit of Length (meter) We can also make a converter


10 mm = 1 cm diagram to simplify the changing of units
10 cm = 1 dm of length, as shown below.
10 dm = 1 m
10 m = 1 dkm Multiply by 10
10 dkm = 1 hm
10 hm = 1km km hm dkm m dm cm mm

Unit of Weight (gram) Divide by 10


10 mg = 1 cg
10 cg = 1 dg Example 1: Convert 5.43 meters to
10 dg = 1 g centimeters.
10 g = 1 dkg
10 dkg = 1hg Solution 1:
10 hg = 1 kg
1. There are 100 cm in a meter.
Unit of Capacity (Liter) 2. You multiply because a meter is
10 mL = 1 cL larger than a centimeter.
10 cL = 1 dL 5.43 x 100 = 543cm
10 dL = 1 L
10 gL= 1 dkL
10 dkL = 1hL
10 hL = 1 kL
Example 3: Write the missing unit:
Solution 2: 8,523 m = 85.23 _____

To convert meter to centimeter count Solution:


the spaces from m to cm in the diagram and
move the decimal point in the same The decimal point was moved 2 spaces
direction as many spaces as are indicated in to the left. Therefore, the missing unit is
the diagram. In this case, 2 spaces to the hectometer.
right.
km hm dkm m
m dm cm m

Move 2 spaces to the right. Therefore: Example 4: How many pieces of string can
5.43 m = 543 cm. be cut from a ball of twine 9-m long if each
piece is 4.2 cm long?
Example 2: Convert 6,500 meters to
kilometers. Solution:

Solution 1: 9 m = 900 cm 9 x 100 = 900 cm


Then you divide 900 cm by 4.2 cm
1. There are 1000 m in 1 km. 900 ÷ 4.2 = 214.285
2. You divide because meter is smaller Therefore, you can cut around 214 pieces of
than kilometer. 4.2 cm string from a 9 m twine.
6,500 ÷ 1000 = 6.5 km

Solution 2:

km hm dkm m

Move 3 spaces to the left.


Therefore:
6,500 m = 6.5 km

Let’s practice for mastery 12

Compare the measurements. Write >,< or = for each box.

1. 1000 km 1000 mm

2. 100 cm 1m

Change each of the following to the indicated unit.

3. 924 cm = _____ m
4. 3m = _____ mm
5. 8,643 dm = _____ m

Solve.
6. A coin has a diameter of 23 mm. How many pieces of coins can be arranged such that one
coin is on top of another that would measure 1 meter?

Let’s check your understanding 12

Compare the measurements. Write >,< or = for each box.

1. 1m 10 mm

2. 100 hm 10 km

Change each of the following to the indicated unit.

3. 421 cm = _____ km
4. 758 mm = _____ m
5. 0.4 m = _____ cm

Solve.
6. A 100 m length of rope is cut into 75-cm pieces. How many pieces can be cut?

• After answering the Test, check your answers with those on the answer key page.
• If your score is 4 or higher, you may proceed to the next lesson; otherwise, read the lesson
once more for the missed items.

Converter Diagram for Weight Converter Diagram for Capacity

Weight Capacity

kg hg dkg g dg cg mg kL hL dkL L dL cL mL

Let’s practice for mastery 13

Give the missing units.

1. The capacity of a pail of water is about 575 _____.


2. The capacity of a glass of juice is about 150 _____.
3. The weight of a cotton a 1.2 _____.
4. The weight of a ground pork is 2 _____.
Change each of the following to the indicated unit.

5. 3,840 L = _____ kL 7. 85 cg = _____ mg


6. 0.5 L = _____ cL 8. 6.5 kg = _____ g

Solve the following problems.

9. Sarah drinks 5 glasses of water everyday. Each glass holds 375 mL. How many liters of
water does Sarah drink each day?

10. A key weighs about 10 g. How many keys are there in a bag of keys weighing 2 kg?

Let’s check your understanding 13

Give the missing units.

1. The capacity of a mineral water is about 45 _____.


2. The capacity of water in a swimming pool is 5000 _____.
3. The weight of a loaf bread is 454 _____.
4. The weight of a baby is 3.2 _____.

Change each of the following to the indicated unit.


5. 29 L = _____ mL 7. 28 g = _____ kg
6. 200 cL = _____ L 8. 15 mg = _____ g

Solve the following problems.

9. It takes 4,500 liters of water to manufacture 1 ton of cement. How many


kiloliters of water is this?

10. What is the weight of a baby 15 kg in grams?

• After answering the Test, check your answers with those on the answer key page.
• If your score is 7 or higher, you may proceed to the next lesson; otherwise, read the lesson
once more for the missed items.
Let’s practice for mastery 14

Match the columns. Then write the letters that match the numbers on the correct lines to
solve the riddle.

1. _____ 10 mm E 1L
2. _____ 1 km Y 5m
3. _____ 1,000 mg A 5,000 g
4. _____ 1,000 mL R 48 mo
5. _____ 1 mm B 0.1 cm
6. _____ 4 km H 100 cm
7. _____ 1 kg S 0.001 cm
8. _____ 500 cm M 1m
9. _____ 3.086 km I 5.23 mL
10. ____ 5 kg O 1,000 g
11. ____ 4 yr D 1,000 m
12. ____ 5 m, 23 cm V 1g
13. ____ 1,000 mm L 3,000 m, 8,600 cm
14. ____ 1 m P 4,000 m

____ ____ ____ ____ ____ ____ ____ ____ ____ ____
5 7 8 2 7 12 1 10 3 4

____ ____ ____ ____ ____ ____ ____ ____


6 11 7 5 9 4 13 14
Let’s practice for mastery 15

In 1994, the Third International Math and Science study was given to randomly selected
students from more than 40 countries. Which country obtained the highest score in mathematics?

To find out, encircle the letter of the correct answer. Then write the letter in the blank that
goes with the number.

1. 34.2 km = _____ cm
R. 342,000 S. 3,420,000 T. 000342

2. What would you use to measure the length of a room?


I. meter J. kilometer K. centimeter

3. Which is the shortest?


M. 0.01 m N. 0.1 dm O. 0.001 km

4. Choose the most sensible measurement for the length of a tennis racket.
E. 68 km F. 68 m G. 68 cm

5. 38,902 m = _____ mm
A. 38,902,000 B. 38.902 C. 0.038902

6. 4.25 cm = _____ hm
N. 42500 O. 0.0425 P. 0.000425

7. Which is the longest?


M. 10 cm N. 0.01 m O. 1000 mm

8. Add 3.4 m + 689 cm + 12 mm


R. 10,302 mm S. 10.41 m T. 1,140 cm

9. The length of an average book is


D. 2 mm E. 24 cm F. 24 m

____ ____ ____ ____ ____ ____ ____ ____ ____


1 2 3 4 5 6 7 8 9
Let’practice for mastery 16

A. Estimate the length of each object in centimeters:

B. Estimate the height of each object in centimeters:


Perimeter, Area, and Volume
Perimeter is the distance around a polygon.

Area is the measure of the surface of a Let’s practice for mastery 17


plane. Solve the following problems.

Volume is the amount of space that a solid 1. Find the area of a rectangular
contains. frame whose length is 8 cm and
width is 5 cm.
Perimeter and Area
Plane Picture Perimeter Area Example
Triangle P = a+b+c bxh 3x3
= 2+3+4 A= A=
= 9 cm 2 2
=4.5 cm²
Rectangle l=3.4 P=2 (l+w) A=3.4(1.4) 2. The diameter of a bicycle
w=1.4 =2(3.4+1.4) A= lxw tire is 70 cm. What is its
= 9.6 cm = 1.76 cm²
circumference? ( Use 3.14 for π)
Square s=7 P= 4s A = s² A = 7²
= 4(7) = 49 cm² bicycle tire
= 28 cm
Trapezoid P=b1+b2+ A=(b1+b2)h A=
b3+b4
2 (2 + 3)1.4 3. The perimeter of a
2
2 handkerchief is 100 cm. How long is
=3.5 cm² each side?
Circle C = 2πr A = πr²

Volume
Solid Figure Volume 4. A cubed-ice has an edge of
V = e³ 2.5 cm. Find its volume.
Cube = (1.5)3
= 3.375 cm3
V = lxwxh
Prism = 3 x 1 x 1.5 5. A bedroom is 8 m long, 6 m
= 4.5 cm3
wide, and 3.5 m high. Find the
Right V = πr²h volume of the air space inside the
Cylinder =3.14 (12)(3) bedroom.
= 9.32 cm2

V = 4/3πr³
Sphere =4/3(3.14)(33)
=4(3.14)(9)
= 111.04 cm3
Right V =1/3 πr²h
Circular =1/3(3.14)(42)(10)
Cone =167.40 mm3
Let’s check your understanding 16

Solve the following problems.

1. A rectangular dining table has a length of 2 m and its width is 1.5 m. Find its area.
2. A circular pizza pie has a radius of 75 cm. How big is the pizza pie
3. A throw pillow has a side of 25 cm. How big is the throw pillow?
4. A cubic box has an edge of 15 cm. find its volume.
5. A classroom in Angeles Elementary School is 9m long, 6 m wide, and 4 m high. Find the
volume of the air space inside the classroom.

• After answering the Test, check your answers with those on the answer key page.
• If your score is 3 or higher, you may proceed to the next lesson; otherwise, read the lesson
once more for the missed items.

Water Meter Reading


In some provinces, water is abundant, available for free from springs, wells, and rivers. It is
then considered a free commodity. However, in the cities, water is quite scarce and expensive.
When the different dams in the country dry up, they cannot supply the needs of the consumers.
Water then becomes an economic commodity. In any case, to ensure the abundance of water supply
in the future, measures must be taken to conserve water.

We must be aware that if we do not try to conserve water now, someday we might not have
anything to drink. There will be no more beaches to swim in, no more lakes for fishing, and no more
dams to irrigate the rice fields.

The water meter is simple to read. The face dial of a typical water meter illustrated below
will guide us on how to read water meters.

Note that the dials are read in alternate counter-clockwise and clockwise directions from left
to right. The last figure passed by each pointer is the figure that is read. Consider the water meter
under the Previous Reading. Under the Previous Meter Reading, the last numbers passed are listed as
follows:

Dial 10,000 1,000 100 10 cu. m. Units


cu. m. cu. m. cu. m.
Last Number Passed 2 3 4 7 6

Thus, the figures we get from the Previous Reading are 23476 and the Present reading is
23661. The number of cubic meters of water used is 185 cu. m., which is the difference between the
present reading and the previous reading.

How to Compute Water Consumption

Consider this example:


If the meter readings of Mrs. Vilma Calvario show: Present Reading, 6252; and
Previous Reading, 6225, what is her water bill from October 2, 2006 to November 3, 2006.

The solution to the above problem is presented below.


Step 1 – Present Reading = 6252
Previous Reading = 6225
27 cu. m.
Therefore, the number of cubic meters consumed is 27 cu. m.

To compute for the monthly water bills of a family, MAYNILAD had added some surcharges
needed for the computation of monthly water bill of a certain family.

Let’s practice for mastery 18


Solve the following problems.
1. The water meter readings of four families during the month of May 2 to June 2 are listed.
Compute for the monthly water consumption of each family.
Family Previous Present
Reading Reading
A 46789 46828
B 27610 27723
C 00482 00498
D 02674 02698

Electric Meter Reading


Our present electrification program tries to reach as far as the remotest areas of the country.
As the population grows, there is a need for more electric power.

Electric power is measured in Watts, which is dependent upon the number of Amperes
(current) and the Voltage. In measuring the amount of power, we also have to consider the time. This
is expressed in hours.

The relationship between Power, Current, and time may be described by the rule,

Power = Current x Time


or P = CT

where current = amperes x force per volt.

Power is expressed in watt hours.

1 kilowatt hour = 1,000 watt hours


Hence, 1 kilowatt hour = power of one ampere of 1,000 volts per hour.

Example: Compute for the number of kilowatt hours consumed by 30 bulbs that
had been used for 8 hours at 25 watts each.

Solution: We wish to find the Power or the number of kilowatt hour used. So that,

P = 30 x 25 x 8
P = 6,000 watt hours or 6 kilowatt hours.
Thus, in computing the power or the number of watt hours or the number of kilowatt hours
used, we multiply the number of bulbs by the number of watts by the number of hours used.

The electric meter shown below consists of four dials. The dials are read from left to right in
the same manner as a water meter. The last figure passed by each is the figure that is read.

In the meter showing the Previous Reading. the last numbers passed are listed as follows:

Dials 1000 kwh 100 kwh 10 kwh Units


Last number Passed 4 6 7 4
Thus the number we get from the Previous Reading is 4674 and in the Present Reading is
4752. The number of kwh used is 78 kwh, which is the difference between the Present Reading and
the Previous Reading.

How to Compute Electric Consumption

Electric Rates are categorized and described as Residential and Commercial.

Let us consider this example of a residential consumption of electric bills.

Example:
Mrs. Vilma Calvario’s electric meter reading for the month of October 6, 2006 shows: Present
Reading, 6983; Previous Reading, 6742. What is her bill from September 5, 2006 to October 6,
2006?

Solution:

Step 1 – Present Reading – 6983


Previous Reading – 6742
241 kwh

Account Name & Billing Address


Vilma Calvario
2011 Maria Clara St.
Sampaloc, Manila

MERALCO ELECTRIC BILL


Therefore, the number of kwh consumed is 241 kwh. To compute for the monthly electric
bills of a family there are some additional surcharges and the value added tax which are being
charged by MERALCO.

Let’s practice for mastery 19

The electric meter readings of the following families were as follows:

Family Meter Readings


January 5 to February 4
A 2674 kwh 2798 kwh
B 1082 kwh 1172 kwh
C 4523 kwh 5926 kwh
D 3124 kwh 4032 kwh

Compute the electric consumption of each family.

Temperature Some temperatures that are often


measured are: atmospheric temperature
Temperature refers to the degree of (usually given in weather reports), body
hotness or coldness. Thermometer is the temperature (used to determine illness) and
instrument used to measure temperature. In oven temperature (used in cooking). You use
the metric system, the common unit of the same scales in measuring all these
temperature is given in degrees Celsius (°C) temperatures; however they have different
while in the English system the common ranges of temperatures.
unit of temperature is given in degrees
Fahrenheit (°F).

In order to work with different


temperatures it is necessary to have some
reference points.

A moderate oven temperature is 175°C


and the normal body temperature is 37°C. In
In the English system, degrees Fahrenheit, the normal body
Water freezes at 32°F temperature is 98.6°F. Thus, if you get the
Water boils at 212°F temperature of a person and it is 39°C, you
while in the Metric system will know that the person has a fever.
Water freezes at 0°C
Water boils at 100°C
Conversion of Celsius to Fahrenheit
and Vice-Versa
Formulas: Example 2:
5 Express 132°F in °C.
a. °C = ( )(°F - 32°) Solution:
9
5
9 °C = ( ) x (132° -32°)
b. °F = ( x °C) + 32° 9
5
5
Example 1: = ( ) x 100
Express 40°C in °F. 9
Solution: °C = 55.56°
9
°F = ( x °C) + 32° Therefore, 132°F = 55.56°C
5
9
= ( x 40°) + 32°
5
= 72° + 32°
°F = 104°
Therefore, 40°C = 104°F

Let’s practice for mastery 20

Give each temperature in degrees Celsius.


1. Normal body temperature
2. Boiling point of water
3. Freezing point of water
Give each temperature in degrees Fahrenheit.
4. Freezing point of water
5. Normal body temperature
6. Boiling point of water
Convert each of the following.
7. 25°C = ___ °F 8. 40°C = ___ °F 9. 142°F = ___ °C 10. 100°F = ___ °C

Let’s check your understanding 20


Convert each of the following.
1. 38°C = ___ °F 2. 57°C = ___ °F 3. 65°F = ___ °C
Solve the following.
4. If Joan’s temperature is 99°F, does she have a fever? Why? Convert her temperature to
degrees Celsius.
5. A weather report stated that the high temperature for the day is 90°. What scale was used?

• After answering the Test, check your answers with those on the answer key page.
• If your score is 3 or higher, you may proceed to the next lesson; otherwise, read the lesson
once more for the missed items.
Rounding Numbers 3. Rounding off to the nearest
thousands
Mount Apo has an elevation of 2953
meters. It is usually easier to remember an 6,145 Round down 6000
estimate of the height such as 3000 meters.
An estimate for a number can be found by 4<5
rounding.
21,836 Round up 22,000
For convenience and for us to
remember the figure, rounding off numbers 8>5
is needed.
4. Rounding off to the nearest tenths
Illustrative Examples
1. Rounding off to the nearest tens 3.4113 Round down 3.4
392 Round down 390
1<5
if the unit digit is < 5 drop it
and replace it by 0 5. Rounding off to the nearest
hundredths
5458 Round up 5460
0.7064 Round up 0.71
if the unit digit is > 5 add 1 to the
tens digit and add 0 for the unit digit 6>5

635 Round up 640 6. Rounding off to the nearest


thousandths
if the unit digit is = 5 add 1 to the
tens digit and add 0 for the unit digit 2.9815 Round up 2.982

2. Rounding off to the nearest 5=5


hundreds
Consider the tens digit.
724 Round down 700

2< 5, drop 24 and replace them by two


zeros

8,687 Round up 8,700

8>5, drop 87 and add 1 to 6

553 Round up 600

5=5
Let’s practice for mastery 21

Refer to the chart below. Is the estimate reasonable?


1. Mt. Halcon is about 2600 m high.
2. Mt Pinatubo is about 1700 m high.
3. Mt. Santo Tomas and Mt. Isarog are about 2000 n high.
4. Mt. Arayat is about 1400 m high.
5. Mt. Irid is about 1400 m high.
6. Both Mt. Baco and Mt. Mayon are about 2400 m high.
7. Mt. Pulog is about 3000 high.
8. Mt. Malinao is about 200 m lower than Mt. Pinatubo.

Altitude in Meters
Mt. Pulog 2930
Mt. Halcon 2587
Mt. Baco 2488
Mt. Mayon 2432
Mt. Santo Tomas 2259
Mt. Isarog 1966
Mt. Pinatubo 1740
Mt. Malinao 1533
Mt. Irid 1448
Mt. Arayat 1125
(Source: Math Time Journal Vol. VIII-no.1, 2002.)

Let’s check your understanding 21

Solve the following problems.


1. A quiz book and a spelling book together cost P92.75. To the nearest P10, what is the
cost of two books?
2. The science book and the spelling book together cost P276.50. What is the cost of these
two books to the nearest peso?
3. On a chessboard one can group squares to make larger squares. There are 204 squares of
varying sizes on any chessboard. What is this number to the nearest ten?
4. The highest score made by a person in a popular word game (Scrabble) is 2313 points.
What is this number rounded to the nearest thousand?
5. Katherine scored 150.4758 points in playing computer game. What was her total score to
the nearest thousandths?

• After answering the Test, check your answers with those on the answer key page.
• If your score is 4 or higher, you may proceed to the next lesson; otherwise, read the lesson
once more for the missed items.
Approximation of Measurements Approximation in measurements is
important especially when devices or
There are common objects and parts of instruments in measuring are not available
the body that help in the approximation. The or accessible. You need to have an
illustrations below give you an idea on how to alternative by estimating or making an
approximate certain objects. approximate measure.

Let’s Do It

Make an approximation or estimate the measure.

1. Estimate the distance from your house to the nearest:


a. church
b. school
c. grocery store
2. Estimate the length of your bed.
3. Estimate the height of a coconut tree nearest to your house.
4. Estimate the age of your best friend.
5. Estimate the floor area of your house.
6. Estimate your height.

Scientific Notation
Do you know that scientists who There is much easier and shorter way to
studied lunar samples from the Apollo space write big numbers like 4,600,000,000 and
missions estimate that the moon is small numbers like 0.00000001. This is by
4,600,000,000 years old. And do you also expressing the number in scientific notation.
know that the unit of length known as the
Angstrom is 0.00000001 of a centimeter?

To express a number in scientific Example 3:


notation, you write it as a product of a
5
number between 1 to 10, and a power of 10. Express 7 x 10 in standard form.

Recall: Solution:
Remember that:
5
-1
7 x 10 = 700000
1
10 = 10 0.1 = 10
2 -2
100 = 10 0.01 = 10 Move the decimal point 5 places to the
1000 = 10
3
0.001 = 10
-3 right since the exponent is positive.
4 -4
10000 = 10 0.0001 = 10
5 -5 Example 4:
100000 = 10 0.00001 = 10
-5
Express 6 x 10 in standard form.
Writing numbers in scientific notation
is useful and a convenient way for scientists Solution:
and mathematicians to work with very large
as well as very small numbers. -5
6 x 10 = 0.00006
Example 1:
Move the decimal point 5 places to the
Express 150,000,000 in scientific
left since the exponent is negative.
notation.

Solution:
Calculators and Scientific Notation
8
150,000,000 = 1.5 x 10
Most calculators shift to scientific
notation when the results are greater than their
You get an exponent of 8 by moving
allowable display of digits. For example, to
the decimal point eight times to the left from 10
150,000,000 to get 1.5. find 25 , the display will show 9.536743163
13
13. This means 9.536743163 x 10 .
Example 2:
If the calculator has the EXP or
Express 0.00000001 in scientific EE key, enter numbers in scientific
notation. notation this way.
4
Solution: To enter 2.75 x 10 press

-8 2.75 EXP 4 27500


0.00000001 = 1 x 10
EXP 4
You get an exponent of -8 by moving 2.75 4 2.75 x 10
the decimal point eight times to the right
from 0.00000001 to get 1.
Let’s practice for mastery 22

Write the numbers in scientific notation.

1. The first manned space flight was made by Vostok I from the USSR. Its average speed
was 27,353 kph.
2. The star Merak is approximately 740,000,000,000,000 m from the Earth.
3. The measure of one calorie is equal to 0.000000278 kilowatt-hours.
4. One light year is approximately equivalent to 9,461,000,000,000 km.
5. The thickness of a particular surface is 0.0000415 cm.
7
6. Write 4 x 10 in standard form.
-2
7. The standard form of 7 x 10 is _____.
8
8. What is the standard form of 4.3 x 10 .

Let’s check your understanding 22

Write the numbers in scientific notation.

1. Earth travels approximately 986 600 000 km in its orbit around the sun every year.
2. The most remote heavenly body visible to the naked eye is the Great Galaxy in
Andromeda, known as Messier 31. It is 37 000 000 000 000 000 000 m from Earth.
3. The height of a certain microorganism is 0.00000007 mm.
4. Proxima Centauri is the closest star (other than the sun) to Earth. It is about 38 290 000
000 000 km away.
5. Earth is approximately 149 600 000 km from the sun.
7
6. Write 6.83 x 10 in standard form.
-5
7. The standard form of 8 x 10 is _____.
8
8. What is the standard form of 9.2 x 10 .

• After answering the Test, check your answers with those on the next page.
• If your score is 6 or higher, you may proceed to the next lesson; otherwise, read the lesson
once more for the missed items.

Unit Test II

Answer the following. Solve if necessary.

1. What is the base unit of measure of length?


What unit of measurement will you use to find the following?
(mm, cm, m, km)
2. average length of toothpick
3. distance between Manila and Quezon City
4. the height of a tree
5. What is the basic unit of liquid measure in the metric system?
6. What is the basic unit of mass measure in the metric system?

Give the missing units.


7. The capacity of a gasoline tank is about 75 _____.
8. The weight of a loaf bread is 475 _____.
9. A newborn baby would weigh about 5.8 _____.
10. An average cup of coffee is about 250 _____.

11. What is the area of rectangular pigpen whose length is 6.5 m and its width is 2.8 m?

12. What is the perimeter of a squared paper whose side is 18 cm?

13. It refers to the hotness or coldness.

14. The temperature on the surface of the sun is approximately (7.5 x 102)°C. What is the
reading in degrees Fahrenheit?

15. The cost of a 10% discounted T-shirt is P299.92. What is the cost to the nearest peso?

16. Earth is approximately 149,600,000 km from the sun. Express in scientific notation.

Round off each of the following peso amounts to the values indicated.
17. P304.85 (nearest peso)
18. P52.945 (nearest centavo)
19. P28.72 (nearest centavot)

20. A melon has a weight of 0.72 kg. How many grams is this?

21. Sarah drinks 5 glasses of water everyday. Each glass holds 375 mL. How many liters
does Sarah drink each day?

22. What is the normal body temperature in degrees Centigrade?

23. If Joan’s temperature is 100°F. Does she have a fever? Why? Convert her temperature to
degrees Celsius.

24. A 100-m wire is cut into 75-cm pieces. How many pieces can be cut?

25. Jomar bought 6 can of pineapple juice, each with 350 mL of juice. How many liters of
juice are in the six cans?
Answer Key

Let’s practice for mastery 1

42
I – 1) = 42 rational
1
1 43
2) - 6 = - rational
7 7
6 26
3) – 2.6 = - 2 =- rational
10 10
23
4) 23% = rational
100
3
5) 0.3 = rational
10

5
II – 6) one dozen = 12
12
50 1
7) = one peso = 100 centavos
100 2
3
8) one week = 7 days
7
45 3
9) = one hour = 45 minutes
60 4
3 1
10) = one year = 12 months
12 4

Let’s check for understanding 1

8 12
1. rational 6.
1 12
35 90 9
2. - rational 7. =
4 200 20
67 12
3. rational 8.
10 7
67 35 7
4. rational 9. =
100 120 24
18 12 1
5. rational 10. =
100 36 3
Let’s practice for mastery 2

1) 48 = 16(3) = 16 • 3 = 4 3 irrational
2) 20 = 4(5) = 4• 5 =2 5 irrational
3) 64 = 8 rational
4) 72 √72 = 36(2) = 36 • 2 = 6 2 irrational
5) 121 = 11 rational

Let’s check for understanding 2

1. irrational
2. rational
3. irrational
4. irrational
5. rational

Let’s practice for mastery 3


1 3
1) 0.25 3) 0.2727…
4 11
repeating
4 1.00 terminating 11 3.000
8 22
20 80
20 77
0 30
22
2
2) 0.66... repeating 80
3
3 2.00 77
18_ 30
20
18
20
7 5
4) 5)
10 12
terminating 0.7 repeatimg 0.4166…
10 7.0 12 5.0000
70 48
0 20
12
80
72
80
72
8
TEST 3

1. .089 terminating decimal


2. 0.3636… terminating decimal
3. 4.5 repeating decimal

Let’s practice for mastery 4

4 2 63
1) 0.4 Æ or 4) 0.63 Æ
10 5 100
45 9
2) 0.45 Æ or
100 20
225 9 984 123
3) 0.225 Æ or 5) 0. 984 Æ =
1000 40 1000 125

TEST 4

15 3 6 3 473 2 1 8 2
1. or 2. or 3. 4. or 5. or
100 20 10 5 1000 10 5 100 25

Try these:

5 52 7825 138
1. 2. 3. 4.
9 99 9999 999

Let’s practice for mastery 5

1) + 700 or 700 6) withdrawal of P32


2) – 150 7) 280 below sea level
3) +15 or 15 8) 25 miles north
4) -50 9) 20 sec after departure
5) 1,000 10) an asset of P5,000

TEST 5

1. 5 6. 30 min earlier
2. -2 7. a decrease of 20 cm in height
3. 1125 8. 12 is located to the right of zero
4. -40 9. 5 km going downstream
5. 33750 10. spending P250
Let’s practice for mastery 6

Which fraction in each pair is greater?

3 2
1) > If fractions have the same denominators, the greater
5 5
the numerator, the greater the value.
5 5
2) > If fractions have the same numerators, the greater the
7 9
denominator, the smaller the value.
3 2 2 3
3) > = 0.666… = 0.75
4 3 3 4
1 2 1 2
4) > = 0.5 = 0.4
2 5 2 5
2 4 2 4
5) > = 0.666… = 0.571…
3 7 3 7

Write <, = or > on the blank to make a true statement.

3 2 7 6 8 9
1) > 3) > 5) >
7 7 9 9 3 4

6 6 8 12
2) > 4) =
11 12 8 12

Problem Solving:

9 10 9
1) Araullo HS > = 0.8181
11 3 11
10
= 0.7692
3

TEST 6

5 7
1. = 0.41666… 2. = 0.777…
12 9
3 8
= 0.6 = 0.888…
5 9
5 3 7 8
< <
12 5 9 9

8 2 5
3. = = 0.666… 4. = 0.833… 5. Magsaysay HS
12 3 6
3 1 7 3 5
= = 0.2 = 0.875 >
15 5 8 4 8
8 3 5 7
> <
12 15 6 8

Let’s practice for mastery 7

1. -12 > -18 6) <


2. 10 > -4 7) -19, -15, -13, -12, -1
3. 5>0 8) -17, -8, -5, 0, 18
4. > 9) 10, 6, 3, -6, -7, -8, -9
5. < 10) a man who is on a hill 207 meters above the sea
level

TEST 7

1. 8 7. -14, -32, -10, 15, 25


2. -9 8. -9, -5, -4, -2, 4, 6, 9
3. 5 9. 17, 16, 15, -15, -16, -17
4. < 10. A student whose score was 20 above
5. > the passing score. Difference = 55

Let’s practice for mastery 8

5 1 6 1 3 4 12 3
1) + = or 7) x = or
12 12 12 2 8 7 56 14
5 1 4 2 3
2) – = or 8) x8=6
6 6 6 3 4
1 2 1 6 7 2 5 2 6 12 4
3) + = + = 9) ÷ = x = or
9 3 9 9 9 3 6 3 5 15 5
3 1 21 5 26 1 7 1 16 8 1
4) + = + = 10) ÷ = x = or 1
5 7 35 35 35 2 16 2 7 7 7
7 3 14 3 11 3 4 7 2
5) – = – = 11) 3 + 1 = 4 or 5
8 16 16 16 16 5 5 5 5
1 1 1 7 3 1
6) x = 12) 5 13.) 7 or 7
2 4 8 8 9 3
3
- 1 14) 2
8
4 1 3
4 or 4 15)
8 2 10
B. 1. answer was done for you
2. 64
3. 5
4. 16
5. 14

TEST 8

6 1 7 1
1. + = 11. 3
18 18 18 5
12 5 7 4
2. – = + 2
15 15 15 5
2 3 (8 + 9) 17 5 5
3. + = = or 1 5 or 6
3 4 12 12 12 6
5 2 (5 + 8) 13 1 3
4. + = = or 1 12. 5
12 3 12 12 12 4
4 1 (8 − 3) 5 1
5. – = = - 2
9 6 18 18 4
5 4 20 5 2 1
6. x = or 3 or 3
8 8 64 16 4 2
6 12 3 3
7. x = 13. 7
8 24 8 5
4 98 2
8. x7=4 14. or 32
7 3 3
3 3 3 7 30
9. ÷ = x =1 15. = 10
7 7 7 3 3
5 3 5 5 25
10. ÷ = x =
6 5 6 3 18

Let’s practice for mastery 9

3 1 30 8 38 19
1) + = + = or of his cornfields was ruined by the
8 10 80 80 80 40
typhoon
3 1 3 2 1
2) 9 – 6 = 3 – = 3 hours
4 2 4 4 4
1 2 1
3) x = part of the cake she ate
2 3 3
1
4) 3 ÷ = 12 boxes
4
7 5 7 10 17 1
5) + = + = or 1 inches total length
16 8 16 16 16 16

TEST 9

5 1 25 − 6 19
1. – = = m
6 5 30 30
3 4 36
2. 9 ÷ = 9 x = or 12 curtains
4 3 3

Let’s practice for mastery 10

1) 8 + (-12) = -4 6) (-6) (-4) = 24

2) -20 + (-13) = -33 7) (32) ÷ (-8) = -4

3) -28 – (-10) = -18 8) (-54) ÷ 9 = -6

4) 49 – (-18) = 67 9) 8 + (-5) + (4) = 7 It’s on the 7th floor

5) (-3) + 11 = 8 10) 65 + (-4) + (-2) + (3) + (1) = 63 kg.

11) -25 + (-23) + 15 = -33 m


The diver is 33 m under water.
12) 10
13) 6 is greater than -19
TEST 10
1. -3 8. (4,260 + 1,059) – 2,115+780 = 3,984 m
2. -25 9. P2000 – P1050 – P450 + P1380 = P1,880
3. 14 – (-6) = 20 10. a) It rises by 9°C
4. 24 – (-6) = 30 b) It drops by 9°C
5. 32
6. -15
7. -45

Unit Test I

Answers
7
1) Rational 10) = 0.875
8
8 2 9
2) or = 0.9
12 3 10
7 9
<
8 10
3) 0.777… 11) -15 > -18
12 3
4) 0.012 = = 12) -30 < 25
1000 250
3 2 5
5) –5° 13) + =
7 7 7
5 3 10 9 1
6) +6 14) – = – =
6 4 12 12 12

5 3 1
7) –9 15) x =
12 5 4
8 2 4
8) +P500 16) ÷ = =1
12 3 4
5
9) = 0.833… 17) (-5) + (8) = 3
6
3
= 0.75 18) Yes
4
5 3
> 19) 15
6 4

20) -9
21) 4
22) -90
23) 37.5 ÷ 2.5 = 15 weeks
24) 5 x (–P20,000) = – P100,000; P150,000 – P100,000 = P50,000
25) 20° above zero

Let’s practice for mastery 11

1) 31 mm 6) 54 g
2) 2 km 7) 1.8 kg
3) 68 cm 8) 165 mL
4) 8 kg 9) 200 mL
5) 453 g 10) 400 L

TEST 11
1. 1 mm 6. 5 g
2. 26 cm 7. 515 mg
3. 10,000 m 8. 250 mL
4. 305 g 9. 500 mL
5. 380 mg 10. 2 L
Let’s practice for mastery 12
1) 1,000 km > 1,000 mm 5) 8,643 dm = 864.3 m

2) 100 cm = 1m 6) 1 m = 1,000 mm
1,000 ÷ 23 = 43.4
3) 924 cm = 9.24 m Therefore, there are around 43
coins

4) 3 m = 3,000 mm

TEST 12

1. >
2. =
3. 0.00421
4. 0.758
5. 40
6. 100 m = 10,000 cm
10,000 = 133 pieces can be cut
75

Let’s practice for mastery 13

1) Liters 9) 5 x 375 ml = 1875


2) Milliliters There are 1,000 ml in 1 liter
3) Milligrams 1,875 ÷ 1,000 = 1.875 liters
4) Kilograms Therefore:
5) 3,840 L = 3.840 kL Sarah drinks 1.875 liters of
water
6) 0.5 L = 50 cL everyday.
7) 85 cg = 850 mg 10) 2,000 g = 2 kg
8) 6.5 kg = 6,500 g 2,000 ÷ 10 = 200 keys

TEST 13
1. mL 4. kg 7. 0.028
2. L 5. 29,000 8. 0.015
3. g 6. 2 9. There are 1,000 L in 1 kL
4500 ÷ 1000 = 4.5 kL.
There are 4.5 kL.
10. 15 kg = 15,000 g

Let’s practice for mastery 14

Boy Do I Have Problems


Let’s practice for mastery 15

SINGAPORE

Let’s practice for mastery 16


Answers may vary depending on the unit of measurement.

TEST 16

1. A = l x w 4. V = e3
= 2 x 1.5 = 153
= 3 cm = 3375 cm3

2. A = πr² 5. V = l x w x h
= (3.14) (75 cm)² =9x6x4
= 17,662.5 cm2 = 216 cm3

3. A = s2
= 252
= 625 cm2

LET’S PRACTICE FOR MASTERY 17

1) l = 8 ; w = 5 4) V = e3
A=lxw = (2.5)3
=8x5 = 15.625 cm3
= 40 sq. cm.

2) C = 2πr ; r = d/2 ; r = 70/2 = 35cm 5) V = l x w x h


= 2 (3.14) (35) = 8 x 6 x 35
= 219.80 cm = 168 m3
3) P = 4s
100 = 4s
100
S=
4
S = 25 cm

LET’S PRACTICE FOR MASTERY 18

A – 39
B – 113
C – 16
D – 24
LET’S PRACTICE FOR MASTERY 19

1) 2,798 2) 1,172 3) 5,926 4) 4,032


- 2,674 - 1,082 - 4,523 - 3,124
124 kwh 90 kwh 403 kwh 908 kwh
Family A Family B Family C Family D
ACTIVITY 20

9
1) 37°C 8) ( x 40 ) + 32
5
2) 100°C 72 + 32 = 104°F
3) 0°C
5
4) 32°F 9) ( ) x (142 – 32)
9
5
5) 98.6°F x 110 = 61.1°C
9
6) 212°F
9 5
7) ( x 25) + 32 10) ( ) x (100 -32)
5 9
5
45° + 32° = 77°F x 68 = 37.7°C
9

TEST 20

1. 38°C = _____ °F 4. Yes, she has a slight fever.


9
= ( x 38°) + 32° 99°F = _____ °C
5
5
= 68.4°F + 32 = x (99° - 32°)
9
5
= 100.4°F = x 67
9
2. 57°C = _____ °F = 37.2°C
9
= ( x 57°) + 32° The normal body temperature is 37°C or
5
98.6°F. = 102.6°F + 32°
= 134.6°F
3. 65°F = _____ °C 5. Fahrenheit
5
= x (65° - 32°)
9
5
= x 33° = 18.3°C
9

LET’S PRACTICE FOR MASTERY 21

1) Yes 2,587 Æ 2,600


2) Yes 1,740 Æ 1,700

3) Yes 2,259 Æ 2,000


1,966 Æ 2,000
4) Yes 1,125 Æ 1,100
5) Yes 1,448 Æ 1,400
6) No 2,488 Æ 2,500 Mt. Baco
Yes 2,432 Æ 2,400 Mt. Mayon
7) Yes 2,930 Æ 3,000
8) Yes 1,533 Æ 1,500 Mt. Malinao
1,740 Æ 1,700 Mt. Pinatubo

TEST 21

1. P92.80
2. P277.00
3. 200
4. 2000
5. 150.005

ACTIVITY 22

1) 27,353 = 2.7353 x 104

2) 740,000,000,000,000 = 7.4 x 1014

3) 0.000000278 = 2.78 x 10-7

4) 9,461,000,000,000 = 9.461 x 1012

5) 0.0000145 = 4.15 x 10-5

6) 40,000,000

7) 0.07
8) 430,000,000

TEST 22

1. 9.866 x 108
2. 3.7 x 1019
3. 7 x 10-8
4. 3.829 x 1013
5. 1.496 x 108
6. 68,300,000
7. 0.00008
8. 920,000,000

UNIT TEST II

ANSWERS

1. meter
2. cm
3. km
4. m
5. liter
6. gram
7. liter
8. g
9. kg
10. mL
11. 18.2 sq m
12. 72 cm
13. temperature
14. 1382°F
15. P300
16. 1.496 x 108
17. P305
18. P52.95
19. P28.70
20. 720 g
21. 0.375 L
22. 37°C
23. 37.78°
24. 133 pieces
25. 2.1 L
Common Errors / Misconceptions in Units I and II

1. Addition and Subtraction of Fractions


Examples
2 3 10 3
1. + 2. −
3 4 12 6

The common answers of the students for this kind of problem is


2 3 5 10 3 7
+ = − =
3 4 7 12 6 6
Students usually add/ or subtract the numerator then write the common
denominator. It is the most common error since to get the sum / difference of fractions
with different denominators we have to look for the LCD

Examples:
2 3 8 + 9 17 5 10 3 10 − 6 4 1
+ = = or 1 − = = =
3 4 12 12 12 12 6 12 12 3

2. Order of Operations (MDAS)


Example:
Simplify: 6 + 12 ÷ 3 ⊗ 5

The common answer of the students is 30, but actually is the 26. we
have to do all multiplication and division first and add then subtract.

6 + 12 ÷ 3 ⊗ 5 = 6 +4 x 5
= 6 + 20
= 26

3. Multiplication / Division of Integers

Examples: 1. (-3)(-4) = -12 2. (-12)/ (-6) = -2


This is a common mistake of the students, since to multiply/ divide integers with
like sign the product / quotient is always positive.
Algebra

1
ALGEBRAIC EXPRESSIONS

N
An aid for navigation and pilotage at sea is the lighthouse , a
tower building or framework very familiar among the navigators,
especially to those who sail by boat. This lighthouse serves as the
compass for the navigators as it provides travelers and mariners
I
information on the wind direction by displaying a light for their
guidance. Lighthouses also provide coordinate location for small
aircraft traveling at night.

Because of modern navigational aids, the number of operational


lighthouses has declined to less than 1,500 worldwide. Lighthouses
are used to mark dangerous coastlines, hazardous shoals away from
the coast, and safe entries to harbors.
T
Just like the lighthouse, in this unit, you will be introduced to
the world of symbols that are used to convey ideas in Algebra. The
concept of variables and algebraic expressions are basic in the study
of Algebra. The use of variables and symbols become more
meaningful when word phrases or verbal phrases are translated to
mathematical phrases . Identifying the key phrases in verbal phrases
facilitates translating them to mathematical phrases . This process is
a prerequisite in solving word problems.

Aside from translating verbal phrases to mathematical phrases,


the constant use of mathematical symbols will also help you discover
III
some patterns in the applications of algebraic concepts. Performing
the operations on algebraic expressions , applying the laws of
exponents , special products and factoring become more interesting
when applied to real life experiences.

2
ALGEBRAIC EXPRESSIONS

Translation of Verbal Phrases to With y as the variable representing the


Mathematical Phrases and vice- number of banana cues sold at P10.00, 10y is the
expression that will complete the table,
versa
If y is the variable , 10y is called an algebraic
Julius helps his parents increase their expression.
family income by selling banana cue. To An algebraic expression consists of one or
facilitate the computation of his sales, he more numbers and variables with one or more
prepared this table. arithmetic operations. Following are examples of
Number of Sales algebraic expressions
Banana Cues Sold (in pesos)
1 10 b
y+2 , +1 , 8 xy + 2 x and a + 3n
2 20 c
3 30
4 40 In multiplication, the quantities being multiplied
6 _____ are called factors and the result is called product.
7 _____
. _____ 3 • n = 3n (3)(8) = 24
. Factors Product Factors Product
. A center dot or parenthesis is often used to
y _____ indicate multiplication. When variables are used to
represent the factors the multiplication sign is
How much will you pay for 5 pieces omitted as shown in the following examples:
of banana cue? 6? 7? What expression
will complete the table? Can you describe 5 x , 2ab , 4 xyz and 6an .
the pattern?

3
Word Phrases Mathematical
Fraction bars indicate division as in Phrases
3. y minus 3
18abc 36 xyz 12ab 3 subtracted from y y-3
, and
9ac 4 yz 4ab the difference of y and 3
To solve word problems in y decreased by 3
mathematics, you can create algebraic 4. n divided by 8 n
expressions by translating verbal phrases the quotient of n and 8 8
to mathematical phrases using symbols.
The chart below shows some of the 5. Eight divided by n 8
words used to indicate the mathematical n
operations: 6. The product of x and
Operation Mathematical y increased by 2 xy + 2
phrases
1. Addition -the sum or total of 7. The square of x
-plus less 8 x2 − 8
-added to
- more than 8. Three times the quantity
-increased by x squared plus 4 (
3 x2 + 4 )
2. Subtraction -decreased by
- subtracted from 9. Three times the 3( x − 5)
-less than quantity x minus 5
-minus 10. Three-fourths of 3
-the difference of the quantity x plus 2 (x + 2)
4
3. Multiplication -the product of
- multiplied by
Do the following exercises:
-times
4. Division -the quotient of Translate the following verbal phrases into
-divided by Mathematical phrases/Algebraic expression
- the ratio of
Verbal Phrases Algebraic
The following are examples of word Expression
phrases translated to mathematical
1. 3 times y minus 1
phrases.
2. Three times x squared
plus 4
Word Phrases Mathematical
3. The product of the
Phrases
square of y and five
1. x plus 2
4. The quotient of 2 and
2 is added to x x+ 2
the product of x and y
x increased by 2
5. One-half times the
quantity twice x minus y
2. ten times y
6. The sum of the squares
the product of 10y
of a and b
10 and y

4
Now, let us do the reverse operation, that Order of Operations
is, changing mathematical phrases to verbal
phrases or word phrases: 1. Simplify the expressions inside the
grouping symbols, such as braces,
Translate the following mathematical phrases parenthesis or brackets and as indicated
to verbal phrases by fraction bars.
2. Evaluate all powers and extract roots
Mathematical Word or Verbal 3. Do all the multiplications and divisions from
Phrases Phrases left to right
(
1. 5 z 3 − 9 ) Five times the quantity cubed
of z minus 9
4. Do all addition and subtraction from left to
right.
2. 2 x 2 + 2(x − y ) The sum of twice the square of
x and twice the difference of Examples:
x and y
k cubed plus twelve
3. k 3 + 12 1. Find the value of 2 x − 4 for x= 3.
2( x + y ) The quotient of twice the sum
Solutions:
4. of x and y and six times the
6( x − y ) difference of x and y 2 x − 4 = 2(3) − 4 Substitute 3 for x

a 3 + b3
The sum of a cubed and b = 6−4 Multiply 2 and 3
cubed divided by the
5. = 2 Subtract 4 from 6
a 2 − b2 difference of a squared and b
squared : . 2 x − 4 = 2 (for x= 3)
6. 5(x 2 y − z 3 ) Five times the quantity x
squared y minus z cubed.
Three – fourths of x to the 2. Evaluate 2(y-4) +3 for y= -2
3
7. x 5 + 2 xy fifth power plus twice the Solutions:
4 product of x and y
2 (y-4) +3 = 2(-2-4) +3 Substitute 2 for y
8. V = lw V is equal to the product of l
and w = 2(-6)+3 Add -2 and -4
9. I = prt I is equal to product of p, r = -12 +3 Multiply -6 and 2
and t
= -9 Add the integers -12
10. P = 2l + 2 w P is equal to the sum of twice
l and twice w and 3
: . 2(y-4)+3=-9 (for y=-2)
Evaluating Algebraic Expressions x
3. Evaluate xy − for x= 6 ; y = -12
y
Is 3y +5 equal to 3 (y + 5)? Let us find Solutions:
out.
x − 12
If you replace y by2, then: xy − = 7(-12) - Substitute x for 6 and y
y 6
3 y + 5 = 3(2 ) + 5 = 6 + 5 = 11 for
-12
3( y + 5) = 3(2 + 5) = 3(7 ) = 21 = -84 – (-12) Multiply 7 and -12
Divide -12 by 6
Hence, 3 y + 5 ≠ 3( y + 5) = -84 +2 Multiplication of
integers
= -82 Addition of integers
In evaluating algebraic expressions, x
you simply substitute or replace variables by : . xy − = -82 ( for x = 6 and y = -12)
y
numbers and carry out the operations
following the order of operations:

5
4. 3{ 4x +[ -3x- (2x+6) ]} for x = 2 Another solution:
Solutions: 3{4x +(-3x – [2x +6])}=
3{4x +(-3x – [2x +6])}=
= 3{ 4(2) + ( -3(2) – [2(2) +6])}Substitute x = 2 =3{4x + ( -3x – 2x -6)} Remove the grouping signs[ ] by
distributive property and since

= 3{8 + (-6– [4 + 6])} Multiply 2 by 2 it is preceded by negative sign,

= 3{8 + (-6 - [10] )} Add 4 and 6 change the signs of each term

= 3{8 +(-6 -10)} Remove the[] grouping sign = 3{4x + ( -5x -6) Combine similar terms

= 3{8 + (-16)} Add -3 and -10 = 3{ 4x + -5x -6} Remove the grouping sign ( ) by
distributive property and since
= 3{ 8 + -16} Remove the ( ) grouping sign it is preceded by plus sign
retain the signs of each term
= 3 { -8} Add algebraically 8 and -13 = 3{ -x -6) Combine similar terms

= -24 Multiply 3 and -5 = -3x -18 Remove the grouping signs[ ] by


distributive property

∴ 3 {4x +(-3x –[2x +6] )} = -24 for x = 2 = -3(2) – 18 Replace x by 2

= -6 -18 Multiply -3 and 2

= -24 Subtract algebraically -6 and 18

∴ 3{4 x + (−3x − [2 x + 6])} = − 24 for x = 2

6
ACTIVITY I

A. Translate the following phrases into algebraic expression:


1. The sum of 6 and x .
2. A number decreased by 8.
3. One-third of a number increased by 4.
4. The product of x and 4 decreased by 2
5. Five less than one-half of a number.
6. Three plus the product of a number and thirteen
7. Two –fifths times the quantity C plus four .
8. The product of negative two times a certain number
a and the quantity a plus nine.
9. The difference between 5 times a number and eight
10. Three plus the product of a number and 5

B. Translate the following algebraic expressions to their equivalent word expressions

1. 9x-4 = ____________________________________________________________

−3
2. x − 4 = _________________________________________________________
4

3.
5
(
3 2
)
x + 2 = ________________________________________________________

4. 4a 3 − 7 = __________________________________________________________

 1
5. z 2  z −  = ________________________________________________________
 4
C. Complete each statement with a variable expression.
1. Lita is 2 cm taller than Cecile. If Cecile’s height is P cm, then Lita’s height is
_____ cm.
2. Josie has twice as much money as Carlos. If Carlos has W pesos, then Josie
has _____ pesos.
3. The sum of two numbers is 12. If one number is Q, then the other number is
_______

4. Marie is 2 years older than Vicky. If Vicky is G years old now, then Marie is _______
years old.
5. The product of two numbers is 15. If one number is y, then the other number is
______.

7
D. Find the value of the following expressions:

1. 3 x + 2 y − 1 for x = 2 ; y = − 2
2. {4 x − 2( x + 4 )} for x = 1
3. [10 z + 4 {2 z + 6 y} − 1] for z = 1; y = 2
x + y
4. for x = 2; y = 8
10
5. (12 x − y ) − (5 x + 3 y ) for x = − 1; y = −1

E. Solve the following problems:

1. Myrna jogs on weekends. The sum of the distance she covered for one month is
(24x – 2)km. What is the actual distance covered if x = 6.

2. A gardener uses water sprinkler to water the plants as well as the garden lawns. The
area covered by the sprinkler defined by A = π r 2 with π = 3.14, r = 1.5 meters.
What is the actual area watered by the sprinkler?

3. Mrs. Campos ,a Health teacher, measured the weight of her 4 elementary students. She
found out that Marie is heavier than Cecile by 1 kg. while Cora is twice as heavy as
Marie . Tessie and Cecile have the same weight denoted by x. What must be the actual
weight of Marie and Cora if x = 20 kg. ? What must be the sum of the weights of the
four students?

TEST 1

A. Fill in the blanks: (2 points each)

1. An algebraic expression for “10 less than d” is ____________.

2. The value of the expression 4m 2 − n(s − m ) if n = 3 , s=5 and m = -2 is ______.


n
3. The expression “ + 6 ” can be written as ________.
4
4. Lilia has two more brothers than sisters. If Lilia has Q brothers, then she has ______
sisters.
5. The dimensions of a rectangular window are 3x meters long and 2x meters wide. If George
needs to enclose the window with moldings, how many meters of molding would he need to
enclose it? The expression that represents the answer is ______.

B. Write the mathematical phrase for each word phrase.

1. Two more than twice a number s.

8
2. The number of centimeters in g meters.( 100 cm = 1 meter)
3. The quotient when a number r is divided by five.
4. A number D increased by four.
5. The difference when 10 is subtracted from the sum of q and r.

C. Evaluate the following expressions if x = 10 ; y = 20 and z=30

1. 2x - 3y + z 4. z + 12 + 18y
2. 123 + y - 12 x 5. y – 83 + x + y
3. 3xy + 2z

Scoring:
• After answering the test, check your answers with those on the answer key page
• If your score is 4 or 5, you may proceed to the next lesson; otherwise, read the
lesson once more and do the test again.
• If you did not make it for the second time, consult your teacher.

9
Laws on Integer Exponents Consider the following examples

n
(2 ) = (2 )(2 ) = 2
3 2 3 3 3+ 3
= 26
An expression of the form x is a power.
The base is x and the exponent is n .
(x ) = x x x = x
6 3 6⋅ 6 6 6+6+ 6
= x18

Based on the above examples, we can say that


to find the power of a power, we simply multiply
Consider the table below which shows the the exponents.
powers of 2.
Power of a Power
For any number a and all integers m and n
21 22 23 24 25 26 27 28 29 210
(am)n= amn
2 4 8 16 32 64 128 256 215 1024
Another case for integral exponents which
Observe the pattern in the products below we will encounter in our study is illustrated by
the following examples:
4 ⋅ 32 = 128 8 ⋅ 128 = 1024 1. ( xyz ) = (xyz )(xyz )( xyz )
3

↓ ↓ ↓ ↓ ↓ ↓ = ( x ⋅ x ⋅ .x )( y ⋅ y ⋅ y )( z ⋅ z ⋅ z ) = x 3 y 3 z 3
2 ⋅2 = 2
2 5 7
2 ⋅ 2 = 210
3 7

2. (2ab c ) = (2ab c )
3 2 3 2

How do we get the exponent of the product, (


= (2 ⋅ 2)(a ⋅ a ) b 3 ⋅ b 3 (c ⋅ c ) )
in the preceding examples?
= 4a b c 2 6 2

Do you see that the exponents of the The above examples suggest that the power of a
factors with the same base are added to get product is the product of the powers.
the exponent of the product? Observe that :
Below are some extensions to the integral
2+5 = 7 and 3 + 7 = 10 powers
Power of a Product
Product of Powers
For any number a and all integers m For all numbers a and b and any integers m ,
and n , a m ⋅ a n = a m + n (ab )m = a m.b m
Power of a Monomial
Examples:
For all numbers a and b and any integers m,
1) 2 3 ⋅ 2 2 = 2 3+ 2 = 2 5 n and p
2) 2 3 ⋅ 2 7 = 2 3+7 = 210 (a
b n ) = a mp b np m p

3) x 4 ⋅ x = x 4+1 = x 5 Now, consider the following quotients. Each


4) b 3 ⋅ b 7 = b 3+ 7 = b10 number can be expressed as power of a number:
( )
5) 4 x 2 yz 2 (2 x 3 y 4 z ) 3 xy z 3 ( 2
) Powers of 2
Applying the commutative and associative 32 256
properties repeatedly, we have 1. =8 2. = 27
4 32
( )(
= (4 ⋅ 2 ⋅ 3) x 2 x 3 x y ⋅ y 4 ⋅ y 3 z 2 z ⋅ z 3 )( ) ↓ ↓ ↓ ↓
= 24 x 2+ 3+1 ⋅ y 1+ 4+3 ⋅ z 2+1+3
25 28
= 24 x 6 y 8 z 6 2
= 2 5− 2 5
= 2 8−5
2 2
= 23 = 23

10
Powers of 3 always equal to 1 and any expressions whose
1. 2. exponent is equal to zero then it is equal to 1.
81 243 Hence , a 0 = 1 for any a ≠ 0.
=9 = 27
9 9 Next, consider the example below. Simplify
↓ ↓ ↓ ↓ 10 7
4 5 = 10 7 − 9
3 3 10 9
2
= 32 2. 2
= 33 1
3 3 10 − 2 =
10 2
3 4− 2 = 3 2 35− 2 = 33
This example illustrates the law for negative
32 = 32 33 = 33
exponents.
What will you do with the exponents of the
Negative Exponent
dividend and the divisor to get the exponent of
For any non-zero number a and any integer n,
the quotient?
1
a −n =
To get the exponent of the quotient you n
simply subtract the exponent of the divisor
An exponent is said to be simplified if it
from the exponent of the dividend,
does not contain a zero exponent or a negative
exponent
25 25
1. a) = 23 b) = 23
2 2
2 2
In simplifying an algebraic expression, you
3 4
35 write an equivalent expression that has positive
2. a) = 3 4− 2 = 3 2 b) 2 = 35− 2 = 33 exponent and no powers of powers, each base
32 3
should appear only once and all fractions should
Quotient of Powers be expressed in simplest form
For all integers m and n, and any non-zero Example: Simplify
am
number a, n = a m − n 16b 4 c  16  b 4  c 
a 1) =   3 
− 4bc 3  − 4  b  c 
Zero Exponent = −4b 4−1c1−3
For any non-zero number a, a 0 = 1 = − 4b 3 c −2
− 4b 3
Consider the following fractions: =
2 3 7 d h2 y a c2
= = = = = =1
2 3 7 d h2 y a 22 a 2 b 5 c 7  22   a   b   c 
2 5 7

2) =        2 
What do you observe about the numerators 11 abc 2
 11   a   b   c 
and the denominators of the fractions?
Applying the quotient of powers what will = 2a 2−1b 5−1c 7 −2
happen to the exponents of each fractions? = 2ab 4 c 5
What conclusion can you give about the − 125 x 6 y 4 z  − 125  x 6  y 4 
fractions with the same numerator and 3) =  3  6  z
− 25 x 3 y 6  − 25  x  y 
denominator? What about if the expression
has a zero exponent? = 5 x 6 −3 y 4 −6 z
= 5 x 3 y −2 z
When the numerator and denominator of a
fraction are equal the value of the fraction is 5x3 z
=
y2

11
ACTIVITY 2
I Solve the problems and write the letter above each matching answer to decode the message.
A. 62 = ______ J. 102= ______ S. 71= ______

B. 122 = ______

C. 53 = ______
K. 51 = ______

L. 22 = ______
T. 43 =______

U. 17 =______
6 #
D. 10 3 =______

E. 101 =______
M. 111= ______

N. 32 = ______
V. 92 = ______

W. 33= ______
<=
F. 25 = ______

G. 42 = ______
O. 03 = ______

P. 63 = ______
X. 52= ______

Y. 112= ______
¼±
H. 72 = ______ Q. 35 = ______ Z. 31= ______

I. 202= ______ R. 23 = ______

____ ____ ____ ____ ____ ____ ____ ____ ____ ____ ____ ____ ____ ____
(2)(2) (9)(4) (9)(3) (8 – 1) (1-1) (8)(4) (40)(10) ((8 +1) (16)(4) (2)(5) (8)(2) (4)(2) (3)(12) (4)(1)

____ ____ ____ ____ ____ ____ ____ _____ ____


(2)(5) (5)(5) (36)(6) (7 -7) (5 +4) (7+3) (6 +3) (4)(4)(4) (3+3+1)

II Classify each statement as true or false. If it is false, modify the right side of the
equality to obtain a true statement.

1. 3 4 ⋅ 3 2 = 38 6. a 2 + 2a 2 = 3a 2
2. 25 ⋅ 2 2 = 47 7. (a b)
2 3
= a 6b 3
3. (x ) = x
−3 2 −6 8. a 2 + a 2 = 2a 4
4. (2 ) = 2
0 3 3
9.
b −5
= b −2
b3
62 2
4
24
5. =1 10.   = 4
62 3 3

12
III. Simplify the following :
1. (a 5 )(a )(a 7 )  r −4 k 5 
2

8.  2 

 5k 
2. (5a ) 2 3
9.
 7 m −1 n 3 
 3 − 2 
−1

 n r 
3, (3 y z )(7 y )
3 4
10.
30 x 4 y 7
− 6 x −12 y 2
-3 (ax 3 y ) y2 ⋅ yb
2
4. 11.
 − 1  − 1 
5.  a  (b )(48c ) 12. x 2 a x 3a x 5 a
 8  6 

6. (− 27ay 3 ) − 1 ay 3  13. (3 )(3 )


2 x+6 3 x −4
 3 

7x3 z 5
7.
4 z 15

IV. Answer each question. You may replace the variables with a number and evaluate the results.
m
a  am 
1. For all numbers, is   =  m  a true statement?
b b 

2. Is (a + b ) = a m + b m a true statement for all numbers a and b and any integer m ?


m

am
3. In the quotient of powers property, , why must a be non –zero?
an

V. Find the measure of the area of each rectangle and the measure of the volume of each
rectangular solid.

1. 3.
7
c p4

c7
p

13
3. 4
x2

z3 x3
x3

z6
.

5.

a3

a3

a3

VI. Write each expression in simplified form.

(
1. − 2a 2 b )
4
5.
(x − 2 y )6
( x − 2 y )2
2. (− 2 x y ) (− 3x y )
3 2 2 2 3
 x −1 
6.  3 
−4

 y 
3. (3ab )(− 2a b )
4 3 2
7. (2 x y )
3 2 0

4.
(x y )
2 4
2
(xy )2  a −1b 2 
8.  − 4 0 
c d 

VII. Answer the following :

1. Is x 5 ⋅ x 3 = x15 ? Why?
2. Simplify (a ) . What property did you apply ?
10 3

3. If you were offered two jobs, one that pays 3x pesos and the other pays 3x pesos for x hours
of work, which pay scale would you choose ? Why?

4. After working 8 hours, how much more could you get for working 3 hours if your hourly
rate is 3x pesos ? ( x = number of hours overtime)

14
TEST 2
Answer the following:
1
1. Is − 3 = −23 ? Yes or No? Why?
2

2. Evaluate each expression below when a = 1 and b=2


a) (a )
2 3

b) (a b )
2 2 5

c) b3 ⋅ b4
d.) (b 2 ⋅ b 3 )(b 2 )
5

e) (a ⋅ b)
3 3

3. Simplify:
a) (− 3) (− 2)
2 3

5 x 0 y −2
b)
x −1 y − 2
2 2
 3a 2   − 2a 
c)  3   
 b   3b 
[(− 7) (− 3) ]
−1
2 2
d)

Scoring:

• After answering the test, check your answer with those on the answer key page
• If your score is 7 or higher , you may proceed to the next lesson, otherwise, read the lesson
once again and do the test again
• If you did not make it for the second time, consult your teacher.

15
OPERATIONS WITH ALGEBRAIC EXPRESSIONS

+ + =

x x x x

x + 2x + x = 4x Factors/products
similar terms
Unlike terms
Like terms, combine

Simplifying Algebraic Expressions Examples: Simplify the following:

An algebraic expression is a collection of 1. 6 x + 4 y + 5 x + 3 y


numbers, variables, operations and grouping = 6 x + 5x + 4 y + 3 y by CPA
symbols. = (6 x + 5 x ) + (4 y + 3 y ) Combining like
Expressions separated by + or - are called terms
terms. For example the expression = 11x + 7 y
9 x 4 − x 3 + 5 x 2 + 3x + 1 has five terms. In the
expression 9x 4 , 9 is called the numerical 2. 3x − 4 y − 5 x − 8 y
coefficient and the x is called the literal = 3x − 5 x − 4 y − 8 y
coefficient. If the terms have the same literal = (3 x − 5 x ) + (− 4 y − 8 y )
coefficient and the same exponents then they = −2 x − 12 y
are called like terms or similar terms. For
example, 2xy, 4xy and xy are called like or
similar terms , while 2 x 2 and y 2 are unlike 3. 9k − 4 − 3(2 − 5k )
terms or dissimilar terms. = 9k − 4 − 6 + 15k
= 9k + 15k − 4 − 6
In simplifying algebraic expressions , we = 24k − 10
reduce algebraic expressions to equivalent
expressions that does not contain terms or 4. 3x − 5 − (2 x − 4 )
factors with zero or negative exponent .
= 3 x − 5 − (2 x − 4 )
To simplify expressions by addition or by = 3x − 5 − 2 x + 4
subtraction, start with removing grouping = 3x − 2 x − 5 + 4
symbols the innermost symbol first and work = x −1
from inside out. Combine similar terms as
they appear.

16
5. {3ab − [2ab − 2 + (6 − 3ab )]} − (ab + 1) 6. 4- − [3(3 x − 3) − 6] + 5(4 y − 2 )
= {3ab − [2ab − 2 + (6 − 3ab )]} − (ab + 1) = 4 − [9 x − 9 − 6] + 20 y − 10
= {3ab − [2ab − 2 + 6 − 3ab]} − ab − 1 = 4 − [9 x − 15] + 20 y − 10
= {3ab − 2ab + 2 − 6 + 3ab} − ab − 1 = 4 − 9 x + 15 + 20 y − 10
= {4ab − 4} − ab − 1 = 4 − 9 x + 20 y + 5
= 4ab − 4 − ab − 1 = −9 x + 20 y + 9
= 3ab − 5

ACTIVITY 3
A. 1. Give the numerical coefficients of each term:
a. 3d2 d. − 25mn
2 5
b. x e. − .28kl 2
3
c. qt 6
2. Tell whether the following group of terms are like or unlike
a. 4r, 2r, -5r d. 3c 3 ,4c −3 , c
2
b. t ,−t , t e. 3x 2 y,−5 x 2 y,9 x 2 y
5
c. 4h5 ,−4h −5 ,−5h −4
B. Simplify the following algebraic expressions:

1. 4 x 2 − 5x + 6 x 2 − 2 x
2. − 5(6 y 3 − 4 y 2 + y − 3)
3. 2(3u − 4c ) − (5u − 3c )

4. ( ) (
4 − t 2 + 3st − 2s 2 − 6 7t 2 − st − s 2 )
5. − ( x − 10 ) − 4[2 x − 3(6 + 2 x ) − 5]
6. 3(3h + 3e ) − 2[5(2h + e ) + 8]
7. 2 x + [5 y + 3(2 x − 2 y ) − (3 y − x )] + 4 y

8. 5q 3 2q{q 2 − 3[2q − 5(q + 4) + 3q ] − q 3 }


9. 3(2a + 5b ) − 6[4(3a + b ) + 6]
10. 7 x − {3 y − [9 x − (2 y − 3x )]} − (3x + 6 y )

17
C. 1. Complete the following table by evaluating the expression x + 1 for the given values of x
x 0 1 2 3
x +1

Based on your results, what can you say about the value of x + 1 as the value of x increases
2. Repeat the process in # 1 for the expressions a .(x + 2), b. x + 3 . Describe the results.

3. Repeat the process in # 1 for the expressions x − 1 and x − 2 . Write a brief description
of the results obtained.

Test 3

Simplify the following: (2 points each)


1. 3x − 2(2 x + 3 y ) − (2 x − 3 y )
2. − (− 4d + 5e ) − [− 2e + (4d − 3e )] − 5(e + d )
3. 7 x − {3 y + [8 x − (2 y + 4 x )]} − (4 x + 3)
4. (5w − 2 x ) − {(4 x − 3w) − [10w − (3x + w) + 5 x]}− {− [(18w + 4w) − (2w + 3x )]}
5. 3c − {2a − [(3b + c ) − 4(− 2c + a )] − [− (a + b ) − (2a − 3c )]} − 4[(5a − 3c ) − 4b]

Scoring:
• After answering the test, check your answer with those on the answer key page
• If your score is 7 or higher , you may proceed to the next lesson, otherwise, read the lesson once
again and do the test again
• If you did not make it for the second time, consult your teacher

18
Polynomials
A special type of algebraic expressions is Examples:
the polynomial. a) 3x 4 − 2 x 3 + 2 x 2 + x − 7
How many terms are there in this
Definition:
polynomial? What are these terms? What is
A polynomial is an algebraic expression
the degree of each term?
whose variables have exponents that are non-
negative integers. 2 x 3 + 3x 2 − 5 x + 1 2x +1 is Notice that the terms are
an example of a polynomial, notice the
exponent of each term are non-negative or 3x , − 2 x , 2 x , x and 7. with corresponding
4 3 2

positive also with the exponent of the constant degrees are 4,3,2, and 0, respectively . The
which is equal to zero it is non- negative. greatest degree is 4 . Hence the degree of the
5 polynomial is 4.
However, in the expression 5 + 2 , we can
x
say that this is not a polynomial since x 5 is in b) 6 x y + 3x y z − 2 x yz
3 2 2 2 2 2

the denominator which denotes that the


exponent is negative applying the previous This polynomial has 3 terms. Their
concept on negative exponent. degrees are 4,6 and 5 respectively. Therefore,
Examples: Tell whether the expression is a the degree of the polynomial is 6.
polynomial or not, if not why
1. 8 x 3 + 4 xy a polynomial
The terms of the polynomial are
usually arranged so that the powers of the
2. 3x a polynomial
variable are either arranged in ascending or
2 2 descending order.
3. +5 not a polynomial since 2
x2 x
denotes negative exponent
1
Examples of polynomials arranged in
4. 4 x + 2
2
not polynomial since the exponent is
ascending order are as follows:
not an integer but a fraction
Kinds of Polynomials 1) 7 − x + 2 x 2 − 2 x 3 + 3x 4
2) 1 + x 2 + x 3 + x 4 − 2 x 5
1. Monomial- a polynomial with one term,
a product of a number and variables Examples of polynomials arranged in
Examples: 2x , 4xy and 3x3y2 descending order are shown below.
2. Binomial - a polynomial of two terms
Examples: x+3y , 2x-4yz and 2x2+1
3. Trinomial – a polynomial of three terms 1) 3x 4 − 2 x 3 + 2 x 2 − x + 7
Example: 3x2-2x-1 2) − 2 x 5 + x 4 + x 3 + x 2 + 1
The degree of the polynomial refers to the
greatest exponent or sum of the exponents of
the variables in each term. Let us try these exercises.
Examples: A. Find the degree of each polynomial
below
Monomial Degree
5x2 2 1. 5x 2 − 2 x 5
3 4
4ab c 1+3+4 = 8 2. 7 x 3 + 4 xy + 3xz 3
8 0 3. 13s 2 t 2 − 4st + 5s 3t 3
4. 32 xyz − 11x 2 y + 17 xz 2
The degree of a polynomial, is the same as
the highest/greatest degree of any of its terms. 5. 4 x 3 y + 3xy 4 − x 2 y 4 − x 2 y 3 + y 4

19
B. Arrange the terms of each polynomial so The additive inverse of any number a is
that the powers of x are in descending –a such that a + (− a ) = 0 and (− a ) + a = 0 .
order.
1. − 6 x + x 5 + 4 x 3 − 10 x 4 − 20 On the other hand, you can subtract
3 3 polynomials by adding the additive inverse of
2. x y + 3xy 4 − x 2 y 3 + y 4 the subtrahend to the minuend.
4
Addition and Subtraction of To find the additive inverse of a
Polynomials polynomial, replace each term with its additive
inverse.
We can perform operations in polynomial,
just like the operations with numerals. Polynomials Additive inverse
However, we should always remember that we x + 3y
2
− x2 − 3y
can only add or subtract similar terms. 3 x 2 − 2 xy + y 2 − 3 x 2 + 2 xy − y 2
Otherwise, we simply copy them. − 2 x + 5 xy − 7 z 2 x − 5 xy + 7 z
12 x − 7 x + 9
2
− 12 x 2 + 7 x − 9
Polynomials can be added in different ways:
we can use the associative and the
Example 2 : Subtract :
commutative properties of addition or we can
use the vertical or column format where ( )
12t 2 − 9t − 1 from 4t 3 − 10t 2 + 5t + 8 ( )
similar terms are aligned and their coefficients Method 1: Grouping like terms
are added or subtracted as the case maybe.
(4t 3 − 10t 2 + 5t + 8) − (12t 2 − 9t − 1)
Let us try the following examples: = (4t 3 − 10t 2 + 5t + 8) + (− 12t 2 + 9t + 1)
1. Add = 4t 3 − 10t 2 + 5t + 8 − 12t 2 + 9t + 1
( ) (
4 x 3 − 10 x 2 + 5 x + 8 and 12 x 2 − 9 x − 1 ) ( )
= 4t 3 + − 10t 2 − 12t 2 + (5t + 9t ) + 8 + 1
= 4t − 22t + 14t + 9
3 2

Method 1: Combine like terms (applying


the Associative and the Commutative Method 2: Subtraction in column form
Properties of Addition)

(4 x 3
− 10 x 2 + 5 x + 8) + (12 x 2 − 9 x − 1)
(4t 3
− 10t 2 + 5t + 8) − (12t 2 − 9t − 1)

= 4 x 3 + (12 x 2 − 10 x 2 ) + (5 x − 9 x ) + (8 − 1)
= 4x3 + 2x 2 − 4x + 7
4t 3 − 10t 2 + 5t + 8 ⇒ 4t 3 − 10t 2 + 5t + 8
Or Method 2: Using the vertical or - 12t 2 − 9t − 1 ⇒ + − 12t 2 + 9t + 1
column format ________________________________________________________________________________________

4t 3 − 22t 2 + 14t + 9
(4 x − 10 x 2 + 5 x + 8) + (12 x 2 − 9 x − 1)
3

4 x 3 − 10 x 2 + 5 x + 8
+ 12 x 2 − 9 x −1
_______________________________________

4x3 + 2x 2 − 4x + 7

20
Example 3: Given ∆ABC, the perimeter of S = 12 x 2 − 7 x + 9 − 3 x 2 − 2 x + 1 − 5 x 2 + 8 x − 5
∆ABC is P = 12 x 2 − 7 x + 9 and the two sides S = (12x 2 − 3x 2 − 5x 2 ) + (− 7x − 2x + 8x) + (9 + 1 − 5)
have lengths 3x 2 + 12 x − 1 and 5 x 2 − 8 x + 5 .
S = 4x 2 − x + 5
Find the measure of the third side.
Solution:
Perimeter is the sum of the measures of the Therefore , the measure of the third side of
three sides of the triangle. Let S represent the the triangle is S = 4 x 2 − x + 5
measure of the third side of the triangle.

P = S + (3 x 2 + 2 x − 1) + (5 x 2 − 8 x + 5)
( ) (
P − 3x 2 + 2 x − 1 − 5 x 2 − 8 x + 5 = S )
ACTIVITY 4

A. State whether each expression is a polynomial. If it is so, identify the kind of polynomial. If
it is not a polynomial , explain why.
a2
1. 4 x 2 − 2 xy 4.
3
6 2 x 1
2. 2
− +1 5. x2 − +
x x 2 3
1
3. x 2 + 2x

B. Find the degree of each term of the polynomial and the degree of the polynomial

1. 2 x 2 − 3 x 3 4. 6 x 2 y 3 − 5 x 3 y 2 − x 4 y 2
2. 10 x 2 y 4 − 2 x 4 y 2 − x 3 y 3 5. 17bx 2 + 11b 2 x − x 2
3. − 5 xyz 4 + 10 x 2 y 4 z

C. Arrange the terms of each polynomial so that the power of x are in descending order.

1. - 6 x + x 5 + 4 x 3 − 20 2. 4 x 3 y + 3 xy 4 − x 2 y 3 + y 4

Test 4
Answer the following:
2 3
1. Is x + 3x 2 − 2 x 0 a polynomial? why?
5
2. Using the expression in no 1 problem find the degree of each term.

3. Are the expressions 3 x 2 , 3 y 2 , 3 z 2 similar terms? why?

21
Activity 5
Answer the following:

1. What is the first step when adding or subtracting polynomials in column form?
2. What is the best way to check on the subtraction of polynomials?
3. Give the additive inverse of each polynomial:
a. x 2 − 4 x + 7
b. − 4h 2 + 6hk − 2k 2
c. 5ab 2 + 6a 2 b + 1
d. 4 x 2 + 2 x − 1

4. Find each sum or difference


a. (3 x 2 5 x − 5) + (5 x 2 − 7 x + 9 )
b. (4 x 3 + x 2 + 2 x − 4 ) − (2 x 2 − 3 x + 5)
c. (a 3 − b 3 ) + (3a 3 + 2a 2 b − b 2 + 2b 3 )
( ) (
d. 5ax 2 + 3a 2 x − 5 x + 2ax 2 − 5a 2 x + 7 x )
e. (3 x − 7 y ) + (3 y + 4 x )
4a + 5b − 8c − d
f.
2 a + 7b − 2c + 8d
+ 2a - b + 6d

5. Find the measure of the side S of the square if


the perimeter of the square is P = 4x-4y .
S
S

Test 5
Find the sum or difference of the following:

1. 12m 2 n 2 + 3mn − 11
- 5m 2 n 2 − 8mn + 7

2. 2 x 2 − 5x + 7
5x 2 + 7 x − 3
+ x 2 − x + 11

( ) (
3. n 2 + 5n + 4 + 3n 2 + 8n + 8 = )

22
4. (3 + 2a + a ) − (5 + 8a + a ) =
2 2

5. Given the perimeter P of the hexagon and the measure of the five sides of the hexagon
Find the measure of the side S.
P = 11x 2 − 29 x + 10 x 2 + 3x + 24

x2 −1 2x + 3

x+4 x 2 − 4x

6. Mario decided to construct a square casing for his component if one side
measures (4 x 2 +2 x + 8) cm , what will be the perimeter of the square casing.

Scoring:
• After answering the test, check your answer with those on the answer key page
• If your score is 7 or higher , you may proceed to the next lesson, otherwise, read the lesson once
again and do the test again
• If you did not make it for the second time, consult your teacher

23
Multiplication of Polynomials by a
Monomial 1. Product of two binomials
A 6x cm 7cm B
The distributive property can be used in 2x I II
multiplying a polynomial by a monomial.
3 cm III IV
Examples:
1) 3x(2 x + 4 ) = 3 x(2 x ) + 3 x C D
= 6 x 2 + 12 x The area of the rectangle is the product of the
2) 2a (4a − 3a + 5) = 2a (4a ) + 2a (− 3a ) + 2a (5)
2 2 length and width . You can multiply (6 x + 7 )cm
= 8a 3 −6a 2 + 10a and (2 x + 3)cm to find the area of the rectangle.
Notice that this big rectangle ABDC is
(
3) 3xy 2 x 2 y − 3 xy 2 − 7 y 3 ) composed of four smaller rectangles I,II, III and
= 3 xy (2 x y ) + 3 xy (− 3 xy 2 ) + 3 xy (− 7 y 3 )
2 IV , Therefore, to find the area of the big
rectangle we get sum of the area of the four
= 6 x 3 y 2 − 9 x 2 y 3 − 27 xy 4 small rectangles.
4. Find the measure of the area of the
shaded region in simplest terms. (6 x + 7 )(2 x + 3) = 6 x(2 x ) + 6 x(3) + 7(2 x ) + 7(3)
= 12 x 2 + 18 x + 14 x + 21
3a 2 + 5a − 4 = 12 x 2 + 32 x + 21

3a The above solution illustrates the short cut


method for multiplying binomials ,that is, the
6 FOIL method (first terms, outer terms, inner
terms, last terms)

F O
(6 x + 7 )(2 x + 3) = 6 x(2 x ) + 6 x(3) + 7(2 x ) + 7(3)
2a2 − 4a + 1
I L

Solution:
F O I L
Subtract the area of the smaller rectangle
from the measure of the area of the larger FOIL METHOD :
rectangle
(Area of big rectangle – Area of small rectangle ) To multiply binomials using FOIL Method,
( )
3a 3a + 5a − 4 − 6(2a − 4a + 1)
2
find the sum of the products of
= 9a 3 + 15a 2 − 12a − 12a 2 + 24a − 6
= 9a 3 − 3a 2 + 12a − 6 F- the First Terms I – the Inner Terms
O – the Outer Terms L – the Last Terms

24
Examples: ( )( ) (
3. x 2 + 5 x − 4 2 x 2 + x − 7 − 4 2 x 2 + x − 7 )
Find the indicated product:
( ) ( )
= x 2 x 2 + 5 x − 4 + 5 x 2 x 2 + x − 7 − 8 x 2 − 4 x + 28
1. ( x + 5)(x + 6 ) = x 2 + 6 x + 5 x + 30
= x 4 + 5 x 3 − 4 x 2 + 5 x 3 + 5 x 2 − 35 x − 8 x 2 − 4 x + 28
= x 2 + 11x + 30
= x 4 + 10 x 3 − 7 x 2 − 39 x + 28

2. (2 x + 3)(5x + 8)
( )( ) ( )( ) ( ) ( )
= 2x 5x + 2x 8 + 3 5x + 3 8
= (10 x ) + 16 x + 15 x + 24
2

= 10 x 2 + 31x + 24

ACTIVITY 6

1. Find the sum of the product of the inner terms and the product of the outer terms:

a) (5b − 3)(2b + 1) c. (4a + 3)(2a − 1)


b) (2a + 1)(3a − 2 ) d. (5r − 7 s )(4r + 3s )

2. Find the product


a) (c + 2 )(c − 8) c. (x + 7 x − 9)(4 x − 3 y )
2

b) (2a + 3b )(3a − 2b ) d. (5x − 6 x + 9)(4 x + 3x + 11)


2 2

3. Complete the product of the following :


a . ( x − 2 y )(2 x − y ) = 2 x 2 + _______ + y 2
b. (3a + 2c )(2b − 3c ) = 6b 2 + _____ − 6c 2

c. (q − 2 y )(q 2 + y ) = _____+ qy − 2q 2 y − 2 y 2
d. (12 x 2 + 3 y )(x − y ) = 12 x3 + _____ + _____ − 3 y 2
4. Ben owns a fishpond whose length is 4 meters more than the width. A path of 2 meters
surrounds the fishpond with a total area of 216 m2. What are the dimensions of the fish
pond?

5. Marlo wishes to construct a perimeter fence around his farm. If the dimensions of the farm
are (2x + 4 y ) by (3x − 5 y ) respectively, find the polynomial that will represent the length of the
wire that will be needed to fence his farm.

25
10

2x + 5

6. Refer to the above figure : a) Find the polynomial that represents the area of the rectangle.
b) Assuming that the area of the rectangle is 150 meter2, find the length
of the rectangle.
c) Suppose the rectangle is your living room and you want to change
the floor tiles into bigger tiles. If it costs P65 per square meter,
how much will it cost you to tile the entire living room?

Test 6
Find the product:

1. (7 y − 1)(2 y − 3)
2. (6 x 2
− 5 xy + 9 y 2 )(5 x − 2 y )

3. − 8d 2 (4d + m )(4d − m )
4. If R= 2x-1, S=3x+2, and T = -3x2, find each of the following:

a) R⋅S b) R(S + T ) c) S ⋅ T d) T (R + S )

5. Find the polynomial that represents the area of the shaded region .
y +8
a.

y
y +8
y

Scoring:
• After answering the test, check your answer with those on the answer key page
• If your score is 7 or higher , you may proceed to the next lesson, otherwise, read the lesson once
again and do the test again
• If you did not make it for the second time, consult your teacher

26
Division of Polynomials Example 2: Divide 18a 4 b 2 − 12a 2 b + 6ab by
1. Division of Monomials by Monomials 3ab
Dividing monomials by monomials is done by 18a 4 b 2 12a 2 b 6ab
considering the variables and their exponents. In this Solution : − +
case, we have to follow the laws of integral exponents. 3ab 3ab 3ab
Let us recall the rule on Quotient of Powers = 6a b − 4a + 2
3

am Here is the standard form of writing the


1. n = a m − n if m >n ,
a division of Polynomials
a3 P ( x) R ( x)
Example 2 = a 3− 2 = a = Q( x) + , where P(x) is the given
a D( x) D( x)
m
a 1 polynomial, D(x) is the Divisor, Q(x) is the
2. n = n − m if n>m,
a a quotient and R(x) is the remainder if there is.
a2 1 1 3. Division of Polynomial by a Binomial
Example 3 = 3− 2 =
a a a Steps in dividing polynomial by a
m
a Binomial
3. n = 1 if m=n,
a 1. Arrange the polynomial in descending
a3 order of the exponents of common
Example : 3 = a 3−3 = a 0 = 1
a variable.
Here are the steps in dividing a 2. Using the long division, divide the first
monomial by another monomial: term of the dividend by the first term of the
Step 1. Divide the numerical coefficients quotient. Multiply the divisor by the
following the rules for dividing divisor and then subtract the product from
integers. the dividend.
Step 2. Divide the variables by subtracting
the exponents of each like terms. 3. Using the remainder as the new dividend,
repeat the process to obtain the next term
Example 1. Divide 6x 3 by 2x 2 of the quotient.
6x3 4. Do the same procedure until the remainder
Solution: = 3x
2x 2 is zero or a polynomial with degree less
than the degree of the divisor.
Example 2. 12a 4 b 6 ÷ 4a 2 b 2
Example
12a 4 b 6 1. Divide 42c 3 + 46c 2 d − 22cd 2 + 30d 3 by
Solution: = 3a 2 b 4
2 2
4a b 6c + 10d
7c 2 − 4cd + 3d 2
2. Division of Polynomial by a Monomial
6c + 10d 42c 3 + 46c 2 d − 22cd 2 + 30d 3
To divide a polynomial by a monomial we
simply write the division as product, and use the − (42c 3 + 70c 2 d )
distributive property and simplify each resulting
fraction. − 24c 2 d − 22cd 2
Example1: Divide ( 12 x 2 + 4 x ) by 2x - (− 24c 2 d − 40cd 2 )
12 x 2` + 4 x 12 x 2 4 x 18cd 2 + 30d 2
Solution: = + = 6x + 2
2x 2x 2x - 18cd 2 + 30d 2
0

2. Divide x 2 − 2 x + 1 by x + 1
Solution:

27
x−3 r 4
x + 1 x 2 − 2x + 1
- (x 2 + x )
− 3x + 1
- (− 3 x − 3)
4

4
Answer: ( x − 3) +
x +1

ACTIVITY 7
Find the quotient:
27a 2 − 8a 4
1. 4. 4 2 x ÷ (− 4 3 x )
9a 2
15r 6 y 6 − 20r 3 y 4 + 10r 8 y 7 15 x 2 − 10 x 3
2. 5.
5r 2 y 3 5x

2 x 3 + x 2 y + xy 2 + 4 y 3
3. 6. 6e y ÷ 3e y
x+ y
7. The area of a rectangular garden is 2d 3 + 7d 2 − 5d − 4 sq. meters, Find the length if
the width is 2d + 1

Test 7
Divide :
16r 4 s 2 − 8r 3 s
1. (2 points)
4rs
20 x 5 − 12 x 4 y − 2 x 3 y 2 10 x 2 y 3 − 4 xy 4 + 8 y 5
2. (2 points)
4x 2 − 2 y 2
3. A vegetable garden has 3h 3 − 2h −1 vegetables planted in h-1 rows , with equal number of
vegetables in each row . How many vegetables are planted in each row. (3 points)
( )
4. Show that ( x + y ) is a factor of x 3 + y 3 (3 Points)
Scoring:
• After answering the test, check your answer with those on the answer key page
• If your score is 7 or higher , you may proceed to the next lesson, otherwise, read the lesson once
again and do the test again
• If you did not make it for the second time, consult your teacher.

28
SPECIAL PRODUCTS

Discovering Patterns in Special Product


2. Using the marks on the product mat as
Some products of binomials occur guide, make the rectangle with algebra
so often in the study of Algebra that we tiles
easily notice or identify patterns. x+3
Try to discover the patterns in order to get
the product of the following binomials
x
x x
A. Use algebra tiles to find the product x2
x x x
(x + 2)(x + 3) . x+2

1. The rectangle has a width of x+2 x x 1 1 1


1 1
and length of x+3. Mark off the dimension
on the product mat xx 11 11 11
1 1 1
x

1
1
3. The rectangle has one x2 –tile, and five x-
tiles and six 1-tiles. The area of the rectangle
then is x 2 + 5 x + 6 . Thus,
(x + 2)(x + 3) = x 2 + 5x + 6

29
B. Find the product ( x − 1)( x − 1) C. Find the product ( x + 1)( x − 1)

1. The rectangle has a width of x – 1


and the length of x-1 units. 1. The rectangle has a length of x +1 and
width x -1 units
x -1
x x 1

-1
x

1
2. Using the marks on the product as
guide make the rectangle using the 2. Using the marks as guide to make the
algebra tiles rectangle using the algebra tiles
x −1 a x ←I
(x-1 )
x
2 -
x x x(x-1)
x −1 x

-x 1

1(x-1 ) 1

x +1

3. The rectangle has one x2 – tile, two 3. The rectangle has one x(x-1) – tile ,
x – tiles, and one one unit-tile. one(x-1) – tile. The one x-tile and one
The area of the rectangle is one unit tile are to be subtracted from the
x2 − 2x + 1 . resulting figure The area of the rectangle
is x(x-1) +(x-1) = x2 -1
4. Hence
( x − 1)( x − 1) = x 2 − 2 x + 1
4. The product of (x+1)(x-1) = x2 -1

30
Multiplication using algebra tiles c) (abc + 2 )(abc − 2 ) = (abc ) − (2 )
2 2

illustrates special products


= a 2b 2c 2 − 4
Let us have more exercises on special
The product of the sum and difference of two
products:
terms is equal to the difference of the squares
of terms
Example 1. Product of Two Binomials
(2 x + y )(3x − 2 y ) = 6 x 2 − 4 xy + 3xy − 2 y 2 Example 4. (Cube of a Binomial)
= 6 x 2 − xy − 2 y 2
(3b + 2c )(2b + 3c ) = 6b 2 + (9 + 4)bc + 6c 2 a) ( x + y ) = x 3 + 3 x 2 y + 3 xy 2 + y 3
3

b) ( x − y ) = x 3 − 3 x 2 y + 3 xy 2 − y 3
3
= 6b 2 + 13bc + 6c 2
What pattern have you discovered?
How do you find the product of this kind of
binomial? Cube of Binomial The product is equal to
the cube of the first term plus /minus three
Example 2. (Square of Binomials) times the product of the square of the first
a) (2 x − y )(2 x − y ) term and the last term plus the 3 times the
first term and the square of the last term
= (2 x ) − 2(2 x )( y ) + y 2
2
minus/plus the cube of the last term.
= 4 x 2 − 4 xy + y 2
Example 5: Squaring a trinomial- this is an
b) (x + 3 y )(x + 3 y ) 2
extension when you square a binominal
= x 2 + 2(x )(3 y ) + (3 y ) 2
a) (2a − 3b − c ) = [(2a – 3b) – c]2
2
= x + 6 xy + 9 y
2 2

= (2a – 3b)2 – 2c(2a-3b) + c2


c) (4b + c ) = (4b ) + 2(4b ) c + c 2
2 2
( ) = (2a)2 - 2(2a)(3b) +(3b)2-2c(2a-3b) +c2
= 16b 2 + 8bc + c 2 = (2a ) + (− 3b ) + (− c ) + 2(2a )(− 3b ) +
2 2 2

2(2a )(− c ) + 2(− 3b )(− c )


Is there a shorter way of squaring the
binomial? = 4a + 9b 2 + c 2 − 12ab − 4ac + 6bc
2

Squaring Binomials,
b) ( x + 2 y + 3z ) =
2
square the first terms and also the
last term and add to these twice the = ( x ) + (2 y ) + (3 z ) + 2( x )(2 y )
2 2 2

product of the two terms.


+ 2(x )(3z ) + 2(2 y )(3z )
Example 3. (Sum and Difference of Two = x 2 + 4 y 2 + 9 z 2 + 4 xy + 6 xz + 12 yz
Terms)
a) (a − b )(a + b ) = a 2 − b 2
b) (4 x − y )(4 x + y ) = (4 x ) − y 2
2

= 16 x 2 − y 2

x− y

31
Let us try the following 3. x

Each picture illustrates an algebraic fact,


y
complete the statement
1. a b

b x x(x − y )
b b
x+y
=
a a a a

y2
y y (x − y )
a b
a b
(x + y )(x − y ) = _____________
b b

In summary, the different special products:

a a 1. Product of Two Binomials


(ax + b )(cx + d ) = acx 2 + (ad + bc )x + bd
2. Product of a Sum and Difference of Two
a b
Terms
(a + b )(a + b ) = ___________ (x + y )(x − y ) = x 2 − y 2
(a + b )2 = _____________ 3. Square of a Binomial
( x + y ) 2 = x 2 + 2 xy + y 2
x+2 2
(x − y )2 = x 2 − 2 xy + y 2
2. x 4. Cube of a Binomial
(
(x + y )3 = (x + y ) x 2 − xy + y 2 )
x+1
=
(x − y ) = (x − y )(x
3 2
+ xy + y 2
)
x x x 5. Square of a Trinomial

(x + y + z )2 = x 2 + y 2 + z 2 + 2 xy + 2 xz + 2 yz
(x+1) (x+2) = ________
1 1

32
Activity 8
A. Find the indicated product

1. (2a − 7b )(3a + 5b )
2. (4 x − 3)(x + 4)
3. (x − 6)(x + 6)

4. (2 x + y )2

5. (2b + 2c + d )2

B. What does the diagram at the right represent if the y


shading represents regions to be removed or
subtracted.
y

x− y
x

x− y

a b c
C. The diagram at the right represents
(a + b + c )2 . Use the diagram to write a
the polynomial that will represent the
area of the figure.

33
Test 8.
A. Find the product of the following
1. ( x − 3)( x + 4 )(x + 3)( x − 4 )
2. (7 a 2 + b )(7 a 2 − b )
2
1 
3.  a + k 
2 
4. (4 x − 5)(4 x − 5)( x − 4 ) .
4 2  4 2 
5.  d − h  d + h 
3  3 

B. Find the missing term.

1. (2 x + y )2 = 4 x 2 + _____ + y 2
2. (3x − 2 y )2 = ______ − 12 xy + 4 y 2
3. (2x – 4y)(x+y) = 2x2 _______ - 4y2
4. (2 y ) − (3z ) = (2 y − 3z )(_____________ )
3 3

5. (a − b )2 − (c + d )2 = _________[(a − b ) + (c + d )]
Scoring:
• After answering the test, check your answer with those on the answer key page
• If your score is 7 or higher , you may proceed to the next lesson, otherwise, read the lesson o
again and do the test again
• If you did not make it for the second time, consult your teacher.

34
Identifying and Finding Special
Products 2. Using problem 1 , check the
answer by using (a + b ) (a + b ) .
3

In the previous lessons you have learned


and discovered the patterns on finding special Solution:
products.

Given another problem,can you apply the (a + b )3 (a + b ) =


formulas in the special products to solve it? (
= a 3 + 3a 2b + 3ab 2 + b3 (a + b ) )
= a 4 + 3a 3b + 3a 2b 2 + ab + a 3b + 3a 2b 2
1. Find a pattern for the fourth
+ 3ab3 + b 4
power of the binomial (a + b ) .
4

= a 4 + 4a 3b + 6a 2b 2 + 4ab3 + b 4
How are you going to get the product
indicated? What special product formula are
you going to use?

Solution: 3. Find the product of (x 2


+y )
4

In this type of problem you can apply the


square of a binomial Solution :

(a + b )4 = (a + b )2 (a + b )2 (x 2 4 2 2
[
+ y ) = (2 x 2 + y ) (2 x 2 + y ) or (2 x 2 + y ) ]
2 2

( )(
= a 2 + 2ab + b 2 a 2 + 2ab + b 2 ) (
= 4x4 + 4x2 y + y 2 )
2

= a + 2a b + a b + 2a b + 4a b + 2ab +
4 3 2 2 3 2 2 3

a 2 b 2 + 2ab 3 + b 4 =
16 x8 + 16 x 4 y 2 + y 4 + 16 x 6 y + 4 x 4 y 2 + 4 x 2 y 3
= a 4 + 4a 3 b + 6a 2 b 2 + 4ab 3 + b 4

Activity 9
A. Use special product formulas to find the following:
1. (2 g − s )(2 g + s ) 6. (3x 2
−4 )
2

2. (m 3
+3f 2
)(m 3
−3f 2
) 4  4 
7.  x 2 − y  x 2 + y 
3  3 
( y − 5)(2v + r )( y + 5)(2v − r ) 8. [(8a − 2b ) + 3]
2
3.
4. (9a − 2q )2 9. [( p 2
− 3q 2 − 2q 2) ]
3

5. [(2a − c ) − 5 f ][(2a + c ) + 5] 10. [(a + b ) + c + (d − e)]2

35
B. The figure below shows the model for ( x + 2 )(x + 1) separated into four parts. Write a
paragraph explaining how the model shows the use of distributive property

1
x

x x

2
2
x 1

Test: 9
A. Tell whether each statement is true or false. Justify your answer with a
drawing

1) (x + 2)(x − 2) = x 2 − 4 2) (x + 3)(x − 2) = x 2 − x − 6
B. Find the missing term:

1. (2 x + 2 y )2 = 4 x 2 + _____ + 4 y 2
2. (2 x − 7 )2 = 4 x 2 + _____ + 49
C. Find the indicated product :

1. (u + 7v )
2 3

2. [(x − x ) + (x
5 3 2
][ ]
− x ) (x 5 − x 3 ) − (x 2 − x )
3. [(2 x + 3 y ) − (5 x − 4 y )] 2

4. (3 p + 2r 2 )
4

Scoring:
• After answering the test, check your answer with those on the answer key page
• If your score is 7 or higher , you may proceed to the next lesson, otherwise, read the lesson
once again and do the test again
• If you did not make it for the second time, consult your teacher

36
FACTORING

The living room is the place where you and your family may possibly bond together. Stories are
shared among members of the family and sometimes kept secret within the four walls of the room.

This is also a place where you seem to be comfortable especially in entertaining people.
Arrangement of the furniture and appliances is important to make the room cozy and comfortable
thus the area of the room is important in determining the size and the kind of furniture the room
should have.

The application of the formula for finding the area of the rectangle, A = lw , in solving problems
leads to a trinomial expression of degree 2. In this unit, you will learn the reverse operation of
multiplying binomials , that is, you are going to look for the expressions to be multiplied to get the
given product, this is called factoring .

37
Factoring can be best illustrated by using
Algebra Tiles -x -x -x -x
A. Factoring using the Distributive Property 2
x
Using the algebra tiles model , you
know the area of the rectangle and are asked
to find the length and the width. b. Arrange the tile into a rectangle
1. Use algebra tiles to factor 3x + 6 x−4
a. Model the Polynomial 3x +6

1 1 1 -x -x -x -x
x x x x
1 1 1

c. Therefore the rectangle has a width of x


b. Arrange the tiles into a rectangle on the units and length of x - 4
product mat
x +2 A = lw
= x 2 − 4 x = x( x − 4 )

3
Factors of x 2 − 4 x are x and x+4

4. Use algebra tiles to factor x 2 − 4 x + 4


a. Model the polynomial x 2 − 4 x + 4

c. The rectangle has a width of 3 and a


1
length of x + 2 . Therefore -x -x -x -x
2
x
3 x + 6 = 3( x + 2 ) . Hence the factors of 1
3x + 6 are 3 and ( x + 2 ) 1 1

2. Use algebra tiles to factor x 2 − 4 x b. Place the x2 – tile to the corner of the
product mat and arrange the 1 – tiles into 2 by
2 rectangular array
a. Model the Polynomial x 2 − 4 x x-2
2 -x -x
x

x-2
-x 1 1

-x 1 1

38
In the previous lessons you multiplied Another way of finding the GCF of the
two binomials to find the product. Here we are to factors
study factoring 25 x 2 y 2 = 5 ⋅ 5 ⋅ x ⋅ x ⋅ y ⋅ y
Factors Product 5 xy = 5 ⋅ x ⋅ y
(x + y )(x − y ) = x − y2
2
GCF = 5 ⋅ x ⋅ y = 5xy
( x + y )2 = x 2 + 2 xy + y 2 Example2: Factor the expression
or ( x + y )( x + y ) = 18 x 2 + 12 x .
(x − y )3 = x 3 − 3 x 2 y + 3 xy 2 − y 3
What factor is common to the two
terms of the given expression?
(x + y )3 = x 3 + 3 x 2 y + 3 xy 2 + y 3
Solution:
( x + y + z )2 = x 2 + y 2 + z 2 + 2 xy + 2 The common factor is 6x. Therefore
the factors are 6x and 3 x + 2 . Thus,
The factors are multiplied together to get the 18 x 2 + 12 x = 6 x(3x + 2 )
product. This time you will do the reverse Example 3: Find the factors of
operation that is given the product , you are going 12 x 2 y 2 + 3 x 4 y + 6 x 3 y 3
to look for the factors. The process of doing it is
called factoring or finding the factors. Notice that 3 x 2 y is the greatest
common factor. Therefore the other
FACTORING- is the process of expressing the factor is (4 x y + x 2 + 2 xy ) . Thus,
given number or expression in terms of its prime 12 x 2 y 2 + 3 x 4 y + 6 x 3 y 3 =
factors
Types of Factoring
(
3x 2 y 4 x y + x 2 + 2 xy )
B. Factoring Special Products
Difference of two squares- Factors of
A. Greatest Common Factor –
this product are two binomials which are
To find the greatest common factor of two or the sum and difference of the square root
more expressions, determine the product of of the two squares.
prime factor common to the given expression.
To recognize that it is the difference
Example: Find the factors of 25 x 2 y 2 − 5 xy . of perfect square for the variables raised
Solution: Notice that the term common to both to the second power.
is 5xy. Find out the other factor by dividing each Examples:
term by 5xy. 1) x 2 − y 2 = ( x + y )(x − y )
25 x 2 y 2 5 xy 2) 4a 2 − b 2 = (2a − b )(2a + b )
− = 5 xy − 1
5 xy 5 xy
(
3) 49a 4 − 4b 2 = 7 a 2 − 2b 7 a 2 + 2b )( )
Checking:
4
(
4) 8 j − 18k = 2 4 j − 9k
4 4 4
)
5 xy(5 xy − 1) = 25 x 2 − 5 xy What do you notice in problem
number 4?
Therefore, the factors of 25 x 2 y 2 − 5 xy are:
5xy and 5xy -1. Hence,
(
One of the factors 4 j 4 − 9k 4 can )
still be factored since this is still the
25 x 2 y 2 − 5 xy = 5 xy (5 xy − 1) .
difference two squares. Therefore, the
complete factorization is:

( )(
2 2 j 2 − 3k 2 2 j 2 + 3k 2 )

39
2. Perfect Square Trinomial

Example: Factor x 2 + 2 xy + y 2 , this is a perfect


square trinomial. How many terms are there ?
Examples: Factor:
What do you notice about the first and the last
terms? What can you say about the middle term?
(
1. m 3 − n 3 = (m − n ) m 2 + mn + n 2 )
To recognize a perfect square trinomial, the (
2. 8a 3 − b 3 = (2a − b ) 4a 2 + 2ab + b 2 )
first and the last terms must be positive and perfect (
3. g 3 − 27 h 3 = g 3 − 33 h 3 )
squares, and the middle term is twice the product
of the square roots of the two terms. (
= ( g − 3h ) g 2 + 3 gh + h 2 )
Product Factors How do you factor the difference of
x + 2 xy + y = ( x + y )( x + y ) or ( x + y )
2 2 2
two cubes?

How do you factor a perfect square trinomial? 1. Get the difference of the cube
roots of the first and second terms,
Get the square root of the first term and the This is the first binomial factor.
third term. Then copy the sign of the middle term 2. To get the trinomial factor square
raised this binomial to the second power. the first term of the binomial factor
plus the product of the cube root of
More examples: Factor the two terms and square the last
1. x 2 − 4 x + 4 = (x − 2 )
2
term.
2. 9 x 2 − 6 x + 1 = (3 x − 1)
2
Now, if we are asked to factor
3. x 2 + 16 x + 64 = ( x + 8)
2
x +y ,
3 3

The factors are:


3 Sum or Difference of Two Cubes
a. Is x 3 − y 3 = (x − y ) x 2 + xy + y 2 ? ( ) (
b. x 3 + y 3 = ( x + y ) x 2 − xy + y 2 )
Verify the product of ( x − y ) x 2 + xy + y 2 .( ) Examples: Factor:
Solution:
x 2 + xy + y 2
(
1. d 3 + h 3 = (d + h ) d 2 − dh + h 2 )
x− y
2. 125q 3 + 27t 3 = (5q ) + (3t )
3 3

( )
_____________________________________

− x 2 y − xy 2 − y 3 = (5q + 3t ) 25q 2 − 15qt + 9t 2


x 3 + x 2 y + xy 2
27 k 3 + 1 = (3k ) + 1
3
x3 − y3 3.
This shows that ( x − y )(x 2 + xy + y 2 ) are the (
= (3k + 1) 9k 2 − 3k + 1 )
factors of x 3 − y 3 .
4. Factoring Quadratic Trinomials
ax 2 + bxy + cy 2 = ( px + ny )(qx + my )

a = pq
where b = pm + nq
c = nm

40
Example: Factor the following completely:

1. 4 x 2 − 35 xy − 9 y 2 = ( x − 9 )(4 x + 1)

2. x 2 + 6 x + 8 = ( x + 4 )( x + 2 )

3. x 2 − 5 x + 6 = ( x − 3)( x − 2 ) Solution:
3ab − 21b + 5a − 35 =
( Group terms that have common
monomial factor)
= (3ab − 21b ) + (5a − 35)
5. Factoring by Grouping
(Factor )
In your previous lessons have tried factoring = 3b(a − 7 ) + 5(a − 7 )
binomials also with trinomials, what if you will be ( a-7 is the common factor by
asked to factor polynomials with four terms, what distributive property)
will you do? = (3b + 5)(a − 7 )
Therefore ,the dimensions of the picture
Example: frame are (3b + 5) and (a − 7 )
Other examples:
1.Factor: 8 x 2 y − 5 x − 24 xy + 15
Solution:
8 x 2 y − 5 x − 24 xy + 15
( )
= 8 x 2 y − 24 xy + (− 5 x + 15)
= 8 xy ( x − 3) -5 ( x − 3)
The area of a rectangular picture frame is = (8 xy − 5)( x − 3)
(3ab-21b+5a-35) cm 2 what are its dimensions?
2. Factor 2 f 2 + 2 gc + fg + 4 fc
You need to find the length and the width of Solution:
the rectangular picture frame and since ( )
= 2 f 2 + fg + (2 gc + 4 fc )
Area = length x width , so you need to factor the
polynomial in order to get the dimensions. = f (2 f + g ) + 2c( g + 2 f )
= ( f + 2c )(2 f + g )

41
ACTIVITY 10
A. Factor the following completely:

1. 6a 7 b 3 − 32a 8 b 4 8. 12ax + 20bx + 32cx


2. 144b 2 − 100 9. c 2 + 2c − 3
3. 3cb 3 − 81cd 3 10. 5 x 2 − 13 x − 6
4. x 2 + 2 x − 24 11. 2ax + 6 xc + ba + 3bc
5. 10(x − 2 y ) + 19( x − 2 y ) + 6
2
12. a 2 − 2ab + a − 2b
6. 9k 2 + 30km + 25m 2 13. 5a 2 − 4ab + 12b 3 − 15ab 2
7. 64a 3 b 3 − 27c 3 14. 28a 2 b 2 c 2 + 21a 2 bc 2 − 14abc

B. When you factor 15a +6a2, what property did you use?
C. The perimeter of a square is 16 y 2 + 8 y . Find the area of the square.

Test 9

Fill in the blanks: (2 points each)

1. The factors of the trinomial x 2 − 5 x + 3 are ( x + 2 )(_____ ).


2. The middle term of the perfect square trinomial 4 x 2 + ____ + 9 y 2 is equal to _____.
3. The factors of y 2 + 12 y + 27 are _______ and _______.
4. Factor completely. b 9 − c 9 .

Scoring:
• After answering the test, check your answer with those on the answer key page
• If your score is 7 or higher , you may proceed to the next lesson, otherwise, read the lesson
once again and do the test again
• If you did not make it for the second time, consult your teacher

42
UNIT TEST
I Fill in the blanks.
1. _______________ refers to the expression whose value does not change.
2. The degree of the polynomial 4 xy 3 + x 2 y is _____________
3. _______________ must be added to 7m 2 − 2mn + 5n 2 to get 8m 2 + mn − 2n 2 .
4. The middle term of the product of (3 x − 5)(5 x + 2 ) is ______________.
5. The factored form of 16m3n 2 + 2mn − 32m 2 n 4 is _____________
6. The value of the polynomial 4 x 3 − 3 x 2 y + 3 xy 2 + 2 y 3 when x= 1 and y = -1 is
________________
7. The product of the expression (ax + b ) is _____________
3

8. Given are measures of the sides of the triangle,


The perimeter of the triangle is _____________
2x-5 3x+2

3x-4

36 x 4 y 6 z 10 − 12 x 2 y 2 z 3
9. The simplified form of is _______________
6 xyz 2
10. The equivalent mathematical expression of the phrase “ The quotient of eight times the
product of the square of m and cube of n and the sum of m and n” is ______________.

11. One factor of 3a 2 + a − 2 is (3a − 2 ) the other factor is _______________.


( )
12. Find the dimensions of the rectangle whose area is 9z 2 − 12z + 4 m 2 is ____________.
13. The two sides of a school garden of uniform width are
utilized as sidewalks as shown in the figure. x
2
If the area of this side walk is 82 m , What is its width ?
Width = _______ 15 m
10m

14. In the expression (3a + b ) a 2 − (3a + b )b 2 the common factor x


is _____________.
15. In symbols “ The average of M,N,O, and P” is ____________

43
II A. Perform the indicated operation and simplify.

1. Add 4 p 3q + 8 p 2 q − 2 pq , 2 p 3q − 2 p 2q + 8 pq , 6 p 3q + 12 p 2 q + 4 pq
2. Subtract 4a − 6b + c from the sum of (2a − 6b + 3c ) and (3a − 2b − c )
3. Find the sum of 7 x 3 + 2 x 2 + 3 x + 1 , − 2 x 3 + 3 x 2 − 2 x + 3 , 4 x 2 − 2 x + 2
4. − (3 x − 2 y )(2 x + 3 y )
( )
5. Divide 18 x 3 y 3 − 12 x 2 y 2 + 6 xy by 3 xy

B. Find the indicated product

1. (2 x + 3)(2 x − 3)
2. (4 y − 2z )
2 2

3. 4ab(8ab − 2a + c )
4. [(x + y ) − (a + b )][(x + y ) + (a + b )]
5. (x + 9)(x + 3)

C. Factor the completely.

1. a 2 − 10a + 21
2. 4 g 2 + 20 g + 25
3. 49m 4 − 16
4. 27 x 3 + 8
5. 4 x 3 + 14 x 2 + 6 x

44
Answer to Activity 1
A.
1. 6+ x
2. a −8 B.
1. Nine times a number x minus four.
1 n
3. n + 4 or + 4
3 3 2. Negative three-fourth x minus four.
4. 4 x − 2 3. Three –fifths times the quantity x-
1
5. x−5 squared minus four.
2
6. 13 y + 3 or 3 + 13y 4. Four times the cube of a minus seven.
2
7. (c + 4) 5. Z-squared times the quantity Z minus
5
one-fourth.
8. − 2a(a + 9 )
9. 5 x − 8 C. 1. P+2
10. 5 x + 3 or 3 + 5x 2. 2w
3. 12 − Q
4. G+2
15
5.
y
Answers to Test 1

A.
1. d − 10 4. Q − 2
2. -5 5. (6 x + 4 x )meters or
3. One fourth of n increased by 6 10 x meters
Or n divided by 4 plus 6.
B. 1. 2s + 5 C. 1. 20
2. 100g m 2. 660
r
3. 3. 263
5
4. D+4 4. 402
5. (q + r) - 10
5. -23

45
ANSWERS TO ACTIVITY 2
A = 36
B = 144 N =9 II
C = 125 O=0 1. False, 36
D = 1000 P = 216 2. False, 2 7
E = 10 Q = 243 3. True
R=8 4. False, 1
F = 32
S =7
5. True
G = 16
6. True
H = 49 T = 64
7. True
I = 400 U =1
8. False, 2a 2
J = 100 V = 81 1
W = 27 9. False, 8
K =5 b
L=4 X = 25 10. True
M = 11 Y = 121
Z =3
III.
1.a 13
2 .125 a 6
L AW S O F I N T E G R AL
3.21 y 7 z
E X P O N E N T S 4 . − 3a 2 x 6 y 2
5.abc
6 .9 a 2 y 6
7x3
7.
IV. 1. yes 4 z 10
2. No kK 6
3. Division by zero is not allowed since 8.
25 r 8
this will lead to undefined expression
m
9. 2
7r
10 . − 5 x 16 y 5

11. y 2 + b
12. x10 a
13. 35 x + 2

46
VI
V.
1. A = c14 sq. unit 1. 16a 8b 4
2. A = p5 sq. unit 2. − 108 x12 y 8
3. A = z9 sq. unit 3. − 6 a 4b 6
4. x6 y 2
4. V = x8 unit 3
5. ( x − 2 y )
4
9 3
5. V = a unit
6. x 4 y12
7. 1
b8c 4
8.
a2

VII. 1. No, x 5 ⋅ x 3 = x 8 , by the product of Powers the exponents are added.


a 30 , using the Power of Powers. i.e. (a m ) = a mn
n
2.
3. 3 x ,rather than 3x, because powers of three gives more pay rather than multiples of
3
4. 27.00 pesos

Answers to Test 2
1 3. a -72
1. False , −3
= 23 = 8 b) 5x
2
2. a) 1 4a 6
c)
b) 1,024 b8
c) 128 1
d) 32,768 d)
441
e) 8

Answers to Activity 3
Answers to Test 3

A. 1. a. 3 2. a . like
b. 2/3 b. like
c. 1 c. unlike
d. -25 d. unlike
e. -.28 e. like
1. -3x -3y
B1. 10 x − 7 x
2

2. − 30 y 3 +20 y 2 − 5 y + 15 2. -10c-5d +5e


3. u − 5c 3. –x –y -3
4. − 46t 2 + 18st − 2s − 2 42t 2 4. 34w -7x + 3y
5. - 3x + 87 5. 27c - 29a + 2b + 16ab

47
6. -11h – e -16 C. 1. The value of x + 1increases as x
increases
7. 9x 2. The same results is observed in x+2
8. 5q 3 + 24q 2 + 120q also in x+3
9. − 66a − b − 36 3.for x-1 the value starts at -1 and
increases by 1, while x-2 starts at -2
10. 16 x − 11 y The same change is observed as x - 1

Answers to Test 4
Answers to Activity 4
1. Arrange the terms in ascending 1. 7m 2 n 2 − 5mn − 4
order
2. Add the difference and the 2. 8 x 2 + x + 15
subtrahend to get the minuend
3. 3. 4n 2 + 13n + 12
a. − x + 4x − 7
2

b. 4h 2 − 6hk + 2k 2 4. − 2 − 6a
c. − 5ab − 6a b − 1
2 2 5. 8 x 2 − 27 x − 20
d. − 4x 2 − 2x + 1
4. a. 8x 2 − 2 x + 4
b. 4 x 3 − x 2 + 5x − 9
c. 4a 3 + 2a 2 b − b + b 3
d. 7ax 2 − 2a 2 x + 2 x
e. 7x - 4y
f. 8a + 11b − 10c + 13d
5. S=x-y

Answers to Activity 5 Answers to Test 5

1. a) -b 1. 14y2 – 23y +3
b) -a 2. 30x2 – 37x2y + 55 xy2 – 18y3
c) 2a 3. -128d4 – 8d2m2
d)-13rs 4. a) 6 x 2 + x − 2
2. a) c 2 − 6c − 16 b) 6x 3 + 9 x 2 + x − 2
b) 6a 2 + 5ab − 6b 2 c) − 9 x 3 − 6 x 2
c) 4 x 3 + 25x 2 − 57 x + 27 d) − 15x 3 − 3x 2
d) 20 x 4 − 9 x 3 + 73x 2 − 39 x + 99 5. A = 16y +64
3. a) -5xy
b) -5bc
c) q3
d) -12x2y
4. w = 23 m l= 27m

48
5. P = 10x – 2y
6. a) A= 20x +10
b) length f the rectangle is 15 m
c) Php 9,750.00

Answers to Activity 6 Answers to Test 6

1. 3 – 2a2 1. 4r 3s − 2r 2
2. 3r 4 y 3 − 4ry + 2r 6 y 4 10 y 5
2. 5x 3 − 3x 2 y + 2 xy 2 + y 3 +
4x 2 − 2 y 2
3. 2 x 2 − xy + 2 y 2 r 2y 3 3. 3h 2 + 3h + 1 vegetables
1 4. By division of polynomials
4. − x
4 x 2 − xy + y 2
3
x + y x 3 + ox 2 y + oxy 2 + y
5. 3x – 2x2 - x3 + x2 y
6. 2 - x2 y
− x 2 y − xy 2
7. d 2 + 3d − 4 xy 2 + y 3
- xy 2 + y 3
by checking:
(x + y )(x 2 − xy + y 2 ) = x 3 + y 3
therefore (x+y) is a factor of x 3 + y 3

Answers to Activity 7 Answers to Test 7

A. 1. 6a 2 − 11ab − 35b 2 A. 1. x 4 − 25x 2 + 144


2. 4 x 2 + 13x − 12 2. 49a 4 − b 4
3. x 2 − 36 1 2
3. a + ak + k 2
4
4. 4 x 2 + 4 xy + y 2 4. 16 x 3 − 40ax + 25x − 64 x 2 + 160a
5. 4b 2 + 4c 2 + 4d 2 + 8bc + 4bd + 4cd 16 4
5. d −h2
9
B. 1. 4xy
2. 9x2
3. -2xy
4. 4y2 +6zy + 9z2
5. [(a-b) – (c-d)]
C. a + b 2 + c 2 + 2ab + 2ac + 2 bd
2

49
Answers to Activity 8 Answers to Test 8

A. 1. 4g 2 − s 2 A.1. True
2. m 6 − 9f 4 2. False, x 2 + x − 6
3. 4 v 2 y 2 − y 2 r 2 − 100 v 2 + 25r 2 B. 1. 8xy
4. 81a 2 − 36aq + 4q 2 2. -28x
5. 4a 2 − 4ac + c 2 − 25f 2 C. 1. v 6 + 21u 4 v + 147 u 2 v 2 + 343v 3
6. 9x 4 − 24 x 2 + 16 2. x 10 − 2 x 8 + x 6 − x 4 + 2 x 3 − x 2
16 4 3. 9 x 2 − 42 xy + 49 y 2
7. x − y2
9
8. 64a 2 − 32ab + 4b 2 + 48a − 12b + 9
4. 81p 4 + 144p 2 r 4 + 16r 8 + 216p 3 r 2 + 72p 2 r 4 + 96pr 6
9. p 6 − 15p 4 q 2 + 37 p 2 q 4 − 81q 6
a2 + 2ab+ b2 +c2 +d2 −2 de + e2 +
10.
2ac+ 2bc+ 2ad−2ae−2bd−2be+ 2cd−2ce

Answers to Activity 9 Answers to Test 9

A. 1. 2a 7 b 3 (3 − 167ab) 1. (x -3)
2. 4(6 b − 5)(6 b + 5) 2. 12xy
3. 3c(b − 3d )(b 2 + 3bd + 9d 2 3. (y + 9) ( y + 3)
4. (x+6)(x-4) 4. (b − c)(b 2 + bc + c 2 )(b 6 + b 3 c 3 + c 6 )
5. [2(x − 2 y ) + 3][5(x − 2 y ) + 2]
6. (3k + 5m )2
7. (4ab − 3c)(16a 2 b 2 − 12abc + 4c 2 )
8. 4 x (3a + 5b + 8c )
9. (c − 3)(c + 1)
10. (5x + 2)(x − 3)
11. (2x + b )(a + 3c )
12. (a + 2b )(a + 1)
13. (a − 3b 2 )(5a + 4b)
14. 7abc (4abc + 3ac − 2 )

50
B. Distributive property
C. Area of the square = 16 y 4 − 16 y 3 + y 2

Answers to Unit Test

I. 1. constant II A. 1. 12p 3 q + 18p 2 q + 10pq


2. 4 2. a − 2b + c
3. m 2 + 3mn + 7 n 2 3. 5x 3 + 9 x 2 − x + 6
4. 19x 4. − 6 x 2 − 5xy + 6 y 2
5. 2mn(8m2n + 16mn3+1) 5. 6x 2 y 2 − 4 xy + 2
6. 8 B. 1. 4 x 2 − 9
7. a 2 x 2 + 2 bax + b 2 2. 16 y 4 − 16 y 2 z + 4z 2
8. 8x - 7 3. 32a 2 b 2 − 4a 2 b + 4abc
9. 6x3y5z8 – 2xyz 4. ( x + y) 2 − (a + b ) or
2

8m 2 n x 2 + 2 xy + y 2 − a 2 − 2ab − b 2
10.
m+n
11. (3a − 2)(a + 1) 5. x 2 + 12 x + 27
12. (z – 2) m and (2z – 3) m C. 1. (a − 7 )(a − 3)
13. x 2 = 82 − 25x 2. (2g + 5)(2g + 5)
14. (3a + b) 3. (7m − 4 )(7 m 2 + 4 )
2

15.
M+ N+ O+P 4. (
(3x + 2) 9x 2 − 6x + 4 )
4
5. 2 x (x + 3)(2x + 1)

51
Common Errors / Misconceptions in Unit III

1. Application of the Law of Exponents


Example:
(33 )(34 ) = 312
2 x 5 + 3 x 5 = 5 x10

In this kind of problem it is very common among students to multiply the exponent
whenever they are multiplying expression instead of adding the exponent and copy the common base.
and for the addition of algebraic expression the students usually add the exponent instead of just
copying the common exponent. Thus the right answer to this problem is. 3 7 and for the second
5
problem it should be 5x

2. Special Products or multiplication of binomials


Example: ( x + y ) = x 2 + y 2 here the student usually distribute the exponent to every term but
2

this process is wrong it should be ( x + y ) = x 2 + 2 xy + y 2


2

3. Simplifying polynomials especially on operations of integers removing grouping signs

(3x + 5 x − 5) + (5 x 2 − 7 x + 9 ) = 3 x 2 + 5 x − 5 − 5 x 2 + 7 x − 9
2

This process is wrong since the grouping sign is preceded by a positive sign hence no need of
changing the signs of the values inside the parenthesis. This process is applicable only if the grouping
sign is preceded by a negative sign.

52
1
FIRST DEGREE EQUATIONS AND INEQUALITIES IN
ONE VARIABLE

Planting rice is the usual scenario in the rural areas for the month of
June and July.
T
Mang Mike commissioned the brothers Edgar and Tony to plant
rice in his rice field. Edgar can finish the job in 12 hours while Tony can
do the same job in 15 hours. The number of hours it will take both of them
to complete the whole job can be solved using first degree equations in one

V
variable.

In this unit, you will learn the different concepts involved in first
degree equations and inequalities in one variable. As you go over the
exercises, you will develop the skills in solving first degree equations and
inequalities and apply them in real-life problems.

2
Cash register

Painting the wall of a room

EQUATIONS AND INEQUALITIES

In this lesson, you will be able to


distinguish an equation from an inequality
by the symbol which relates two
expressions.
Mother and daughter
An equation is a mathematical
statement that uses the equality symbol “=”,
Activity 1:
while an inequality uses the symbols, >, <,
≥, ≤ or ≠. A. Tell whether each statement is an
equation or an inequality.
Examples:
1. –5 + 7 = 2
1. 2 + 7 = 9 is an equation 2. 7 + 3 < 15
3. 25 > 12 + 11
2. 9 – 3 = 5 is a false equation and may 4. 5 + 7 = 12
be written as 9 – 3 ≠ 5 5. 13 – 6 > 4
3. 5 – 2 < 6 is an inequality B. Write the symbol =, >, or < in a
to form a true relation.
4. 4 + 6 > 5 is an inequality
1. 8 + 7 14
5. x + 5 ≥ 7 is an inequality 2. 6 + 4 5
3. 4 – 3 7

3
4.12 + 4 22 Examples:
5. 25 – 17 12
1. 2x + 4 = 9 is a first degree equation
Test 1: in one variable. The variable is x and
the exponent is 1.
A. Tell whether each statement is an
equation or an inequality. 2. 5y + 7 = 6y + 5 is a first degree
equation in one variable, the variable is
1. –6 < -5 y and the exponent is 1.
2. 14 + 2 = 16
3. 9 + 7 > 13 3. 5x – 7y = 0 is a first degree equation in
4. –3 + 7 < 6 two variables because there are two
5. –7 + 3 = - 4 variables, x and y with both variables
of degree 1.
B. Write the symbol =,>,or < in a
to form a true relation. 4. x2 + 5x + 6 = 0 is not a first degree
equation in one variable because the
1. –11 –3 -14 highest exponent of x is 2.

2. –7 + 4 3 Activity 2:

3. 15 – 20 -5 A. Which of the following is a first


degree equation in one variable?
4. 18 – 12 6
1. 4a + 3b = 8
5. –26 + 16 12 2. 5x – y = 0
1
Instructions: 3. x + 4 = 12
2
4. 6x + 8 = -2
• After answering the test, check your 5. 3x + 6 = 0
answer with those on the answer Test 2:
key.
• If your score is 3 or higher, you may A. Which of the following is a first
proceed to the next topic; otherwise, degree equation in one variable?
read the lesson once more and do
the Test again. 1. –5x + 3 = 8
x+5
2. =6
2
First Degree Equations in one 3
Variable 3. = 16
x+6
First degree equations in one variable is 4. x(x - 6) = -7
an equation that can be written in the form
ax + b = c, where a and b are constants and 5. a2 + 5a + 8 = 0
a ≠ 0. Remember the letter x represents a
variable of degree 1.

4
Instructions: 1
5. + 7 > 8 is not a first degree
x
• After answering the test, check your inequality since the variable x is in the
answer with those on the answer denominator the exponent is –1.
key.
• If your score is 3 or higher, you may Activity 3:
proceed to the next topic; otherwise,
read the lesson once more and do A. Identify the first-degree
the Test again. inequalities in one variable.
1. a + 3 = -4
First Degree Inequalities in one 2. x–5–4>8
Variable 3. –a–7=0
4. x2 – 6 ≥ 0
It would be easy for you to recognize a 5. 3n – 2 < 0
first-degree inequality. It is similar to a first
degree equation in one variable that has a Test 3:
degree of one and uses one variable and any
of the symbols >, <, and ≥, instead of the A. Identify the first-degree
equal ( = ) sign. inequalities in one variable.
1. x + 7 = 10
Examples:
2. x – 3 ≥ -9
3. x4 – 5 = 1
1. 3x - 5 > 4 is a first degree inequality in
4. a + 3x > -4
one variable. The variable is x and the
5. 9–b<6
exponent of the variable is 1. The
inequality symbol “>“ is read as “is
Instructions:
greater than”.
2
2. 5x + 4 < x + 6 is a first degree • After answering the test, check
3 your answer with those on the
inequality in one variable. The answer key.
inequality symbol “< “ is read as “is less • If your score is 3 or higher, you
than.” may proceed to the next topic;
otherwise, read the lesson once
3. 3a – 2b ≥12 is a first degree inequality more and do the Test again.
but in two variables since there are two
variables (a and b) involved. The
inequality symbol “≥” is read as “is Translation of Verbal Statements to
greater than or equal to”.
Equations and Vice -Versa
4. a2 – 4b ≤ 5 is a first degree inequality
Translating verbal statements into
but in two variables because the
mathematical symbols is an essential skill
exponent of the variable is 2. The
for you to develop in solving word
inequality symbol “≤” is read as “is less
problems. In section you will learn how to
than or equal to”.
identify the key words that would indicate

5
the operation to be used in forming You will notice that in the two
equations and inequalities in one variable. statements the word “is” was used in place
of “equals.” With the inclusion of “is”, the
A single operation in an algebraic statement gives rise to an equation.
expression can be interpreted in several
ways. For examples, With this knowledge you will be able
to translate verbal statements into
The worded expressions: equations.
x plus 5
the sum of x and 5 Examples: Below are a few verbal
x increased by 5 statements and their translations.
5 more than x
can be translated into a single algebraic 1. The difference of 15 and n is 10.
expression x + 5.
15 - n = 10
Variables are used to represent numbers
that are unknown. 2. Thrice p is 21.
3p = 21
Examples:
3. Twice the sum of x and 4 is 20.
Consider the following verbal phrases:
and their corresponding algebraic 2(x + 4) = 20
expressions where x = the number.
4. The quotient of 3x – 9 and 10 is 30.
1. Three times a number
3x 3x − 9
= 30
2. A number decreased by 5 10
x–5
3. Two times the sum of x and 4 5. Three more than a number is 12.
2(x + 4) n + 3 = 12
4. Six less than twice a number
2x - 6 6. Eight less than the product of p and
5. The quotient of a number and 4 4 is 20.
x
4 4p – 8 = 20
7. The sum of three consecutive integers is
The word expressions such as” x plus 15.
2” can be meaningful by transforming the n + (n +1) + ( n + 2) = 15
expressions into a statement such as: where, n is the 1st integer
x plus 3 equals 8 n + 1 is the 2nd integer
or x plus 3 is 8 n + 2 is the 3rd integer

Using algebraic symbols, this is 8. The sum of three consecutive odd


translated as x + 3 = 8. integers is -12.

n + (n + 2) + (n + 4) = -12

6
where, n is the 1st odd integer 1. The sum of a number n and 4 is greater
n +2 is the 2nd odd integer than 6.
n + 4 is the 3rd odd integer
n+4>6
9. The sum of four even consecutive 2. Two more than n is less than 6 minus n.
integers is -300.
n+2<6–n
n + (n + 2) + (n + 4) + (n + 6) = -300

where, n is the 1st even integer Activity 4:


n + 2 is the 2nd even integer
n + 4 is the 3rd even integer A. Translate the following word phrases
n + 6 is the 4th even integer into algebraic symbols.
1. The sum of a number n and 5
Consider 85 which is a two – digit number. 2. Three more than p
Note that 8 is the tens digit and 5 is the 3. Subtract 5 from y
units digit. Hence, an expanded form: 4. The product of y and –3
85 = 80 +5 5. Thrice x
= 8(10) + 5(1)
10. The sum of the digits of two digit B. Translate the equations into verbal
number is 6. statements.
If we let x = unit digit
x
6- x = tens digit 1. =8
Thus, x + ( 6 – x ) = sum 5
2. y–9=2
3. 2x + 4 = 10
4. x + 7 = 12
Translation of Verbal Statements to
5. 7x = 28
Inequalities and Vice -Versa

Any of the relation symbols such as >, Test 4:


<, ≥, or ≤ may be used to replace in
statements indicating inequality. However, A. Translate each verbal statement into an
the presence of the verb “is” should be equation or inequality. Let x = the
noted. The inequality symbol number in each case..

> means “is greater than” 2


1. A number divided by 5 is .
< means “is less than” 5
≥ means “is greater than or equal to” 2. A number decreased by 6 is –9.
≤ means ”is less than or equal to” 3. Eight times a number is 56.
4. The product of x and 7 is 84.
With this knowledge you will be able to 5. Twelve is equal to eight plus four.
translate verbal statements into equations
or inequalities. B. Translate the equations and
inequalities into verbal statements.
Examples: Below are verbal statements and
their translations. 1. x + 18 = 54
2. 2n – 10 > 16

7
3.
3p
= 21
Using a number line.
5
12 + t Equations and inequalities in one
4. < 8
3 variable can be graphed or represented on
5. 3q = 78 a number line. The graph of an open
sentence is the solution set of the open
Instructions: sentence in the number line. The solution
set makes use of a pair of braces { }.
• After answering the test, check your
answer with those on the answer Examples:
key.
• If your score is 3 or higher, you may 1. Graph x + 5 = 11
proceed to the next topic; otherwise, x=6
read the lesson once more and do •
-2 -1 0 1 2 3 4 5 6 7
the Test again.
Notice that the graph of the solution set of
Solution Set of Equations and x + 5 = 11 is the point which corresponds
to the real number 6 on the horizontal
Inequalities number line or {6}.
A sentence which contains a variable 2. Graph 3x < 15
such as 3x – 5 = 22, or any formula is also x<5
an equation. Such an equation is neither
true or false until you choose numbers to
replace the variables. Equations that -2 -1 0 1 2 3 4 5 6 7
contain variables are open sentences. The graph of the solution set is a ray
When you find all the values for the whose endpoint is hollow which means 5
variable that make a sentence true, you have is not included in the solution. The
found the solution set. solution set of the inequality consists of
real numbers less than 5, or {x / x < 5}
For example, when x = 9, 3x – 5 = 22 which is read as “ set of all number x such
is a true sentence. Thus, the solution of 3x that x is less than 5.”
– 5 = 22 is 9 and the solution set is{ 9}.
3. Graph x – 5 > -7
Solution of an equation or inequality is x > -2
a value of a variable that converts an open
sentence into a true statement, where a
solution set of an equation and inequalities -2 -1 0 1 2 3 4 5 6 7
is a set of numbers that makes the statement The graph is a ray whose endpoint is
true. This can be represented on a shaded which means -2 is one of the
horizontal number line, taken from a solution. The solution set of the
replacement set and solved by inspection or inequality consists of real numbers greater
by using the properties of equality. than – 2. In symbols {x/x > -2} which is
read as “ the set of all numbers x such that
x is greater than and equal to -2..

8
The Replacement set The correct replacement for x is – 6.
Thus, the solution of x < -5 is -6 and the
Equations and inequalities containing solution set is { -6 }.
variables are called open sentences. An
open sentence is a pattern for the different Set A is called the replacement set
statements, you obtain by replacing the and the replacements which make the
variable by numbers in the replacement set open sentence true are the solutions or
of the variable. Some of these statements solution set.
may be true. Some may be false.
By Inspection
The statements x + 4 = 6 and x < -5 are
open sentences. The variable in an open A question such as “what number
sentence must be replaced by a number multiplied by 3 will result 36?” will
from a given set of numbers. Verification automatically give you an answer of 12.
can be done to see if the replacement makes The solution of an equation or inequality
the equation or inequality a true statement. by inspection is similar to an on-the-spot
questioning where you can do mental
Examples: computation or guessing.

Find the replacement from Examples:


A = {-6, -2, 2, 6} that would give a true
statement for: 1. 5+x=9
a. x + 4 = 6
b. x < -5 What number added to 5 will give a
sum of 9?
Solution:
The answer is 4.
a) x + 4 = 6
Replace x with –6, -6 + 4 ? 6 False 2. 4x – 6 = 18
Replace x with –2, -2 + 4 ? 6 False
What number multiplied by 4 and
Replace x with 2, 2 + 4 ? 6 True decreased by 6 equals 18?
Replace x with 6, 6 + 4 ? 6 False
The answer is 6.
The correct replacement for x is 2 3. x<5
Thus, the solution of x + 6 = 6 is 2 and the
solution set is { 2 }. What numbers are less than 5?

b) x < -5 Answer: {4, 3, 2, 1, 0…}


Solution: 4. 5x + 2 > 17

Replace x with -6, -6 < -5 True What number multiplied by 5 and


Replace x with -2, -2 < -5 False added to 2 is greater than 17?
Replace x with 2, 2 < -5 False
Replace x with 6, 6 < -5 False The answer is {4, 5, 6…}

9
Activity 5: B. Find the solution set from the given
replacement set {1, 2, 3, 4, 5}
A. Represent the solution in the number
line. 1. x–1=3
2. 2x - 4 = 2
1. 2x = 6 3. x+2>3
1 4. 2x + 4 = 12
2. x=2 5. 8x –9 > 20
3
3. x + 3 = 5
4. x + 7 > 10 C. Find the solution of the following
5. x – 1 < 4 equations by inspection.

B. Find the solution set from the given 1. x–4=2


replacement set {1, 2, 3, 4, 5} 2. 2x = 10
3. x+5<9
1. x–1=3 4. 3x > 12
2. x–5=0 5. 2x – 3 < 7
3. 3x + 2 = 5
4. 2x +5 > 13 Instructions:
5. 4x – 6 <10
• After answering the test, check
C. Find the solution of the following your answer with those on the
equations by inspection. answer key.
• If your score is 3 or higher, you
1. 3x – 6 = 3 may proceed to the next topic;
x otherwise, read the lesson once
2. =6 more and do the Test again.
3
3. x + 8 = 15
4. x + 5 < 9 Properties of Equality
5. 3x > 12
The following properties are necessary
Test 5: for you to find the solution of an equation.

A. Represent the solution in the number A. Reflexive Property of Equality


line
The number 3 = 3 tells us that a
1 number is equal to itself.
1. x=4
2
Thus, for each real number a,
2. 2x = 6
a=a
3. x + 7 > 10
1
4. x<3 B. Symmetric Property of Equality
3
5. x+5>6 The statements,

12 + 5 = 17 or 17 = 12 + 5

10
The result is an equation that is
7 · 2 = 14 or 14 = 7 · 2 equivalent to the original equation.

have the same values even if they are not Thus, for all real numbers a, b, and c,
expressed in either way. if a = b, then a + c = b + c.

Thus, for all real numbers a and b, if a F. Multiplication Property of Equality


= b, then b = a. (MPE)

C. Transitive Property of Equality Multiplying the same number to an


equation will give an equivalent equation.
The sum or product of two real numbers
can be equal to the sum and product of two Example: Given the equation 8 · 2 = 16
other real numbers.
8 · 2 = 16 Given
If 4 + 6 = 10 and 10 = 3 + 7, then
4 + 6 = 3 + 7. (8 · 2) · 4 = 16 · 4 Multiply 4 to
64 = 64 both sides of the
If 4 · 3 = 12 and 12 = 6 · 2, then equation.
4·3=6·2
The result is an equation that is
Thus, for all real numbers a, b, and c, if equivalent to the original equation.
a = b and b = c, then a = c.
Thus, for all real numbers a, b and c,
D. Substitution Property of Equality if a = b, then ac = bc.
If x is a number in x + 3 = 7, and Activity 6:
x = 4, then 4 + 3 = 7.
A. Tell the property illustrated by the
For all real numbers a and b, if a = b, following statements:
then a may be replaced by b.
1. If 8 · 3 = 24 and 24 = 12 · 2.
E. Addition Property of Equality (APE) then 8 · 3 = 12 · 2
Adding the same number to an equation 2. -6 = -6
will give an equivalent equation.
3. If a + 6 = 10, then a = 4
Example:
4. If 10 + 5 = 15, then
Given the equation 4 + 1 = 5 15 = 10 + 5.

5. If 6 + 8 = 14 and 14 = 10 + 4,
4+1=5 Given then 8 + 6 = 10 + 4
4 + 1 + (-1) = 5 + (-1) Add –1 to
4=4 both sides of B. Complete the statements below using
the equation. the properties of equality.

11
1. –10 = ____ Reflexive Property Property

2. If 6 + 7 = 13, Instructions:
then 3 = ____ Symmetric Property
• After answering the test, check
3. (12 · 3) · 2 your answer with those on the
= 36 · ____ Multiplication answer key.
Property • If your score is 3 or higher, you
may proceed to the next topic;
4. 5 + 4 = 9 and 9 = 7 + 2, then otherwise, read the lesson once
5 + 4 = ____ Transitive Property more and do the Test again.

5. 3 + 9 + (-9) Finding the Solution Sets of First


= 12 + ____ Addition Property Degree Equations in One Variable
Test 6:
When you solve equations, find a
A. Name the property illustrated in the particular value for the variable that would
following statements: make both sides of the equations equal.
This value of the variable is referred to as
1. (3 · 5 ) 2 = 15 · 2 the solution of the equation. The set of
2. 7 = 5 + 2, then 5 + 2 = 7 solutions for a given equation is called the
3. (6 + 2) + (-2) = 8 + (-2) solution set. A first degree equation in one
4. 7 = 7 variable has only one solution.
5. If 8 · 5 = 40, then 40 = 8 · 5
The properties of equality will be
B. Complete the statements below by applied in the process of finding the
using the indicated property of equality. solution of first degree equations.

1. 3x = 15, then To find the solution of equation,


1 consider the following examples.
(3)x = _____ Multiplication
3 Examples:
Property
Find the solution set of the equations:
2. If 2· 5 = 10, then 1. x + 7 = 15
10 = _____ Symmetric Property x + 7 + (-7) = 15 + (-7) APE
x + 0 = 8 Additive inverse
3. If x + 10 = 15, property (AIP)
then x = ___ Substitution x = 8 Identity property of
addition (IPA)
4. If 15 + 1 = 16 and
16 = 10 + 6, then ____ Transitive The solution set of the equation
Property x + 7 = 15 is {8}
5. 4 + 3 = 7 and 7 = 10 – 3, then 2. y – 9 = -2
4 + 3 = _________ Transitive y – 9 + 9 = -2 + 9 APE

12
y+0=7 AIP The solution set of the equation
y=7 IPA 2x + 9 = 3 – 4x is {-1}

The solution set of the equation 7. 8(3 – 2x) = 56


y – 9 = -2 is {7}. 24 – 16x = 56 Distributive
Property
3. 2x = 42 24 – 24 – 16x = 56 –24
1 1 -16x = 32 APE
(2x) = (42)
2 2 1 1
- (-16)x = - (32)
2x 42 16 16
= x = -2 MPE
2 2
x = 21 MPE
The solution set of the equation
The solution set of the equation 8(3 – 2x) = 56 is {-2}
2x = 42 is {21}.
Activity 7:
x
4. = 12 A. Find the solution sets of the following
3
equations.
x
3( ) = 12(3)
3 1. a + 3 = 19
x = 36 MPE 2. x – 4 = 10
3. –20 = b – 28
x 4. x + 1 = 12
The solution set of the equation = 12 is
3 5. y + 15 = 28
{36}.
B. Solve for the value of x.
5. 5y – 4 = 21
5y – 4 + 4 = 21 + 4 1. 3x + 7 = 22
5y = 25 APE 1
2. x + 16 = 4
1 1 2
(5y) = (25)
5 5 3. 4x – 13 = 11
y=5 MPE 4. 5x + 30 = 40
2
The solution set of the equation 5. x – 12 = -18
3
5y – 4 = 21 is {5}
C. Find the solution set of each equation
6. 2x + 9 = 3 – 4x below. State the properties used in
2x + 9 – 9 = 3 – 4x – 9 each step.
2x = -4x – 6 APE
2x + 4x = -4x – 6 + 4x 1. 3(13y – 2) = 72
6x = -6 APE 2. 5x + 7 = 22 + 2x
1 1 3. 7x – 5 = 3x + 4
(6x) = (-6)
6 6 4. 5(x+1) + 1 = 1 – (x + 1)
x = -1 MPE 5. 3(5x + 4) = 12 + 3x

13
Test 7: The different properties of inequality
that will be used in the process are as
A. Find the solution set of the following follows.
equations. 1. Trichotomy property

1. 6x = 42 For any number x and y exactly one


1 of the following is true:
2. y = 3
6
4 x < y, x = y, or y < x
3. x = 16
5
2. Transitive Property
4. 2p = 16
2
5. q = 8 For any number x and y
3 If x < y and y < z, then x < z
B. Find the solution set of the following: 3. Addition Property of
State the properties used in each step. Inequality (API)
1. 4y + 5 = 37 – 4y For any number a, b and c
2. 5x = 2(30 – x) + 10
3. 2(2y + 5) = 3(5-3y) + 8 a) if a > b, then a + c > b + c;
2 x b) if a < b, then a + c < b + c.
4. x = 20 +
3 2
5. 3x – 1 = x + 5 4. Multiplication Property of
Instructions: Inequality (MPI)

• After answering the test, check your For any real numbers a, b and c
answer with those on the answer a) if c > 0 and a < b, then ac < bc;
key. b) if c > 0 and a > b, then ac > bc;
c) if c < 0 and a < b, then ac > bc;
• If your score is 3 or higher, you may
d) if c < 0 and a > b, then ac < bc.
proceed to the next topic; otherwise,
read the lesson once more and do
Examples:
the Test again.
Find the solution set of the inequalities
Finding the Solution Set of First 1. x + 12 > 25
Degree Inequalities in One Variable x + 12 – 12 > 25 – 12 API
x > 13 AIP
The solution set of first degree
inequalities in one variable can be found by The solution of x + 12 > 15 consists of
using similar procedure as in solving any number greater that 13.
equations. Unlike in first degree equations
in one variable, a first degree inequality The solution set can be written in the
may have more than one solution. form {x/x >13}. This is read as “the set of
all numbers x such that x is greater than
13.”

14
3
Check by substituting numbers that are 4. x<-6
4
greater than 13 into the inequality. 5. 3(4y + 7) < 21

2. –3y > 18 Test 8:


1 1
- (-3y) < - (18) MPI A. Determine the solution set of the
3 3 following inequalities.
y < -6 MIP and
MPE 1. x – 15 ≤ 19

The solution of –3y > 18 consists of 2. 9y ≤ - 27


any number less than – 6.
x 2
The solution set can be written as 3. >
2 3
{y/y < -6}. This is read as “the set of all
number y such that y is less than {-6}
4. 12x – 40 ≥ 11x – 50
By applying multiplication property of
5. 7y + 8 < 15 + 4y
inequality states that “ if c < 0 and
a > b, then ac < bc” was applied in this
Instructions:
problem.
• After answering the test, check
This can be checked by assigning values
your answer with those on the
for y that is less than –6.
answer key.
• If your score is 3 or higher, you
3. 3x + 5 ≥ 12 + 2x
may proceed to the next topic;
3x + 5 – 2x ≥ 12 + 2x – 2x API otherwise, read the lesson once
x + 5 ≥ 12 more and do the Test again.
x + 5 – 5 ≥ 12 – 5 API
x≥7

The solution set which is written as Solving Problems Involving First


{x/x > 7} is read as “the set of all numbers
x such that x is greater than or equal to 7.”
Degree Equations in One Variable
First degree equations and inequalities
can be used in solving word problems.
Activity 8: Consider the following procedure in
solving word problems.
A. Determine the solution set of each of the
following inequalities: 1. Represent the unknown with a
variable.
1. x – 3 > 12 2. Translate the conditions stated in the
2. y + 10 ≤ - 9 problem into an equation.
x 3. Solve the equation.
3. ≥ 10
5

15
4. Check the solution with the problem Check: 40 is 5 more than 35
and not with the equation.
5. Answer the problem. The sum of 40 and 35 is 75. Therefore,
the two numbers are 35 and 40.
Use these steps to solve problems on
number relations, motion, age, mixture,
work, coins and business. 2. The sum of the digits of a two- digit
number is 10. If the number is divided by
Examples: the units digit, the quotient is 3 and the
remainder is 4. Find the number.
A. Number Relations
Solution:
1. A number is five more than another
number. If the sum of the two numbers Let x = the units digit
is 75, find the numbers. 10 – x = tens digit
x + 10 (10 – x) = the number
100 − 9 x 4
=3+
x x
100 − 9 x 4
x( =3+ )
x x

100 – 9x = 3x + 4
-9x – 3x = 4 – 100
-12x = -96
x = 8 the units digit
10 – x = 10 – 8 = 2 the tens digit
2(10) + 8 = 28 the number

Representation: Check: The sum of 8 and 2 is 10


Let x = the 1st number 28
x + 5 = the 2nd number = 3, remainder is 4
8
x + (x + 5) = the sum of two numbers Therefore, the number is 28.
Equation: 3. The sum of three consecutive integers
x + (x + 5) = 75 is 15. Find the integers?

Solution: Solution:

2x + 5 = 75 Let x = the 1st integer


2x = 75 – 5 x + 1 = 2nd integer
2x = 70 x + 2 = 3rd integer
x = 35 one number
x + 5 = 35 + 5 x + ( x + 1) + ( x + 2 = 15
= 40 the other number 3x + 3 = 15

16
3x = 15 – 3
3x = 12 4x = 300-12
x=4 4x = 288
x = 72
Thus, x = 4 is the 1st integer
x+1=4+1 Thus, x = 72 is the 1st even integer
= 5 is the 2nd integer
x+2=4+2 x + 2 = 72 + 2
= 6 is the 3rd integer = 74 is the 2nd even integer
Therefore, 4 + 5 + 6 are the three integers.
x + 4 = 72 + 4
4. Find the three consecutive odd integers = 76 is the 3rd even integer
whose sum is -12.
x + 6 = 72 + 6
Solution: = 78 is the 4th even
integer
Let x = 1st odd integer
x + 2 = 2nd odd integer Therefore, 72 + 74 + 76 +78 = 300
x + 4 = 3rd odd integer
B. Age Problem
x + ( x + 2) + (x + 4 ) = -21
3x + 6 = -21 To solve age problem, you often
3x = -21 – 6 have to represent a person’s present age,
3x = -27 person’s age a number of years ago, and
x = -9 the person’s age a number of years from
st
Thus, x = -9 is the 1 odd integer now.
x + 2 = -9 + 2
= - 7 is the 2nd odd integer
x + 4 = -9 + 4 Age Age 4 yrs Age 7 yrs.
= - 5 is the 3rd odd integer now ago from now
Mae 15 15 – 4, or 11 15 + 7 or 22
Therefore, -9 , -7 & -5 are the three
Integers. Mia x x-4 x +7

5. The sum of four even consecutive


integers is 300. Find the integers. Thus to represent a past age, subtract from
the present age. To represent a future age,
Solution: add to the present age.
Example:
Let x = 1st even integers
x + 2 = 2nd even integer Mrs. Inac is now three times as old as
x + 4 = 3rd even integer her daughter, Sarah. In 14 years, Mrs.
x + 6 = 4th even integer Inac’s age will be twice Sarah’s age then.
How old is each?
x + ( x + 2) + ( x + 4 ) + ( x + 6 ) = 300
4x + 12 = 300

17
To save on budget, a housewife
mixed first class rice which she bought
forP30.00 per kilo with third class rice
which sells for P20.00 per kilo. How
many kilos of third class rice must she
add to 8 kilos of first class rice to
produce a mixture that would cost to
P22.00 per kilo?

Representation:
Age now Age 14 yrs.
from now
Sarah x x + 14
Mrs. Inac 3x 3x + 14

To write an equation:
Solution:
Mrs. Inac’s age will twice Sarah’s
in 14 yrs be in 14 yrs. P20 + P30 = P22

x 8 x+8
3x + 14 = 2(x + 14)
20x + 30(8) = 22(x + 8)
Solution:
20x + 240 = 22x + 176
240 – 176 = 22x – 20x
3x + 14 = 2(x + 14)
64 = 2x
3x + 14 = 2x + 18
32 = x
(-2x) + 3x + 14 = (-2x) + 2x + 18
x + 14 = 18
Therefore, 32 kg of third class rice
x + 14 + (- 14) = 18 + ( -14)
must be added to 8 kg of first class
x = 14
rice.
3x = 42
D. Motion
Check:
Does 42 = 3 (14) ? Yes
Two buses start from the same place at
42 + 14 = 2 (14 + 14) Yes
the same time and go in opposite
Therefore, Sarah is now 14yrs old and Mrs. directions, one traveling 12 km/hr faster
Inac is 42 yrs. old. than the other. Find the rate of each if they
are 324 km apart at the end of 3 hrs.
C. Mixture

18
D = 144 km + 180 km
D = 324 km
Therefore, the rates of the buses are
48km/hr and 60/hr.
E. Work
If Edgar can finish a piece of work
in 12 hrs. His brother Tony can do the
same job in 15 hrs., how long will it
take both of them to complete the
work?

Since the two buses are 324 km apart in 3


hrs, then the rate can be solved using the
distance formula d = rt, where
d = distance
r = rate
t = time
Distance traveled by one bus + distance
traveled by the other bus = 324 km.

(Rate · time) + (Rate · time) = 324


Bus1 + Bus 2 Let x = number of hrs. to complete the
Let x = rate of the one bus work
x + 12 = rate of the other bus x
= the part of work Edgar can
12
3x + 3(x + 12) = 324
finish in 1 hour.
6x + 36 = 324
x
6x = 288 = the part of work Tony can
x = 48 km/hr rate of one 15
bus finish in 1 hour.

x + 12 = 48 + 12 The work that can be finished by both


= 60 km/hr rate of the other in 1 hour is given by the equation.
bus
x x
+ =1
Check: 12 15

The distance traveled by the two buses 4x + 5x


must be 324 km. =1
60
9x = 60
Distance (D) of two buses from each other 60
x=
= (48km/hr)(3hrs) + (60 km/hrs)(3hrs) 9

19
2 annum where she invested the rest of her
x=6 savings. If she realized a total yearly
3
2 income of 19% on her two investments,
The brothers can finish the job in 6 hrs. how much did she invest at 20%? How
3
much was her savings?
F. Coins
Let x = Mrs. Ycoy’s savings
A mini store has 190 coins in a cash 500,000 = investment at 16%
register consisting of P5 and 25¢ coins. x – 500,000 = investment at 20%
If all of these coins amounted to
P285.00, how much of each kind did the .16(500,000) + .2(x – 500,000) = .19x
cash register has? 80,000 + .2x – 100,000 = .19x
.2x - .19x = 20000
x = 2,000,000
Mrs. Ycoy’s savings is P2,000,000
x – 500,000 = 1,500,000 the
investment at 20 %
Check:
.16(500,000)+.2(x – 500,000)
=.19% income from two
investments.
Let x = number of P5 coins
190 – x = number of 25¢ coins 380,000 = .19(2,000,000)
The total amount: Therefore, the amount of Mrs. Ycoy’s
saving is P2,000,000 and her investment
5x + .25(190 – x) = 285 at 20% is P1,500,000.
5x + 47.5 - .25x = 285
4.75x = 237.5 Activity 9:
x = 50
A. Solve the following problems.
Check: x = 50 the number of P5
coins 1. The sum of the digits of a two-digit
number is 12. If the number is divided
190 – x = 140 the number of by the unit digit, the quotient is eight
25 ¢ coins and the remainder is 1. Find the
5(50) + .25(140) = 285 number.
Therefore, the number of P5 coins is 50 2. Mr. Fernando is four times as old as
and the number of.25¢ coins is 140. his son. In 10 yrs, he will be 2 yrs
more than twice the age of his son.
G. Business Find their present ages.

Mrs. Ycoy invested P500,000 of her 3. Wendell’s scores on four tests in


savings at 16% per annum. She found Algebra were 87, 92, 88, and 86. What
another investment opportunity at 20% per score does Wendell need to get the next

20
test in order to have an average score Test 9:
greater than 90?
Solve the following problems

4. To save on budget, Mang Tommy the 1. Find all sets of 3 consecutive positive
baker mixed sugar which he bought for odd integers such that the sum of the
P20.50 per kilogram with third class sugar integers in the set is greater than 4
which sells P16.00 per kilogram. How times the least integer in the set. (Ex.of
many kilogram of third class must he add consecutive odd nos. 1 ,3,5,7…)
to 10 kilograms of first class sugar to .
produce a mixture that would cost to
P18.50 per kilogram?

2. Aling Rona and Mang Ryan who were


engaged in vending vegetables earned
5. Two tricycle start from the same place
P520.00. Aling Rona earned 3 times
at the same time and go in opposite
as much as Mang Ryan. How much
directions, one traveling 10 km/hr
did each earn?
faster than the other. Find the rate of
each if they are 480 km apart at the end
of 4 hours.

21
3 Mari wants to fence a garden plot using problems involving linear inequalities
a 44 m wire. If the length of the lot is 2m sometimes give several answers.
more than the width, find the dimensions
of the garden plot. Examples:
4. Jake can paint the wall of the room in 4 1. Gina is 7 years older than Anna, and
hrs., while Jun can do the job in 5 hrs.. the sum of their ages is less than 23.
How many days will they finish the job Calculate their possible ages.
if they work together?
Let x = age of Anna
x + 7 = age of Gina

x + x + 7 < 23
2x < 23 – 7
2x < 16
x < 8 age of Anna

Since the age of Anna is less than 8


yrs, then their possible ages are:

{7,14}, {6, 13}, {5, 12} … {1, 8}

2. Find all possible integers such that 4


less than 3 times a given integer is
greater than 64. Which is the least
integer?
5.. Dante is 2 years older than his brother
Dino and the sum of their ages is less Let x = the least integer
than 16. Find their possible ages.
3x – 4 > 65
3x > 65 + 4
Instructions: 3x > 69
x > 23
• After answering the test, check your
answer with those on the answer The least integer is 24 and all the
key. positive integers are 25, 26….
• If your score is 3 or higher, you may
proceed to the next topic; otherwise, Summary:
read the lesson once more and do
the Test again.
* An equation is a mathematical
statement that uses the equality symbol
Solving Problems Involving “=”, while an inequality uses the
Inequalities in One Variable symbols >, <, ≥, ≤ or ≠.

In this lesson, you will find that there * A first degree equation in one variable
are some problems which would not result is an equation that can be written in the
in only one answer. The solution of

22
form ax + b = 0, where a and b are a. if a > b, then a + c > b + c
constants and a ≠ 0. The letter x is the b. if a < b, then a + c < b + c
variable and its exponent is 1.
2. Multiplication Property of
* The solution set of the equations and Inequality
inequalities in one variable can be
represented on a horizontal number line, For any number a, b, and c,
taken from a replacement set and solved
by inspection or by using properties of a. if c > 0 and a < b, then ac < bc
equalities. The solution set makes use of b. if c > 0 and a > b, then ac > bc
a pair of braces {}. c. if c < 0 and a < b, then ac > bc
d. if c < 0 and a > b, then ac < bc
* Properties of Equality
* Procedure in Solving Problems
a. Reflexive Property of Equality
a. Represent the unknown with a
For each real number a, a = a. variable.
b. Write an Equation.
b. Transitive Property of Equality c. Solve the Equation.
For all real numbers a, b and c d. Check the solution with the problem
if a = b and b = c, then a = c. and not with the Equation.
e. Answer the problem
c. Symmetric Property of Equality
CHAPTER TEST
For all real number a and b, if a = b,
then b = a. A. Answer the following:
d. Substitution Property of Equality 1. Which of the following is a first degree
For all real number a, and b, if equation?
a = b, then a may be replaced by b.
e. Addition Property of Equality a. x2 – 3 = 1 c. x – 6 < 8
b. x + 4 = 7 d. x – 3
For real numbers a, b and c, if a = b,
then a + c = b + c. 2. Which of the following is a first degree
inequality?
f. Multiplication Property of
Equality a. 2x – 3 < 9 c. x2 + 6 = 15
b. x – 5 = 4 d. 6x2 – 3
For all real numbers a, b and c,
if a = b, then ac = bc. 3. What expression represents the
phrase “two less than four times a
* Properties of Inequality number n”?

1. Addition Property of Inequality a. 2 – 4n c. 2n - 4


b. 4 – 2n d. 4n - 2
For any numbers a, b, and c,
4 What represents the statement

23
“Twice the sum of a and 4 is 20?”
1. A number is 7 more than the 3 times
a. 2(a + 4) = 20 c. a + 4 = 20 another number. If the sum of the two
b. 2a + 4 = 20 d. 2a(4) = 20 numbers is 23, find the numbers.

5. Write the correct symbol to form a 2. In a math class there are three tests.
true relation of 12 – 3 15 – 4. A student must get a total score of 270
for a grade of A. If a student got 91
a. > c. < and 86 in the first two tests what score
b. > d. < must bon last test to get a grade of A?

6.What is x in the equation 3x = 15? 3. Margie is 3 times older then Lilet. In


15 years, the sum of their ages is 38
a.. 5 c. 11 years. Find their present ages.
b. -5 d. 4
4. Two men jog in opposite directions
B. Tell the properties used in starting from the same place at the same
each of the following . time. The average rate of the westbound
men A is 2 km/h more than the rate of
7. If 2 + 4 = 6, then 6 = 2 + 4. the eastbound men B. If they are 15 km
1 apart after 1.5 h, what is the rate of each
8. 3( ) = 1 men?
3
9. (4 + 2) + (-2) = 6 + (-2)
5.The Parents Teachers Association of St.
10. If 8 ·3 = 24 and 24 = 12 · 2, then Andrew’s School held a benefit show. If
8 · 3 = 12 · 2. the number of P200.00 tickets is 5 less
than twice the number of P150.00 tickets,
C. Find the solution set of the following: find the number of tickets sold at each
price if the total sales is P120,000.00.
1. x + 8 = -3
2. 3x – 6 = 12
x
3. =5
4
x
4. + 7 = 10
2
5. 5(x – 2) = 3x + 12
6. x – 3 > 10
7. 2x + 7 < 13
x
8. > 3
5
x
9. -6<4
3
10. 4x – 6 < 7x – 12

D. Solve the problems:

24
2. not a 1st degree equation in one variable
Answer key 3. 1st degree equation in one variable
4. 1st degree equation in one variable
Activity 1 5. 1st degree equation in one variable

A Tell whether each statement is an Test 2


equation or inequality A. Which of the following is a 1st degree
equation in one variable?
1. –5 + 7 = 2 is an equation
2. 7 + 3 < 15 is an inequality 1. a 1st degree equation in one variable
3. 25 > 12 + 11 is an inequality 2. a 1st degree equation in one variable
4. 5 + 7 = 12 is an equation 3. not 1st degree equation in one variable
5. 13 – 6 > 4 is an inequality 4. not 1st degree equation in one variable
5. not 1st degree equation in one variable
B. Write the correct relation symbol
for each Activity 3:
1. >
2. > A. Identify which of the following are 1st
3. < degree inequalities in one variable.
4. <
5. < 1. not 1st degree inequalities in one
variable
Test 1: 2. a 1st degree inequalities in one variable
3. not 1st degree inequalities in one
A. Tell whether each statement is an variable
equation or an inequality 4. not 1st degree inequalities in one
variable
1. -6 < -5 is an inequality 5. a 1st degree inequalities in one variable
2. 14 + 2 = 16 is an equation
3. 9 + 7 > 13 is an inequality Test 3:
4. –3 + 7 < 6 is an inequality
5. –7 + 3 = - 4 is an equation A. Identify which of the following are 1st
degree inequalities in one variable
B. Write the correct relation symbol for 1. not 1st degree inequalities
each 2. a 1st degree inequalities
1. = 3. not 1st degree inequalities
2. < 4. not 1st degree inequalities
3. = 5. a 1st degree inequalities
4. =
5. < Activity 4:

Activity 2 A. Translate the following word phrases


A. Which of the following is a first into algebraic symbols
degree equation in one variable
1. n + 5
1. not a 1st degree equation in one variable 2. p + 3
3. y – 5

25
4. –3y Three times q equals 78.
5. 3x
Activity 5
B. Translate the equations and inequalities
into verbal statements. A. Represent the solution in the number
line.
1. A number divided 5 is 8
2. The difference of y and 9 is 2 1. 2x = 6
3. Four more than the product of x and 2 •
-2 -1 0 1 2 3 4 5 6 7
is 10
4. The sum of x and 7 is 12 1
2. x=2
5. The product of 7 and x is 28 3

Test 4: -2 -1 0 1 2 3 4 5 6 7

A. Translate the verbal statements into 3. x + 3 = 5


equation or inequalities •
-2 -1 0 1 2 3 4 5 6 7
n 2
1. =
5 5 4. x + 7 > 10
2. n – 6 = -9
3. 8n = 56
4. 7x = 84
5. 12 = 8 + 4 -2 -1 0 1 2 3 4 5 6 7
5. x – 1 < 4
B. Translate the equation and inequalities
into verbal statement.
01 2 3 4 5 6 7 8 9
1. Eighteen more than some number x B. Find the solution set from the given
is 54. replacement set {1,2,3,4,5}
Eighteen added to x is 54.
The sum of x and eighteen equals 54. 1. x – 1 = 3
2. Ten less than twice a number is replacement for x is 4
greater than or equal to 16.
Twice the number minus 10 is greater 2. x – 5 = 0
than or equal to 16. replacement for x is 5
The difference of twice a number and 10
is greater than or equal to 16. 3. 3x + 2 = 5
3. The quotient of 3 times an unknown replacement for x is 1
number, p, and 5 is 21.
The quotient of three times p and 5 is 4. 2x + 5 > 13
21. replacement for x are { 4, 5…}
4. Eight is greater than or equal to the
sum of 12 and t is divided by 3. 5. 4x – 6 < 10
The sum of 12 and t divided by 3 is less replacement for x are { 3,2,1 …}.
than or equal to 8.
5. The product of 3 and q is 78.

26
C. Find the solution of the following B. Find the solution set from the given
equation by inspection replacement set {1, 2, 3, 4, 5}

1. 3x – 6 = 3 1. x – 1 = 3
x=3 replacement for x is 4
2. 2x – 4 = 2
x replacement for x = 3
2. =6 3. x + 2 > 3
3
x = 18 replacement for x are {2,3,.4,5}
4. 2x + 4 = 12
3. x + 8 = 15 replacement for x is 5.
x=7 5. 8x – 4 > 20
replacement for x are {4,5 }
4. x + 5 < 9
x = {3,2,1,0…} C. Find the solution of the following
equation by inspection
5. 3x > 12 1. x – 4 = 2
x = {5, 6…} x=6

Test 5: 2. 2x = 10
x=5
A. Represent the solution in the number
line. 3. x + 5 < 9
x<4
1
1. x=4 4. 2x > 12
2
x = {7,8…}
x=8

-2 -1 0 1 2 3 4 5 6 7 8 5. 2x – 3 < 7
x = {4, 3…}
2. 2x = 6
x=3
Activity 6:

-2 -1 0 1 2 3 4 5 6 7
A. Tell the property illustrated by the
3. x + 7 > 10
following statements.
x>3
1. Transitive property of equality
-2 -1 0 1 2 3 4 5 6 7 2. Reflexive property of equality
3. Substitution property of equality
1
4. x<3 4. Symmetric property of equality
3 5. Transitive property of equality
x<9
B. Complete the statements below using
0 1 2 3 4 5 6 7 8 9
the properties of equality
5. x + 5 > 6
x>1 1. -10

-2 -1 0 1 2 3 4 5 6 7

27
2. 6 + 7 x=5
3. 2
4. 5 + 4 = 7 + 2 1
5. (–9) 2. x + 16 = 4
2
1
Test 6 x + 16 – 16 = 4 – 16
2
A.
1
1. Multiplication property of equality (2) x = -12(2)
2. Symmetric property of equality 2
3. Addition property of equality x = -24
4. Reflexive property of equality
5. Symmetric property 3. 4x – 13 = 11
4x – 13 + 13 = 11 + 13
B. Complete the statement below using the 4x = 24
properties of equality x=6

1 4. 5x + 30 = 40
1. 15( )
3 5x + 30 – 30 = 40 – 30
2. 2 · 5 5x = 10
3. 5 x=2
4. 15 + 1 = 10 + 6
5. 10 - 3 2
5. x – 12 = -18
3
Activity 7: 2
x – 12 + 12 = -18 + 12
3
A. Find the solution set of the following: 2
(3) x = -6(3)
1. a + 3 = 19 3
solution set is {16} 2 x = -18
x =-9
2. x – 4 = 10
solution set is {14} C. Solve the solution set of each equation
below. State the properties used in each
3. –20 = b – 28 step. .
solution set is {8}
1. 3(13y - 2) = 72
4. x + 1 = 12 39y – 6 = 72 Distributive
solution set is {11} property
39y – 6 + 6 = 72 + 6
5. y + 15 = 28 39y = 78 APE
solution set is {13} 1 1
(39y) = (78) MPE
39 39
B. Solve for the value of x y=2

1. 3x + 7 = 22 2. 5x + 7 = 22 + 2x
3x + 7 – 7 = 22 – 7 5x + 7 – 7 = 22 +2x – 7 APE
3x = 15 5x = 15 + 2x

28
5x – 2x = 15 + 2x – 2x APE solution set is {18}
3x = 15
1 1 4
(3x) = (15) MPE 3. x = 16
3 3 5
x=5 solution set is {20}

3. 7x – 5 = 3x + 4 4. 2p = 16
7x – 5 + 5 = 3x + 4 + 5 APE solution set is {8}
7x = 3x + 9
7x – 3x = 3x – 3x + 9 2
4x = 9 5. q=8
3
1 1 solution set is {12}
(4x) = (9) MPE
4 4
9 B. Solve the solution set of the
x= following: State the properties involve.
4

4. 5(x + 1) + 1 = 1 – (x + 1) 1. 4y + 5 = 37 – 4y
5x + 5 + 1 = 1 –x - 1 DPE 4y + 5 – 5 = 37 – 5 – 4y APE
5x + 6 = -x 4y = 32 – 4y
5x + 6 – 6 = -x – 6 APE 4y + 4y = 32 – 4y + 4y APE
5x + x = - 6 8y = 32
6x = -6 1 1
(8y) = (32) MPE
1 1 8 8
(6x) = (-6) MPE y=4
6 6
x = -1
2. 5x = 2(30 – x) + 10
5. 3(5x + 4) = 12 + 3x 5x = 60 – 2x + 10 DPE
15x + 12 = 12 + 3x DPE 5x + 2x = 60 – 2x + 2x + 10 APE
15x + 12 – 12 = 12 + 3x – 12 APE 7x = 70
15x = 0 + 3x 1 1
(7x) = (70) MPE
15x – 3x = 0 + 3x – 3x APE 7 7
1 1 x = 10
(12x) = (0) MPE
12 12
x=0 3. 2(2y + 5) = 3(5 – 3y) + 8
4y + 10 = 15 – 9y + 8 DPE
Test 7: 4y + 10–10 = 15–10–9y + 8 APE
4y = 13 – 9y
A. Find the solution set of the following 4y + 9y = 13 – 9y + 9y APE
Equations 13y = 13
1 1
(13y) = (13) M.E
1. 6x = 42 13 13
solution set is {7} y=1

1 2 x
2. y=3 4. x = 20 +
6 3 2

29
2 x x x 4
x - = 20 + - APE solution set {x/x > }
3 2 2 2 3
4 x − 3x
= 20
6 4. 12x – 40 ≥ 11x – 50
x 10
6( ) = (20) 6 MPE solution set {x/x ≤ }
6 9
x = 120
5. 7y + 8 < 15 + 4y
5. 3x – 1 = x + 5 7
solution set {y/y < }
3x – 1 + 1 = x + 5 + 1 APE 3
3x = x + 6
3x – x = x – x + 6 APE Activity 9:
2x = 6
x=3 Solve the following problems:

Activity 8: 1. x = is the unit digit


12 – x = tens digit
A. Determine the solution set of the x + 10(12 – x) = number
following inequality 1
x + 120 – 10x = 8 +
x
1. x – 3 > 12 120 − 9 x 1
solution set is {x/x > 15} x[ =8+ ]
x x
120 – 9x = 8x + 1
2. y + 10 ≤ -9
-9x – 8x = -120 + 1
solution set is {y/y ≤ - 19} -17x = -119
x = 7 unit digit
x
3. ≥ 10
5 12 – x = 12 – 7
solution set is {x/x ≥ 50}
3 5 = tens digit
4. x < -6
4
solution set is {x/x < -8} 5(10) + 7 = 57 the no.

5. 3(4y + 7) < 21 Check:


solution set is {y/y < 0} Sum of 7 and 5 is 12
57
= 8 and the remainder is 1
Test 8 : 7
1. x – 15 ≤ 19
solution set {x/x ≤ 34} 2. let x = the son’s age
4x = age of Mr. Fernando
2. 9y ≤ -27 4x + 10 = 2(x + 10) + 2
solution set {y/y ≤ -3} 4x + 10 = 2x + 20 + 2
4x – 2x = 22 – 10
x 2 2x = 12
3. > x = 6 age of the son
2 3

30
4(6) = 24 age of Mr. Fernando x>6

3. Let x = Wendell’s score on the next test set of integers are:


( 1,3,5), (3, 5, 7 )( 5, 7, 9) …
87 + 92 + 66 + 86 + x > 90
5 2. 520 – x = amount earned by Mang
5( 87 + 92 + + 88 + 86 + x ) > 90(5) Ryan
5 3(520 – x) = amount earned by Aling
87 + 92 +88 + 86 + x > 450 Rhona
353 + x > 450
x > 97 Equation:
Therefore wendell’s next score must be 520 – x + 3(520 – x) = 520
greater than 97 as 98, 99 or 100 4x = -1560
x = 390
4.
20.5 16.00 18.50 520 - 390 = 130 amount
10 x x + 10 earned by Mang
Ryan
(20.5)10 + 16x = 18.5(x + 10)
205 + 16x = 18.5x + 185 3(130) = 390 amount earned
205 – 185 = 18.5x – 16x by Aling Rhona
20 = 2.5x
8=x 3. Let x = width
x = 8 kg of 3rd class sugar must be x + 2 = length
added to 10 kg of 1st class sugar 44 m = Perimeter

5. let x = rate of one tricycle P = 2l + 2 w


x + 10 = rate of the other tricycle 44 = 2(x + 2) + 2( x)
d = rt formula for distance 44 = 2x + 4 + 2x
rate · time + rate · time = 480km 40 = 4x
x(4) + 4(x + 10) = 480 10 = x width of the garden plot
4x + 4x + 40 = 480 x + 2 = 10 + 2 = 12 length of the
8x = 480 – 40 garden plot
x = 55km/hr rate
one tricycle 4. let x = number of days to
complete the job
x + 10 = 55 + 10 x
= part of job Jake can finish
= 65 km/hr rate of the 4
other tricycle in 1 day
Test 9:
x
= part of job Jun can finish
1. Let x = least integer in the set 5
x + 2 and x + 4 are the two other in 1 day
integers.
Equation:
Equation: x +( x + 2 )+( x + 4) > 4x
3x + 6 > 4x

31
x x x
+ = 1 4. + 7 = 10
4 5 2
5x + 4 x solution set is {6}
20 ( = 1)
20
5x + 4x = 20 5. 5(x - 2) = 3x + 12
9x = 20 solution set is {11}
2
x = 2 days 6. x – 3 > 10
9
solution set is {x/x > 13}
5. Let x = Dino’s age
7. 2x + 7 < 13
x + 2 = Dante’s age
solution set is {x/x < 3}
Equation:
x + x + 2 < 16
x
2x < 14 8. >3
x < 7 5
since x < 7, Dino’s ages can be 6, 5, 4 .. solution set is {x/x > 15}
Dante’s age can be : 8, 7, 6 ..
x
9. -6<4
Chapter Test: 3
solution set is {x/x < 30}
A.
1. a 10. 4x – 6 < 7x – 12
2. a solution set is {x/x > 2}
3. d
4. a C. Solve the problems
5. c
6. a 1. let x = 1st number
7. Symmetric property of equality 3x + 7 = 2nd number
8. Multiplicative inverse property x + 3x + 7 = sum of the nos.
9. Addition property of equality x + 3x + 7 = 23
10. Transitive property of equality 4x + 7 = 23
4x = 16
B. Find the solution set: x = 4 1st number.
3(4) + 7 = 19 2nd number.
1. x + 8 = -3
solution set is {-11} 2. Let x = score to get in the last test
91, 86 are the first two sores
2. 3x – 6 = 12
solution set is {6} Equation:
91 + 86 + x > 270
x 177 + x > 270
3. =5 x > 270 – 177
4
solution set is 20 x > 93

since x > 93, scores can be 93,


94,95…

32
3. Let x = age of Delia
3x = age of Lina

Equation:
x + 3x = 48
4x = 48
x = 12 age of Delia
3x = 3( 12) = 36 age of Lina

4. let x = rate (km/h) of the eastbound


Jogger
x + 2 = rate (km/h) of the westbound
Jogger
1.5 h = time

Rate ⋅ Time + Rate ⋅ Time = Distance

x(1.5) + x + 2(1.5) = 15
x(15) + x + 2 (15) = 150
5x + 15x + 30 = 150
30x = 150 – 30
30x = 120
x = 4 rate of east
bound jogger

x + 2 = 4 + 2 = 6 rate of the west


bound Jogger.

5. let x = number of tickets at P150.00


2x – 5 = number of tickets at P200.00
Equation:

x(150) + 200(2x – 5) = 120,000


150x + 400x -1000 = 120,000
550x = 121,000
x = 220 number
of tickets sold at P150.00

2x- 5 = 2(220) – 5 = 435 number


of tickets sold at
P200.00

33
Common Errors / Misconceptions in
Unit V

1. What expression represents the phrase “


two less than four times a number n”?

Student’s answer : 2 < 4n or 2 – 4n

Both answers are wrong. To translate


verbal phrases to an algebraic expression
you have to identify the key words that
would indicate the operation to be used like
“less than” meaning 2 will be subtracted
from four times a number n”.
Thus, 4n -2 is the correct answer.

2. The students translation of n + 4 > 6 as n


plus 4 greater than 6, instead of n plus 4
is greater than 6.

3.The students failed to write the correct


representation for solution set and solution
of an equation.

34
Module 1
Geometry of Shape and Size Geometry of Shape and Size

What this module is about


This module is about undefined terms and angles. As you go over the exercises you
will learn to name the real-world objects around you that suggest points, lines and planes.
You will develop skills in naming a point, a line and its subsets. You will also learn to name
the parts of an angle and determine its measure in degrees.

What you are expected to learn


This module is designed for you to:

1. describe the ideas of


• point
• line
• plane
2. name the subsets of a line
• segment
• ray
3. name the parts of an angle
4. determine the measure of an angle using a protractor
5. illustrate different kinds of angles
• acute
• right
• obtuse

How much do you know


Identify the term described.

1. It has no length, width, or thickness.


2. It has length but no width and no thickness.
3. It is a flat surface that extends infinitely in all directions.
4. It is the union of two noncollinear rays with a common endpoint.
5. It is an instrument used to determine the approximate measure of an angle.
2
6. An angle with a measure greater than 0 but less than 90
7. It is a subset of a line with two endpoints.
8. An angle with a measure greater than 90 but less than 180.
9. The geometric figure suggested by the ceiling of your room.
10. It is the intersection of two distinct planes.

What you will do


Lesson 1

Undefined Terms

The three undefined terms are point, line and plane. These three undefined terms
form the foundation of geometry. Although they will not be defined they will however be
used in defining other important terms. For example, space is defined as a set of all points.
A point is an exact location in space. It has no length, width or thickness. It is represented
by a dot. Look at the tip of your pen. It suggests a point. A point is named by using a
capital letter. The points below are named points P, Q and R respectively.

• P • Q • R

A line has infinite length, but no width and no thickness. It is an infinite set of points
that extends infinitely in opposite directions. The pen or pencil you are holding right now is
a real world object that suggests a line. A line is represented by . The arrow
suggests that the line continues without end in both directions.

You can name a line in two ways. One way of naming a line is by using two different
capital letters. Observe the line below. It is named line AB written as AB . The double-
headed arrow placed over AB indicates that the line has no endpoints.

A B
• •

Example:

Give six names for the line below.

S T U
• • •
Answers:

ST, TS, TU, UT, SU, US

2
3
The second method of naming a line is by using a small letter. The line below is
named line m.

Like a line, a plane is also a set of infinite points. However, a plane has infinite width
and length but no thickness. It is a flat surface that extends infinitely in all directions. The top
of your dining table, the wall of your room and even a page of this module are examples of
real-world objects that suggest planes.

A slanted four sided figure similar to the one below is used to represent a plane.

You can name a plane in three ways. You may use a capital letter placed at one of its
corners. The plane below is named plane P.

You may use a small letter placed at one of its corners. The plane below is named
plane m.

You may named it by using three points not on a straight line. The plane below is
named plane PRQ.

• P • Q
• R

3
4
The three points below are collinear. Points are collinear if they are on the same line.

D E F
• • •

Example:

List all sets of three collinear points in the figure.

•D
A C
• • •
B

•E
Answers:
:
A, B, C and D, B, E

Consider the three points below. It is not possible to draw one straight line through
the three points A, B and C. These three points are non collinear points.

•A •B

•C

In the figure below, points A, B, and C are in the same plane. Points such as points A,
B, and C, which are in the same plane are called coplanar points. In the same figure, points
A, B, C and D are not coplanar because they do not lie in the same plane. Points A, B, C lie
in plane P, whereas point D lies in plane Q.

P Q
A•

•D
B•
C•

4
5

The following statements describe some basic relationships among points, lines and planes

1. Two points determine exactly one line.


.
a. Through two different points B and C below, you can draw one and only one
line.

B C
• •

In geometry, line means straight line.

b. It is not possible to draw more than one straight line through given two points.
In the following illustration, there is only one straight line that passes through
points C and D. The other line is a curve line.

C D
• •

2. Three collinear points are contained in at least one plane.

. R
•A
Q • B
P • C

In the figure, points A, B, and C are collinear. They lie in plane P, plane Q and
plane R. In fact they can be contained in an infinite number of planes.

3. Three non collinear points are contained in exactly one plane.

Q
•E
P •D

F•

In the figure, points D, E, and F are not collinear. They are contained in exactly
one plane P.

5
6

4. The intersection of two distinct lines is a point.

In the figure, line m and line n intersect and their intersection is point A.

m
A

5. The intersection of two distinct planes is a line.

In the figure below, planes P and Q intersect and their intersection is line AB.

A B

6. If two points are in a plane, then the line containing the points is in the same plane.

If the two points A and B are in plane P, then the line l which contains them lies also
in plane P.

A B
l • •
P

7. A line and a point not on the line are contained in exactly one plane.

In the figure, point A does not lie on line BC. This point and line BC are contained in
one plane P. This is the same as saying they determine exactly one plane P.

C

A B
P • •

6
7
8. Two intersecting lines are contained in exactly one plane.

Example:

Lines a and b which intersect at point P are contained in exactly one plane Q. There
is no other plane that can contain them.

b
Q

9. If a line not contained in a plane intersects the plane, the intersection is a single point.

In the figure, plane P does not contain line m. The intersection of line m and plane P
is a single point Q.

Try this out


Set A.

Determine the undefined term suggested by each of the following.

1. the tip of a pencil


2. the top of a coffee table
3. telephone wires
4. the wall of a room
5. the surface of the page of a book
6. the ruler’s edge
7. the tip of a needle
8. a window pane

7
8
9. the floor of your bedroom
10. the string on a guitar

Set B.

Write True or False

Use the three-dimensional figure below for exercises 1-10.


A

F I E

J D

B H
G
C
1. Points A, F, B are collinear.
2. Points A, E, B are collinear
3. Points B, G and C are on the same line
4. Points G, C, D are not on the same line.
5. Points A, I, H are coplanar.
6. Points A, F, G are coplanar.
7. Points A, F, G , E are coplanar.
8. Points A, F, B, G are coplanar.
9. Points A. I, C are collinear and coplanar.
10. Points A, F, C are collinear and coplanar.

Set C.

Complete the following statements.

1. A ______ is an exact location in space.


2. A ______ has infinite length but no width and no thickness.
3. A ______ has infinite width and length but no thickness.
4. Two points determine exactly one ______
5. Three _________points are contained in at least one plane.
6. Three _________points are contained in exactly one plane.
7. The intersection of two distinct planes is a ________.
8. The intersection of two distinct lines is a ________.
9. Two intersecting lines determine a _________.
10. If a line not contained in a plane intersects the plane, the intersection is a single
_____.

8
9
Lesson 2

The Subsets of a Line

The subsets of a line are segment and ray. A segment has two endpoints. It is
named by its endpoints.

The segment below may be named AC or CA. A vinculum is placed above its
name to distinguish it from the name of a line where the same letters are used.

A C
• •

Example:

Write the name of each segment.


E F M N
a. • • b. • •

Answers:

a. EF or FE b. MN or NM

The length of a segment is the distance between its endpoints.

Example:

If the distance between points C and D below is 9 cm. then the length of segment
CD is 9 cm. This is written as CD = 9 cm. Notice that there is no vinculum above CD.

C D
• •

A segment may be defined as the union of points A, C together with all the
points between them.

Illustration:
A B C
• • •

In the above segment, A and C are the endpoints of the segment. There are points
between A and C. These points together with the endpoints A and C make a segment.
In the above figure, point B is just one of the points between A and C.

9
10
A point such as point B above is between point A and C if and only if (1) A, B, and
C are distinct points, (2) they are collinear and (3) AB + BC = AC. These three conditions
must be satisfied before it can be said that B is between A and C. The word distinct in
the first condition means that the three points are different from one another.

Examples:

1. Draw points C, D, and E on a line. How many different segments are


determined? Name them.
C D E
• • •
Answers:

CD, DE, CE

2. If AB =5 cm, BC = 7 cm, and AC = 12 cm. Is B between A and C?

A B C
• • •

In the figure, A, B, and C are different points on the same line. The sum of the lengths of
AB and BC is equal to the length of AC.

AB + BC = AC
5 cm + 7 cm = 12 cm

Since the three conditions are satisfied, therefore B is between A and C.

Example:

Is F in the figure below between E and G?

F•

E • • G

In the figure , points E, F and G are not collinear, hence F is not between point E and
point G. Also, EF + FG ≠ EG.

A ray is a subset of a line that has one endpoint and extends forever in one
direction.

10
11
Example:

The part of the line from point B that goes on indefinitely to the right is a ray. The part
of the line from point B that goes on indefinitely to the left is another ray.

A B C
l • • •

The ray which starts from point B that goes on indefinitely to the right is named ray
BC denoted by BC. Its endpoint is B. Notice that when you name a ray, you use two
capital letters, and its endpoint is written first. The other ray in the above figure is ray BA,
denoted by BA.

Example: Write a name for each figure.

J K N M
a. • • b. • •

Answers:
a. JK b. MN

Another term you should learn in this lesson is the term opposite rays. Two rays
are opposite if they are subsets of the same line and have a common endpoint.
A B C
• • •

BC and BA are opposite rays. They are parts of the same line l and their common
endpoint is B. F G
E D • •
• •

DE and FG are not opposite rays because they are not subsets of the same line.
A B C D
l • • • •

BA and CD are not opposite rays because they do not have a common endpoint.

Example:
D
Name all the points, segments and rays in the figure.

Answers: A B C

The points are A, B, C, and D. The segments are AB, BC, AC, and BD. The rays
are BA, BC, and BD

11
12
Try this out
Set A:

Use the figure below for exercises nos. 1-10


A B C D
• • • •

1. Name the ray with endpoint at B going in the direction of D.


2. Name the ray with endpoint at C going in the direction of A.
3. Name the segment joining point B with point D.
4. Give two opposite rays with common endpoint C.
5. What is the intersection of ray BD and ray CA?
6. Name the ray opposite BC.
6. Name the ray opposite CA.
7. What point is between points B and D?
8. Give another name for BC
9. Give another name for CB

Set B

Write true or false

Use the following figure


C•
A B D E
• • • •

1. AB + BD = AD
2. AB + BE = AE
3. AC + CD = AD
4. B is between A and D
5. C is between B and D
6. A, B, C, D are collinear
7. AB = AD – BD
8. DE and BA are opposite rays.
9. Ray BE can be named BD.
10. Ray DA can be named AD.

Set C

Fill in the blanks

1. A segment has _________endpoints

12
13
2. A __________is a subset of a line with one definite endpoint and extends infinitely in
one direction.

3. _________are two collinear rays with a common endpoint.

Use the figure at the right for exercises nos. 4-8

• • •
C D E

4. CD + ______ = CE.
5. The ray opposite DE is ________
6. The ray with endpoint C going in the direction of D is _________
7. The ray with endpoint E going in the direction C is __________
8. The point between two other points is ________.

9. If two points P and Q are exactly the same point, then the distance between them is
______

10. The endpoint of each ray in the figure is _______

Lesson 3

Angles

An angle is a union of two noncollinear rays with a common endpoint. The common
endpoint is called the vertex of the angle and the two rays are called sides.

Example:

The figure below is an angle. Its vertex is point B and its two sides are BA and BC.
The symbol used for an angle is ∠. The angle in the example can be named ∠ ABC. It can
also be called ∠CBA. The letter representing the vertex is written between the other two
letters.

A •

B •
C

An angle may be written in other ways.

13
14
Example:

Angle DFG can also be named ∠EFG, ∠GFD, ∠GFE, ∠F and ∠a. Angle HIJ can
be named ∠ 1.

D•
H•
E•
a 1
F • I •
G J

There are times when it is not advisable to use the vertex letter in naming an angle.
Using it may result to confusion.

Example:

Angle ABC below may be named ∠ B

A •

B •
C

Angle ABC below should not be named ∠B. In the figure, there are three angles with
vertex B. They are ∠ABC, ∠DBC and ∠ABD.

A • C•

B •
Example: D

Give three different names for the angle shown below.

P •
• •
Answers: Q R
∠ PQR, ∠ RQP, ∠Q

An angle separates a plane into three sets: the points on the angle, the interior of the
angle, and the exterior of the angle.

14
15
Example:

In the figure, points T and S are on ∠ABC.. Point P is in the interior and points Q
and R are in the exterior of the angle.


•Q

Exterior Interior
•P
B • • •
T C
•R
Exterior
The Measure of an angle
You can determine the measure of an angle in degrees by means of a protractor.
You can do this by placing the center mark of the protractor on the vertex of the angle you
want to measure and then placing the 0 degree mark on one side of the angle. Then read
the number where the other side crosses the scale. You can also use a protractor in
constructing an angle of a given measure.

Example:

Measure angle ABC below. A

B C
The measure of ∠ABC as indicated in the protractor is 90 degrees. This can be
written in two ways.

∠ ABC = 900 (Angle ABC equals 90 degrees.)


m∠ABC = 90. (The measure of ∠ABC is 90.)

15
16

In this module the measure of an angle is always greater than 0 degree but less than
180 degrees. This restriction will be followed in this module because of the definition of an
angle.

Addition of Angles

The measures of two or more angles can be added.

Example

The measure of ∠A is 500 and the measure of ∠ B is 600. Find the sum of their
measures.

A 500 B 600

m ∠ A + m ∠ B = 500 + 600.
= 1100

Example

∠ABD and ∠CBD are two coplanar angles with a common side BD. If m ∠ ABD = 40
and m ∠ CBD = 30, find the measure of angle ABC.

A
D

400

B 300 C

m ∠ ABD + m ∠ CBD = 400 + 300


= 700

Example:

If m∠ABC = 120, m∠ABD = 2x + 10, and m∠CBD =3 x [Use the preceding figure]
Find m∠ABD.

16
17

m∠ABD + m∠CBD = m∠ABC


2x + 10 + 3x = 120
2x + 3x = 120 – 10
5x = 110
x = 22
2x + 10 = 2(22) + 10
= 44 + 10
= 54

Kinds of Angles
There are three kinds of angles according to measure. They are the following.

1. Acute angle- is an angle with a measure grater than 0 but less than 90.
∠ABC below is an acute angle.

450 C
B

2. Right angle- is an angle with a measure of 90.


∠ABC below is a right angle.

E
F

The symbol in the corner of a right of the figure indicates that the measure of the
angle is 90.

3. Obtuse angle – is an angle with a measure greater than 90 but less than 180,

G 1100

H I

17
18
Try this out
Set A.

1. Name the angle below in three ways.

B
C
2. Which is the vertex letter in angle STG?
3. Name the three angles in the figure below.

D
F

E G

4. What are the sides of ∠ BET?


5. What is the common side of ∠ABD and ∠CBD?
6. Into how many sets does an angle separate a plane?
7. Is the vertex of an angle in its interior?
8. How many angles are there in the figure?
E

A B
C
D

9. Is the figure below an angle? Why? Why not?

B C

10. Explain why it is not correct to name the angle below ∠ Q?

P R

Q S

18
19
Set B. Use the figure below for exercises 1-10. The three angles in the figure are
coplanar.

A D

C
B

1. If m∠ABD = 80 and . m∠CBD = 40, find the m∠ ABC.


2. If m∠CBD = 30 and m∠ABD = 85, find the m∠ABC
3. If m∠ABD =45.5 and m∠CBD= 44, find the m∠ABC.
4. If the m∠CBD = 30.5 and m∠ ABD = 65, find the m∠ABC.
5. If m∠ABC =110 and . m∠CBD = 40, find the m∠ ABD.
6. If m∠ABC =115 and . m∠ABD = 40, find the m∠ CBD
7. If m∠ABC =84 and . m∠CBD = 2x, and m∠ABD = 4x, find m∠ABD
8. If m∠ABC =96 and . m∠CBD = x, and m∠ABD = 2x, find m∠CBD

Use a protractor for exercises 9-10

9. Construct an angle with a measure of 45 degrees.


10. Construct an angle with measure of 125 degrees.

Set C. Use the figure below for exercise 1 –3.

A •

D• •F •K
•G
• • •
B E C
•H
1. Name all the points in the interior of ∠ABC.
2. Name all the points in the exterior of ∠ABC.
3. Name all the points that are neither on the exterior nor interior of ∠ABC.

Use the figure below for exercises 4-6

E D

A C

19
20

In the figure BC and BA are opposite rays.

4. Name all the angles determined in the figure.


5. Tell whether the angles in the figure are acute, right or obtuse.
6. Name the two angles with the same measure.
7. In the figure below, BA, BD and BC are coplanar rays. If ∠ABC is a right angle, find x

A
D

(3x)0
x0
B C

8. Which of the following angles is an acute angle

a. b. c.

9. Using your protractor, find the measure of each angle below.

a. b.

10. Draw angles with the following measures.


a. 1250 c. 900
b. 350 d. 1400

Let’s summarize
1. The three undefined terms in geometry are point, line and plane.
2. A line is an exact location in space. It has no length, width or thickness.
3. A line has infinite length, but no width and no thickness.
4. A plane has infinite width and length but no thickness.
5. Two points determine exactly one line.
6. Two distinct lines intersect in only one point
7. Collinear points are points on the same line.
8. Coplanar points are points on the same plane.
9. Three collinear points are contained in at least one plane.
10. Three noncollinear point are contained in exactly one plane.

20
21
11. The intersection of two distinct planes is a line
12. If two points are in a plane, then the line containing the points is in the same line.
13. A line and a point not on the line, are contained in exactly one plane.
14. Two intersecting lines are contained in exactly one plane.
15. If a line not contained in a plane intersects the plane, the intersection is a single
point.
16. A segment is a subset of a line that consists of two endpoints and all the points
between them.
17. A ray is a subset of a line with a definite endpoint and extends infinitely in one
direction.
18. An angle is the union of two noncollinear rays with a common endpoint.
19. An angle separates the plane into three sets: the points in the interior of the angle,
the points in the exterior of the angle and the points on the angle itself.
20. A protractor is used to measure an angle in degrees.
21. An angle with a measure greater than 0 but les than 90 is an acute angle.
22. An angle with a measure of 90 is a right angle.
23. An angle with a measure greater than 90 but less than 180 is an obtuse angle.

What have you learned


Multiple Choice. Choose the letter of the correct answer.

1. It is flat surface that extends infinitely in all directions.


A. Point C. Plane
B. Line D. rectangle

2. It is a set of points that extends forever in opposite directions.


A. Point C. Plane
B. Line D. Space.

3. Which of the following is false?


A. Exactly one plane contains two intersecting lines.
B. Two points determine a line.
C. The intersection of two distinct planes is a line
D. Three collinear points are contained in exactly one plane

4. Which of the following real objects suggest a point?


A. The edge of the beam of a building
B. The corner of the Main street and the 1st Ave.
C. The floor of a newly constructed building.
D. The wall of your room.

5. It is a subset of a line with a definite endpoint and extends infinitely in one direction.
A. Ray C. Opposite Rays

21
22
B. Segment D. Plane

6. It is the union of two noncollinear rays with a common endpoint.


A. Plane C. Space
B. Angle D. Segment

7. It is an angle with a measure greater than 0 but less than 90.


A. Acute angle C. Obtuse angle
B. Right Angle D. non of these

8. It is angle with a measure of 90.


A. Acute angle C. Obtuse angle
B. Right Angle D. none of these

9. Which of the following angles is obtuse?

I III.

II. IV.

A. I only C. II only
B. I and II D. I and III

10. It is used to measure an angle in degrees.

A. Compass C. protractor
B. Ruler D. tape measure

22
23

Answer Key
How much do you know

1. point
2. line
3. plane
4. angle
5. protractor
6. acute angle
7. segment
8. obtuse angle
9. plane
10. line

Try this out

Lesson 1

Set A

1. point
2. plane
3. line
4. plane
5. plane
6. line
7. point
8. plane
9. plane
10. line

Set B

1. True
2. False
3. True
4. True
5. True
6. True
7. False
8. True
9. True
10. False

23
24

Set C

1. point
2. line
3. plane
4. line
5. non-collinear
6. collinear
7. line
8. point
9. plane
10. point

Lesson 2

Set A
.
1. BC or BD
2. CA or CB
3. BD
4. CD and CA or CD or CB
5. BC
6. BA
7. CD
8. C
9. CB
10. BC, AB

Set B

1. True
2. True
3. False
4. True
5. False
6. False
7. True
8. False
9. True
10. False

Set C

1. two
2. ray

24
25
3. opposite rays
4. DE
5. DC
6. CD or CE
7. EC or ED
8. D
9. 0
10. B

Lesson 3

Set A

1. ∠ABC, ∠CBA, ∠B
2. T
3. ∠DEF, ∠GEF, ∠DEG
4. EB, ET
5. BC
6. Three sets including itself
7. No.
8. 4
9. No
10. Q is the vertex of the three angles. ∠Q may mean ∠PQR, ∠RQS, and ∠PQS

Set B

1. 120
2. 115
3. 89.5
4. 95.5
5. 70
6. 75
7. 56
8. 32
9. Use your protractor
10. Use your protractor

Set C.

1. F, K
2. G, H
3. A, D, B, E, C
4. ∠ABE, ∠ABD, ∠DBE, ∠EBC, ∠DBC
5. ∠ABE and ∠DBE are acute angles
∠ABD and ∠DBC are right angles
∠EBC is obtuse

25
26
6. ∠ABD and ∠DBC. Both are right angles with measure of 900 each.
7. 22.5
8. a
9. Use your protractor
10. Use your protractor

What have you learned

1. C
2. B
3. D
4. B
5. A
6. B
7. A
8. B
9. C
10. C

26
Angles, Angle Measures and Angle Pairs

We use angles everyday. The airplanes that we see flying, form angles:
the horizontal line and the line of sight from your eyes to the airplane. Good pool
players use angles to plan their shots. In military, soldiers use angles when firing
canons.

What is an angle? You can use the geometric terms that you defined in Pictures of a wall clock and
a wristwatch
previous lessons. An angle is formed by two noncollinear rays that have the same
endpoint. These rays are called the sides of the angles and the endpoint is the Is the angle between two hands of a
wall clock smaller than the angle
vertex. between the hands of a your
wristwatch?
Side
Vertex: A
Vertex 1
Sides: AB and AC

Side

We can name the angle in the figure above angle BAC or CAB or use the angle symbol and write BAC and
CAB. Take note that the vertex must be the middle letter and the first and last letters each name a point on a
different ray. Since there are no other angles with vertex A, we can also simply call this A. We can also name the
angle using a number in its interior, therefore, the angle could be named as 1.

Example 1: .
W
X
2
Name all the angles in the figure. Y

Z
Solution:

The angles are Z, Y, ZXY (or YXZ), WXZ (or ZXW) . Note that 2 is the shorter name for ZXY.

Which angles in Example 1 seem larger to you? Which seem smaller?

Angles are measured in degrees. The measure of an angle tells how wide open the angle from the vertex
from one ray to the other. Based on this definition, the measure of an angle can be any value between 0° and
180°.
To measure an angle, you can use a protractor. Below is the step on how to use it.

Picture of a Protractor

1
a. Place the center mark of the protractor on the vertex of the given angle.

b. Line – up the zero – mark on the edge of the protractor with one side of the angle.

c. Locate the point where the other side of the angle intersects the curved edge of the protractor.

d. Read the measure on the protractor. This is the measure of the angle.

Use m before the angle symbol to show the measure of an angle. For example, mABC = 500 means the measure
of ABC is 500.

Example 2:

(a) (b)

Use a protractor to measure the two angles. Which of them measures less than 900?

Consider the two angles below.

A X

B C Y
Z

Find the measures of ABC and XYZ using a protractor and compare them.

You will find out that mABC = XYZ. If two angles have equal measure, then they are congruent.

In symbols, ABC  XYZ.

Example 3:

Two angles, ABC and XYZ are congruent. If mABC = 6x + 2 and mXYZ = 8x – 14, calculate the
measurements of ABC and XYZ.

Solution:

ABC  XYZ → 6x + 2 = 8x – 14 Definition of Congruent angles


6x – 8x = – 14 – 2 Solve for x.
– 2x = – 16
2
x=8

Hence, mABC = mXYZ = 6(8) + 2 = 500.

Angles can be classified by their measures.


A < 900
1. Acute Angle – If an angle A has measure 00 < A < 900, then A is said to be acute.
This symbol means a
2. Right Angle – If an angle A has measure equal to 900, the A is said to tbe right. 900 angle.

3. Acute Angle – If an angle A has measure 900 < A < 1800, then A is said to be obtuse.
900 < A < 1800

Examples 4:

. E .
Measure each angle and classify it as acute, right or obtuse.
D .
F

1. ABD acute
2. DBF right
.A .B .C
3. FBA obtuse
4. FBC acute

ANGLE ADDITION POSTULATE


. . O
Let O be the interior of ABC. Then, A
ABO + OBC = ABC.
.
B
.
C

Examples 5:

ABD and CBD have a common side BD. If ABD = 250 and CBD = 400, find the measure of
ABC.

Solution:

250
400

Using angle addition postulate, ABD + CBD = ABC.

Hence, ABC = 250 + 400 = 650.

3
Examples 6:

Given LMN = 1350. If LMV = 45x – 1 and VMN = 23x, find x.

Solution:

LMV + VMN = LMN Angle Addition Postulate


45x – 1 + 23x = 135 Solve for x.
68x – 1 = 135
68x = 136
x=2

ANGLE BISECTOR

Let ABC be a right angle and let D be interior of the angle. If BD divides into ABC into two congruent
angles, what are the measures of ABD and DBC?
If BD divides the angle into two congruent angles, then BD is the angle bisector of ABC.
Hence, ABD = DBC = 450.

The bisector of an angle is a ray or a line segment whose endpoint is the


vertex of the angle and which divides the angle into two equal angles.

CAB  DAB
Example 7:

AC is a straight line. BD bisects ABE and BF bisects CBF.


.D E . .
a. If CBF = 8x – 10 and EBF = 10x – 20, find EBC.
b. If ABD = 5x + 10 and EBD = 8x – 23, find EBD.
F
.C
.B
.
A
Solution:

a. Since BF bisects EBC, then CBF = EBF.

8x – 10 = 10x – 20 → x = 5

Hence, CBF = EBF = 8(5) – 10 = 10(5) – 20 = 300.

Using angle addition postulate, EBC = 2(300) = 600.

b. BD bisects ABE. This means that ABD = EBD.

5x + 10 = 8x – 23 → x = 11

Thus, EBD = 8x – 23 = 8(11) – 23 = 560.

4
ANGLE PAIRS
1. Complementary Angles
If the sum of two angles is 900, then they are complementary.
Example 8:
a. What is the complement of 500?
Solution:
By definition, two angles are complementary if their sum is 900. Hence, the complement
of 500 is 400.

b. The difference between two angles is 120. Find their measures if the two angles are
complementary?
Solution:
Let x be bigger angle.
90 – x be the smaller angle.

x – (90 – x) = 12
2x – 90 = 12
2x = 102
x = 51
90 – x = 390

Hence, the two angles are 510 and 390 and are complementary.
2. Supplementary Angles
If the sum of the two angles is 1800, then they are supplementary.
Example 9:
a. Find the supplement of 700.
Solution:
Use the definiton of supplementary angles. Thus, the supplement of 700 is 1100.
b. The angle is three times its supplement. Find the angle.

Solution:
Let x be the angle. Hence, the supplement of the angle is 180 – x.
Our equation will be,

x = 3(180 – x)
x = 540 – 3x
4x = 540
x = 1350

Therefore, the angle is 1350.

5
3. Linear Pair

On a short bond paper, draw line AB and place a point C between A and B. Choose another point
D not on PQ and draw CD. A linear pair of angles was created.

A linear pair is two adjacent angles whose noncommon sides form two opposite rays.

D
1 and 2 are linear pair.

Using a protractor, place the zero – edge along AC. What is the sum of ABD and CBD? What do
you notice about the sum of the measures of the linear pair of angles?

Linear Pair Postulate


The linear pair postulate states that two angles that form a linear pair are supplementary.

Example 10:

a. Consider the figure on the right. If CBD = 650, D


find the measure of ABD.
650
Solution: A B C

ABD and CBD are a linear pair. By Linear Pair Postulate, they are supplementary.
To find the measure of ABD, subtract the measure of CBD from 1800.

ABD = 1800 – 650


= 1150

b. 1 and 2 is a linear pair. If 1 = 4x – 20 and 2 = x + 10, find the measure of 2.

Solution:

Since 1 and 2 are a linear pair, we have 1 + 2 = 1800.

4x – 20 + x + 10 = 180
5x – 10 = 180
x = 380

Therefore, 2 = 38 + 10 = 480.

6
4. Vertical Angles

Consider a scissor below.

2
1 3
4

1 and 2 are vertical angles. 2 and4 are also vertical agles. Based on the figure, two
angles are vertical if and only if they are not adjacent and their sides are formed by two intersecting lines.

Vertical Angle Theorem

Draw two intersecting lines onto tracing paper. Label the


angles as shown. Fold the paper so that the vertical angles (1 and 3 ;
2 and  4) lie over each other. What do you notice about their
measures?

You discovered the Vertical Angles Conjecture.

Vertical Angle Theorem states that if two angles are vertical, then they are congruent. Does that
also mean that all congruent angles are vertical angles?

Examples 11:

Find 1, 2 and 3. 1 2


3 550
Solution:

1 and 550 are vertical angles. Hence, 1 = 550.

1 and 2 is a linear pair. Thus, 2 = 1800 – 550 = 1250.

2 and 3 are vertical angles. Therefore, 3 = 1250.

7
Example 12:

Solve for x. 4x – 42
2x

Solution:

The two algebraic expressions are measures of vertical angles. Using the vertical angle theorem,

4x – 42 =2x
2x = 42
x = 21
Therefore, the x = 21.

TRY THIS

1. Name the vertex and the sides of the angle.

a. F b. c.
Q L P T

R C U V

2. Write three names for the angle.


G L
a. b. c.
C 1 7 A
S
T X
T

3. Find the measure of the following angles.

900
700
S
R
1600
T
Q 150

U V P

a. RVQ d. SVQ

b. TVR e. SVT

c. QVT

8
4. Complete the table below.

Angle Complement Supplement

400

300

1240

850

71.250

LET US DO THIS

Answer the following problems.

1. BD bisects ABC. If ABC = 1100, find ABD.

2. What is the complement of the supplement of 1200?

3. AB and CD intersect at O. If AOC = 450, find BOD.

4. Four angles are in the ratio 1:2:3:4. Find the measure of the largest angle.

5. P measures 70 more than three times Q. If two angles form a linear pair, find the measure of each
angle.

6. The sum of three angles is 2050. If 1 = x and 2 = 3x and 3 = x – 5, find the measure of each angle.

7. How many degrees is the angle traced by the hands of a clock in 30 minutes?

8. How degrees is generated by 3.5 revolutions?

9. An angle is 8 times its supplement. Find the two angles.

10. 3 and 4 are vertical angles. If 3 is 100 more than twice the measure of 4, find 3 and 4.

11. If B = (4x – 1)0 and C= (5x + 3)0 and they are complementary, solve for x.

12. The supplement of an angle is 160 more than three times its complement. Find angle and its supplement.

13. The sum of the interior angles (S) of a polygon is given by the formula S = 1800 (n - 2) where n is the
number of sides. Find the sum of the interior angles of a decagon.

9
14. Solve for x. (3x – 1)0 (2x + 6)0

15. Aling Maria has a mango three and she attaches some wires from the trunk to the ground. The obtuse
angle the wire makes with the ground is supplementary to the acute angle the wire makes. It is also
known that the obtuse angle is three times as large. Find the obtuse angle.

EXPRESS YOUR IDEAS

1. The sum of the angles of a triangle is 1800. Can a triangle have two obtuse angles? Explain your answer.

2. Are all right angles congruent? Justify your answer.

3. Look up the words complimentary and complementary in an English dictionary. Give their meanings.

STRIVE HARDER

1.

2 rays → 1 angle

3 rays → 3 angles

4 rays → 6 angles

Using the pattern above, find the angles formed by 5 rays. Find the formula in finding the number
angles formed by n noncollinear rays with a common endpoint.

2. Prove that the difference between the measures of the supplementary and complementary of an
angle is always 900.

10
CLICK I.T.

Using MS Excel, complete the table below.

11
Parallel Lines
Module 3
Geometric Relations

What this module is about


This module is about the angles formed by parallel lines (//) cut by a
transversal. You will learn to determine the relation between pairs of angles
formed by parallel lines cut by a transversal and solve problems involving
segments and angles.

What you are expected to learn


This module is designed for you to:

1. identify the angles formed by parallel lines cut by a transversal.

2. determine the relationship between pairs of angles formed by parallel


lines cut by a transversal:

• alternate interior angles


• alternate exterior angles
• corresponding angles
• angles on the same side of the transversal

3. solve problems using the definition and properties involving


relationships between segments and between angles.

How much do you know


The figure below shows lines m // n with t as transversal.

Name: m
7
8
6 n
1. 4 pairs of corresponding angles 5
3
2. 2 pairs of alternate interior angles 4 2
1
3. 2 pairs of alternate exterior angles t
4. 2 pairs of interior angles on the same side of the transversal
5. 2 pairs of exterior angles on the same side of the transversal

Using the same figure:

6. Name all numbered angles congruent to ∠ 7.


7. Name all numbered angles congruent to ∠ 4.
8. Name all numbered angles supplementary to ∠ 8, to ∠ 7.
9. Name all numbered angles supplementary to ∠ 3, to ∠ 4.
10. Name the pairs of equal angles and supplementary angles in the
figure.

B
Given: AB // CD
AD // BC
A

In the figure below,


A
AB ⊥ BD,
DF ⊥ BD,
1 C
BC // DE B
2

3 D
E 4

11. Is m ∠ 2 = m ∠ 3? Why?
12. ∠ 2 is a complement of _____ and ∠ 3 is a complement of ____.
13. Is m ∠ 1 + m ∠ 2 = m ∠ 3 + m ∠ 4? Why?

2
Find the value of x in each of the following figure:

14.

70o

xo 55o

15.

xo

140o

110o

16.
3x

xo

17.

4x

xo

3
In the figure, write down the pairs of parallel lines and the pairs of
congruent angles.

18.
A C B

F
D
E

19. If m ∠ 3 = 135, find the measure of each angle in the figure.

1 2 m
4 3

5 6 n
8 7

20. If m ∠ 6 = 85, find the measure of each numbered angle in the


figure, a // b and c // d.

6 7 9 10 m
5 8 12 11

4 3 13 14 n
1 2 16 15

c d

4
What you will do

Lesson 1

Angles Formed by Parallel Lines Cut by a Transversal

In the rectangular solid below, AB and CD are coplanar in plane x. AB


and EF are coplanar in plane y. EF and HF are coplanar in plane z.

G
C
F
B
z
x y
H
D
E
A

Line E intersect AB and CD at two different points. Line E is a transversal


of lines AB and CD.

Definitions:

Coplanar lines are lines that lie in one plane.

Parallel lines are two lines that are coplanar and do not intersect.

Transversal is a line that intersects two or more lines at different points.

E B
A

C D

Line E intersect AB and CD at 2 different points. Line E is a transversal of


lines AB and CD.

5
Examples:

1. In the figure, lines AB and CD


1 2
are parallel lines cut by transversal
A 4 3 B
line t. The angles formed are:

Angles 1, 2, 7 and 8 are exterior angles


5 6
Angles 3, 4, 5, and 6 are interior angles C 8 7 D

The pairs of corresponding angles are: t

∠ 1 and ∠5
∠ 2 and ∠6
∠ 4 and ∠8
∠ 3 and ∠7

The pairs of alternate interior angles are:

∠ 3 and ∠ 5
∠ 4 and ∠ 6

The pairs of alternate exterior angles are:

∠ 1 and ∠ 7
∠ 2 and ∠ 8

The pairs of exterior angles on the same side of a transversal (SST) are:

∠ 1 and ∠ 8
∠ 2 and ∠ 7

The pairs of interior angles on the same side of a transversal (SST) are:

∠ 4 and ∠ 5
∠ 3 and ∠ 6

2. Given: m // n, s is the transversal. The pairs of angles formed are:

a. Corresponding angles: 3 4 m
6 5
∠ 3 and ∠9
∠ 6 and ∠ 12
∠ 4 and ∠ 10
∠ 5 and ∠ 11 9 10 n
14 11

6
b. Alternate exterior angles

∠ 3 and ∠ 11
∠ 4 and ∠ 12

c. Alternate interior angles

∠ 6 and ∠ 10
∠ 5 and ∠ 9

d. Interior angles on the same side of the transversal (SST).

∠ 6 and ∠ 9
∠ 5 and ∠ 10

e. Exterior angles on the same side of the transversal (SST).

∠ 3 and ∠ 12
∠ 4 and ∠ 11

3. m // n, t is the transversal.

In the figure, name and identify


he pairs of angles formed: 1 2 m
8 7

9 10 n
12 11
a. Corresponding angles:

∠ 1 and ∠9 t
∠ 8 and ∠ 12
∠ 2 and ∠ 10
∠ 7 and ∠ 11

b. Alternate interior angles

∠ 8 and ∠ 10
∠ 7 and ∠ 9

7
c. Alternate exterior angles

∠ 1 and ∠ 11
∠ 2 and ∠ 12

d. Exterior angles on the same side of the transversal (SST).

∠ 1 and ∠ 12
∠ 2 and ∠ 11

e. Interior angles on the same side of the transversal (SST).

∠ 8 and ∠ 9
∠ 7 and ∠ 10

Try this out


1. In the figure, lines g // h and is cut by line k.

4 1 g
3 2

8 5 h
7 6

k
Name and identify the pairs of angles formed

2. In the figure, q // r and s // t.

15 16 7 8 q
14 13 6 5

11 12 3 4 r
10 9 2 1

t
s
Name and identify the pairs of angles formed.

8
3. In the figure, u // w. Lines x and y are transversals.

6 5 1 2 u
7 8 4 3

9
10 14
w
11 13
12
x
y
Name and identify the pairs of angles formed

4. In the figure, identify and name the pairs of parallel lines and its
transversal.
A
B

C D
G

E F

5. In the figure, identify and name the pairs of parallel lines and its
transversal / transversals.
a b

9
Lesson 2

Relationship Between Pairs of Angles Formed


by Parallel Lines Cut by a Transversal

If two lines are cut by a transversal, then:

a. corresponding angles are congruent


b. alternate interior angles are congruent
c. alternate exterior angles are congruent
d. interior angles on the same side of a transversal are supplementary
e. exterior angles on the same side of a transversal are supplementary

Examples:

1. Given: p // q, r is a transversal

Figure:

1 4 p
2 3

5 8 q
6 7

What is the measure of each numbered angles if m ∠ 1 = 120? Give the


reason for your answer.

Answers:

If ∠ 1 = 120o, ∠ 5 = 120o Corresponding angles are ≅ .


If ∠ 5 = 120o, ∠ 3 = 120o Alternate interior angles are ≅
If ∠ 3 = 120o, ∠ 7 = 120o Corresponding angles are ≅
If ∠ 7 = 120o, ∠ 4 = 60o Exterior angles on the same side
of a transversal are supplementary
If ∠ 4 = 60o, ∠ 8 = 60o Corresponding angles are ≅
If ∠ 8 = 80o, ∠ 2 = 60o Alternate interior angles are ≅
If ∠ 2 = 60o, ∠ 6 = 60o Corresponding angles are ≅

10
2. Given: In the figure, if m ∠ 1 = 105, determine the measures of the other
numbered angles. Justify your answers.

Figure: t

1 2
A 4 3 B

5 6
C 8 7 D

Answers:

If ∠ 1 = 105o, ∠ 5 = 105o Corresponding angles are ≅ .


If ∠ 5 = 105o, ∠ 3 = 105o Alternate interior angles are ≅
If ∠ 3 = 105o, ∠ 7 = 105o Corresponding angles are ≅
If ∠ 7 = 105o, ∠ 2 = 75o Exterior angles on the same side
of a transversal are supplementary
If ∠ 2 = 75o, ∠ 8 = 75o Alternate exterior angles are ≅
If ∠ 8 = 75o, ∠ 4 = 75o Corresponding angles are ≅
If ∠ 4 = 75o, ∠ 6 = 75o Alternate interior angles are ≅

3. Given: m // n, s and t are the transversals t s

1 2 3 4
If m ∠ 5 = 110 and m ∠ 12 = 90, determine m
8 7 6 5
the measures of the other numbered ∠ s.
Justify your answer.
10
9 11 n
14 12
13

Answers:

If m ∠ 5 = 110, m ∠ 12 + m ∠ 13 = 110
Since m ∠ 12 = 90, m ∠ 13 = 20. Corresponding angles are ≅

m ∠ 4 = 70 ∠ 4 and ∠ 5 are supplementary

If m ∠ 4 = 70, m ∠ 11 = 70 Corresponding angles are ≅

m ∠ 3 = 110 ∠ 3 and ∠ 5 are vertical angles

11
If m ∠ 3 = 110, m ∠ 9 + m ∠ 10 = 110 Corresponding angles are ≅

If m ∠ 9 = 90, m ∠ 7 = 90 Alternate interior angles are ≅

If m ∠ 12 = 90, m ∠ 2 = 90 Exterior angles on the same side


of a transversal are supplementary

Since m ∠ 9 = 90, m ∠ 1 = 90 Corresponding angles are ≅

If m ∠ 13 + m ∠ 14 = m ∠ 8, m ∠ 8 = 90 Corresponding angles are ≅

Therefore, the measures of the numbered angles are:

∠ 1 = 90o ∠ 8 = 90o
∠ 2 = 90o ∠ 9 = 90o
∠ 3 = 110o ∠ 10 = 20o
∠ 4= 70o ∠ 11 = 70o
∠ 5 = 110o ∠ 12 = 90o
∠ 6 = 70o ∠ 13 = 20o
∠ 7 = 90o ∠ 14 = 70o

Try this out


1. If m ∠ 6 = 85, find the measure of the other numbered angles. Justify your
answers.

Given: a // b, c is the transversal

Figure: a b

1 2 5 6
c
4 3 8 7

12
2. If m ∠ 10 = 118 and m ∠ 4 = 85, find the measures of the other numbered
angles. Justify your answers.

Given: f // g, r // q
8 7 9 10 f
5 6 12 11

4 3 13 14 g
1 2 16 15

q
r

3. In the figure, u // w. Lines x and y are transversals. If m ∠ 8 = 62, m ∠ 14


= 90, find the measures of the other numbered ∠ s. Justify your answers.

Figure:

6 5 1 2 u
7 8 4 3

9
10 14
w
11 13
12
x
y

Form an equation in x and solve the equation.

4.
2xo

xo

13
5.
xo

120o

100o

Let’ summarize
The angles formed by parallel lines cut by a transversal are:

1. corresponding angles
2. alternate interior angles
3. alternate exterior angles
4. exterior angles on the same side of a transversal
5. interior angles on the same side of a transversal

The relationship of the angles formed by parallel lines cut by a transversal


are:

1. pairs of corresponding s are ≅


2. pairs of alternate interior angles are ≅
3. pairs of alternate exterior angles are ≅
4. pairs of interior angles on the same side of a transversal are
supplementary
5. pairs of exterior angles on the same side of a transversal are
supplementary

14
What have you learned
The figure below shows lines m // n with t as transversal.

Figure: t

1 2 m
4 3

5 6 n
8 7

Name:

1. 4 pairs of corresponding angles.


2. 2 pairs of alternate interior angles.
3. 2 pairs of alternate exterior angles.
4. 2 pairs of interior angles on the same side of a transversal.
5. 2 pairs of exterior angles on the same side of a transversal.

Using the same figure:

6. Name all numbered angles congruent to ∠ 7.


7. Name all numbered angles congruent to ∠ 4.
8. Name all numbered angles supplementary to ∠ 8, ∠ 7.
9. Name all numbered angles supplementary to ∠ 3, ∠ 4.
10. Name the pairs of equal angles and supplementary angles in the figure.

Given: AB // CD Figure:
AD // BC
A B

D C

15
11. – 13. In the figure, AB ⊥ BD, DF ⊥ BD, BC // DE

Figure: A

1
2
D B
3
4
E
F

11. Is m ∠ 2 = m ∠ 3? Why?
12. ∠ 2 is a complement of _____ and ∠ 3 is a complement of _____.
13. Is m ∠ 1 + m ∠ 2 = m ∠ 3 + m ∠ 4? Why?

Find the value of x in each of the following figures.

14.

50o

xo xo

15.
100o

120o

xo

16
16.
5x

xo

17.
xo

4x

In the figure, Write down the pairs of parallel lines and the pairs of congruent
angles.

18. B A

F D C

17
19. If m ∠ 1 = 135, find the measure of each angle in the figure below:
t

1 2 m
4 3

5 6 n
8 7

20. If m ∠ 6 = 75, find the measure of each numbered angle in the figure, a //
b and c // d.

6 7 9 10 a
5 8 12 11

4 3 13 14 b
1 2 16 15

d
c

18
Answer key
How much do you know

1. ∠ 8 and ∠ 4, ∠ 7 and ∠ 3
∠ 5 and ∠ 1, ∠ 6 and ∠ 2
2. ∠ 5 and ∠ 3, ∠ 4 and ∠ 6
3. ∠ 8 and ∠ 2, ∠ 7 and ∠ 1
4. ∠ 5 and ∠ 4, ∠ 6 and ∠ 3
5. ∠ 8 and ∠ 1, ∠ 7 and ∠ 2
6. ∠ 3, ∠ 1
7. ∠ 8, ∠ 6
8. To ∠ 8: ∠ 1, ∠ 5, ∠ 7; To ∠ 7: ∠ 2, ∠ 8, ∠ 6
9. To ∠ 3: ∠ 6, ∠ 4, ∠ 2; To ∠ 4: ∠ 5, ∠ 3, ∠ 1
10. ∠ A ≅ ∠ C, ∠ B ≅ ∠ D, ∠ A supplement ∠ B, ∠ B supplement ∠ C
∠ C supplement ∠ D, ∠ D supplement ∠ A

11. They are alternate interior ∠ s


12. ∠ 1, ∠ 4
13. Yes, because they are right angles.
14. x = 55o
15. x = 110o
16. x = 45o
17. x = 36o
18. AB // DE; CD // FG; ∠ ACD ≅ ∠ EDC; ∠ EFG ≅ ∠ CDF
19. ∠ 2 = 45o; ∠ 4 = 45o; ∠ 1 = 135o; ∠ 7 = 135o; ∠ 8 = 45o; ∠ 6 = 45o; ∠ 5
o
= 135
20. ∠ 4 = 85o; ∠ 5 = 95o; ∠ 1 = 95o; ∠ 7 = 95o; ∠ 3 = 95o; ∠ 8 = 85o; ∠ 2 = 85o
∠ 10 = 95o; ∠ 9 = 85o; ∠ 12 = 95o; ∠ 11 = 85o; ∠ 13 = 85o; ∠ 14 = 95o;
∠ 16 = 95o; ∠ 15 = 85o

19
Try this out

Lesson 1

1. ∠ 4 and ∠ 8; ∠ 3 and ∠ 7; ∠ 1 and ∠ 5; ∠ 2 and ∠ 6 are corresponding


angles;
∠ 4 and ∠ 7; ∠ 1 and ∠ 6 are exterior angles on SST;
∠ 3 and ∠ 5; ∠ 2 and ∠ 8 are alternate interior angles;
∠ 4 and ∠ 7; ∠ 1 and ∠ 6 are exterior angles on SST;
∠ 3 and ∠ 5; ∠ 2 and ∠ 8 are alternate interior angles;
∠ 3 and ∠ 8; ∠ 2 and ∠ 5 are interior angles on SST;
∠ 4 and ∠ 6; ∠ 1 and ∠ 8 are exterior angles on SST;

2. ∠ 1 and ∠ 5; ∠ 4 and ∠ 8; ∠ 2 and ∠ 6; ∠ 3 and ∠ 7 are corresponding


angles;
∠ 4 and ∠ 6; ∠ 3 and ∠ 5 are alternate interior angles;
∠ 7and ∠ 2; ∠ 8 and ∠ 1 are exterior angles on SST;
∠ 6 and ∠ 3; ∠ 5 and ∠ 4 are interior angles on SST;
∠ 5 and ∠ 13; ∠ 6 and ∠ 14; ∠ 7 and ∠ 15; ∠ 8 and ∠ 16 are
corresponding angles;
∠ 8 and ∠ 15; ∠ 5 and ∠ 14 are exterior angles on SST;
∠ 7and ∠ 16; ∠ 6 and ∠ 13 are interior angles on SST;
∠ 16 and ∠ 6; ∠ 13 and ∠ 7 are alternate interior angles;
∠ 15 and ∠ 5; ∠ 14 and ∠ 8 are alternate exterior angles;
∠ 15 and ∠ 11; ∠ 14 and ∠ 10; ∠ 16 and ∠ 12; ∠ 13 and ∠ 9 are
corresponding angles;
∠ 14 and ∠ 11; ∠ 13 and ∠ 12 are interior angles on SST;
∠ 15and ∠ 10; ∠ 16 and ∠ 9 are exterior angles on SST;
∠ 4 and ∠ 12; ∠ 13 and ∠ 11 are alternate interior angles;
∠ 15 and ∠ 9; ∠ 16 and ∠ 10 are alternate exterior angles;
∠ 3 and ∠ 11; ∠ 4 and ∠ 12; ∠ 2 and ∠ 10; ∠ 1 and ∠ 9 are
corresponding angles;
∠ 12 and ∠ 2; ∠ 3 and ∠ 9 are alternate interior angles;
∠ 11 and ∠ 1; ∠ 4 and ∠ 10 are alternate exterior angles;

20
∠ 1and ∠ 10; ∠ 4 and ∠ 11 are exterior angles on SST;
∠ 3 and ∠ 12; ∠ 2 and ∠ 9 are interior angles on SST;

3. ∠ 1 and ∠ 9 + ∠ 10; ∠ 4 and ∠ 11 are corresponding angles;


∠ 2 and ∠ 14; ∠ 3 and ∠ 12 + ∠ 13 are corresponding angles;
∠ 4 and ∠ 14; ∠ 3 and ∠ 9 + ∠ 10 are alternate interior angles;
∠ 1 and ∠ 12 + ∠ 13; ∠ 2 and ∠ 11 are alternate exterior angles;
∠ 1 and ∠ 11; ∠ 2 and ∠ 12 + ∠ 13 are exterior angles on SST;
∠ 4 and ∠ 9 + ∠ 10; ∠ 3 and ∠ 14 are interior angles on SST;
∠ 6 and ∠ 10; ∠ 7 and ∠ 11; ∠ 5 and ∠ 9 + ∠ 14; ∠ 8 and ∠ 13 are
corresponding angles;
∠ 7 and ∠ 9 + ∠ 14; ∠ 8 and ∠ 10 are alternate interior angles;
∠ 6 and ∠ 13; ∠ 5 and ∠ 11 + ∠ 12 are alternate exterior angles;
∠ 6and ∠ 11 + ∠ 12; ∠ 5 and ∠ 13 are exterior angles on SST;
∠ 7 and ∠ 10; ∠ 8 and ∠ 9 + ∠ 14 are interior angles on SST;

4. AB // CD with BC as transversal
CD // FE with DE as transversal
DE // FG with FE as transversal

5. a // b, e, c and d are the transversals


6. c // d, a, b and c are the transversals

Lesson 2

1. ∠ 2 = 85o; ∠ 5 = 95o; ∠ 1 = 95o; ∠ 7 = 95o; ∠ 3 = 95o; ∠ 8 = 85o;


∠ 4 = 85o
2. ∠ 12 = 118o; ∠ 9 = 62o; ∠ 11 = 62o; ∠ 16 = 118o; ∠ 13 = 62o;
∠ 14 = 118o; ∠ 15 = 62o
∠ 8 = 85o; ∠ 6 = 85o; ∠ 2 = 85o; ∠ 1 = 95o; ∠ 5 = 95o; ∠ 3 = 95o;
∠ 1 = 95o
3. ∠ 8 = 62o; ∠ 6 = 62o; ∠ 10 = 62o; ∠ 13 = 118o; ∠ 9 = 28o;
∠ 12 = 28o; ∠ 11 = 90o; ∠ 13 = 62o; ∠ 7 = 118o; ∠ 5 = 118o; ∠ 1 = 90o;
∠ 2 = 90o ; ∠ 3 = 90o; ∠ 4 = 90o

21
4. x = 60o
5. x = 140o

What have you learned

1. ∠ 1 and ∠ 5, ∠ 4 and ∠ 8, ∠ 2 and ∠ 6, ∠ 3 and ∠ 7

2. ∠ 4 and ∠ 6, ∠ 3 and ∠ 5

3. ∠ 1 and ∠ 7, ∠ 2 and ∠ 8

4. ∠ 4 and ∠ 5, ∠ 3 and ∠ 6

5. ∠ 1 and ∠ 8, ∠ 2 and ∠ 7

6. ∠ 3, ∠ 5, ∠ 1

7. ∠ 8, ∠ 6, ∠ 2

8. ∠ 8: ∠ 5, ∠ 7, ∠ 1; ∠ 7: ∠ 2, ∠ 6, ∠ 8

9. ∠ 3: ∠ 6, ∠ 4, ∠ 2; ∠ 4: ∠ 5, ∠ 3, ∠ 1

10. ∠ A ≅ ∠ C; ∠ B ≅ ∠ D; ∠ A supplementary ∠B
∠ B supplementary ∠C
∠ C supplementary ∠D
∠ A supplementary ∠D
11. Yes, they are alternate interior angles
12. ∠ 1, ∠ 4
13. Yes, because their sum is equal to 90o
14. x = 65o
15. x = 140o
16. x = 30o
17. x = 36o
18. BA // CF, BC // DE, ∠ ABC ≅ ∠ BCF; ∠ BCD ≅ ∠ CDE
19. ∠ 2 = 45o, ∠ 3 = 135o, ∠ 4 = 45o, ∠ 5 = 135o, ∠ 6 = 45o, ∠ 7 = 135o, ∠ 8
= 45o
20. ∠ 4 = 75o, ∠ 1 = 105o, ∠ 5 = 105o, ∠ 7 = 105o, ∠ 3 = 105o, ∠ 8 = 75o,
∠ 2 = 75o, ∠ 9 = 75o, ∠ 10 = 105o, ∠ 12 = 105o, ∠ 11 = 75o

22
Module 2
GEOMETRY OF SHAPE and SIZE

What this module is about


There are different shapes around us. Everywhere you look, you see
varied geometric figures. Many of these figures are called polygons. This
module will help you be on familiar terms with them. You will also learn to
distinguish types of triangles; knowledge of which will help you communicate
ideas about real life situations.

What you are expected to learn


This module is designed to

1. illustrate different kinds of polygons and identify the parts of a regular


polygon;
2. differentiate convex and non-convex polygons;
3. illustrate a triangle, its basic and secondary parts and
4. classify triangles according to angles and sides.

How much do you know


Select the correct answer for each question:

1. Which of the following is a polygon?

a. b. c. d.

2. A pentagon is a polygon with

a. 4 b. 5 c. 6 d. 7 sides

3. One of the following is NOT a convex polygon. Which one is it?

a. b. c. d.
4. In ∆ABC, AB, AC and BC are called

a. sides b. interior angles c. vertices d. exterior angles

5. Using the figure at the right, BD is a /an B

a. median b. altitude
c. ∠ bisector d. ⊥ bisector
A D C
6. In ∆DEF, m∠E = 120. ∆DEF is

a. acute b. right c. obtuse d. equilateral triangle

7. It is a triangle with all sides congruent

a. scalene b. isosceles c. obtuse d. equilateral

8. The figure below is a regular hexagon. ∠COD is a / an

a. inscribed angle b. central angle


c. obtuse angle d. interior angle C O

9. The longest side in a right triangle is known as D

a. leg b. hypotenuse c. vertex d. base

10. In isosceles triangle ABC, AB = BC. AB and BC are called

a. bases b. vertices c. hypotenuse d. legs

11. A polygon with no given number of sides can be named as

a. dodecagon b. undecagon c. n-gon d. gon

12. The sides of a polygon is made up of

a. segments b. rays c. lines d. planes

13. A line segment from a vertex of a triangle to the midpoint of the opposite
side.

a. altitude b. perpendicular bisector c. median d. angle bisector

14. A right triangle with congruent legs is

a. equilateral b. isosceles c. scalene d. isosceles right triangle

2
15. An equilateral triangle is also isosceles. The statement is

a. always true b. sometimes true c. never true d. can’t be determined

What you will do


Lesson 1

Different Kinds of Polygons and the Parts of a Regular Polygon

The word polygon is from 2 Greek words poly (many) and gon (sides).
The following are polygons:

Each of the figures above is closed, made up of segments and the


segments or sides intersect only at their endpoints.

A polygon is a closed figure made up of segments that intersect at their


endpoints and no two consecutive segments are collinear. Each line segment is
a side of the polygon and each endpoint is a vertex.

The following are NOT polygons:

The figure is made up of segments but it is not closed

The figure is closed but not entirely made up of segments

Curved portion

The figure is closed but it is curved

3
The figure is closed, made up of segments but 2 sides
intersect at another segment.

The intersection should be at the endpoints only.

Look at the figures below. Can you identify the parts/portion that are
polygons?

Polygons have special names depending on their number of sides.

Name of polygon Number of sides


Triangle 3
Quadrilateral 4
Pentagon 5
Hexagon 6
Heptagon 7
Octagon 8
Nonagon 9
Decagon 10

Examples:

Triangle Quadrilateral Pentagon

Hexagon Heptagon Octagon

Nonagon Decagon

4
11 – gon 24 - gon

Polygons with more than 10 sides are often referred to as 11-gon, 12-gon,
13-gon and so on. When the number of sides is not given, the polygon is simply
called n-gon.

Drawing Tip: To draw a polygon easily,

Step 1: Lightly sketch a circle.


Step 2: Place the points you need on the circle.
Step 3: Then connect the points to form your polygon.
Step 4: Erase the circle.

. . . .
. . . .
. . . .
. . . .

Step 1 Step 2 Step 3 Step 4

Regular Polygons:

Polygons with equal sides and equal angles are called regular polygons.
Study the following illustrations:

I II

Equal sides equal sides


and = angles not equal angles

0 0
60 60

III IV

equal angles equal sides


not = sides equal angles

5
Figures I and IV are regular polygons. A regular polygon has the following
parts:
A B
Vertex angle
F o C
Exterior angle
central angle
E D G

ABCDEF is a regular polygon. All sides are equal and all angles have the
same measure.

Vertex Angle

A vertex angle is an angle formed by the intersection of two sides of the


polygon. ∠B is a vertex angle. ∠C is also a vertex angle. Can you name 4
more vertex angles? They are ∠D, ∠E, ∠F, and ∠G.

Central Angle

∠COD is a central angle because the center of the polygon is also the
vertex of the angle. Name some more central angles. There are more than 6.
When you name them, be sure that the middle letter is O.

Exterior Angle

If you extend one side of the polygon, you form an exterior angle. In the
above figure, ∠CDG is an exterior angle. Let’s have some more exterior angles:

1 4 5 8

2 3 6 7

Lesson 2

Differentiate convex and non-convex polygons

Polygons are also classified as convex and non-convex. Let’s find out
how they differ by studying these examples:

6
Convex Polygons:

Non-Convex Polygons:

A polygon is convex if a segment joining any two interior points lies


completely within the polygon.

within outside

convex non-convex

Try this out

A. Which of the following is a polygon?

1 2 3 4 5 6 7

B. Tell whether the polygon is convex or non-convex:

___________ ___________ ___________ ___________ _________

___________ ___________ ___________ ___________ _______

7
C. Name the polygon:

1 2 3 4 5 6 7 8

D. If possible, draw a polygon that fits each description

1. A regular quadrilateral.
2. A non-convex pentagon
3. An equilateral octagon.
4. An equilateral nonagon.
5. A convex 15-gon

Let’s summarize
1. A polygon is a closed figure made up of segments that intersect at their
endpoints and no two consecutive segments are collinear.

2. Each line segment is a side of the polygon and each endpoint is a vertex.

3. Polygons are named based on the number of their sides

4. Polygons with equal sides and equal angles are called regular polygons.

5. A polygon is convex if a segment joining any two interior points lies


completely within the polygon.

Lesson 3

A Triangle. Its Basic and Secondary Parts

You have learned that the triangle is the simplest polygon because it has
the least number of sides.

But how do we name triangles? Triangles are named by using the letters
at their vertices. Starting from any vertex, go clockwise or counterclockwise.

8
Examples:

Clockwise naming of triangle

C B

Triangle ABC is the same as ∆BCA, ∆CAB.

Counterclockwise

C B

∆ACB = ∆CBA = ∆BAC.

The basic parts of a triangle are sides, vertices and angles.

1. The sides of ∆ABC are AB, BC, and AC.

A
Side AC side AB

C B

Side BC

2. The vertices are the endpoints A, B, and C.

A
vertex A

vertex C vertex B

C B

3. The angles. Can you name the 3 angles of the figure below?

9
They are ∠ABC, ∠BCA, and ∠CAB.

A
∠CAB

C B

∠ACB ∠CBA

Name Triangle I and II in different ways and then complete the table below.
P

I II
Q S
R

Name Sides Vertices Angles


Triangle I ∆QPR,__,__ QP,__,__ Q,__.__ ∠PQR,__,__
Triangle II ∆PSR,__,__ PS,__,__ P,__,__ ∠PSR,__,__

Secondary parts of a triangle

a. Median

A median is a segment whose endpoints are a vertex of a triangle and the


midpoint of the opposite side. (A midpoint divides a segment into two equal
segments).

P P P

S T

Q V R Q R Q R

PV is a median of ∆PQR. V is the midpoint of QR.

QS is a median of ∆PQR. S is the midpoint of PR.

RT is a median of ∆ PQR. T is the midpoint of PQ.

10
b. Altitude

An altitude is a segment from a vertex of the triangle perpendicular to the


opposite side or to the line containing the opposite side. (Perpendicular means
the two segments form a right angle).

X U X
Y

U Y V X V U V Y

In all the figures above, XY is an altitude.

c. Angle Bisector

C C

M C M

MC bisects angle M in all the triangles above. MC is called angle bisector.

You will notice that a triangle has 3 medians, 3 altitudes and 3 angle
bisectors.

If the triangle is equilateral, the median is the same as the angle bisector
and altitude. See the figure below:

J K
L

11
Lesson 4

Classification of triangles according to sides and angles

Triangles are classified by their sides into 3 categories:

1. Scalene – all sides have different lengths


2. Isosceles – 2 sides have the same length
3. Equilateral – all 3 sides have the same length

10
7

8
scalene ∆ isosceles ∆ equilateral ∆

Triangles are classified by their angles into 3 categories:

1. Acute ∆ - all 3 angles are acute


2. Right ∆ - one angle is right
3. Obtuse ∆ - one angle is obtuse

All angles are acute

Acute ∆

600

600 600

An acute triangle is equiangular if all 3 angles have the same measure.

Right angle

Right ∆

12
Obtuse angle

Obtuse ∆

Parts of special triangles:

Triangle MON is an isosceles triangle.

Vertex angle
leg

base angle
O N
Base

Legs – the congruent sides


Base – the third side
Vertex angle – the angle opposite the base
Base angles – the angles at the base

Triangle ACB is a right triangle


A
Hypotenuse
legs

C B

Hypotenuse – the longest side; the side opposite the right angle
Legs – the sides forming right angle

Try this out

A. Using the figure at the right, identify the following.


A
1. AB, AC, BC
2. C, B, A
3. AE F D
4. BD
5. CF
B C
E

13
B. Match each triangle with all words that describe it:

a. scalene b. isosceles c. equilateral


d. acute e. right f. obtuse g. equiangular

(1) (2) (3)

600 6 1000 8

600 600 500 300


9
(4) (5)

C. Tell if each statement is true or false. Draw a figure to justify your answer.

1. A triangle can be isosceles and acute.

2. A triangle can have two obtuse angles.

3. A triangle can be obtuse and scalene.

4. A right triangle can be equilateral.

5. A triangle can be right and isosceles.

6. A triangle can have two right angles.

7. A triangle can have at most 3 acute angles.

8. A triangle can have at least one (1) acute angle.

9. An equilateral triangle is also an acute triangle.

10. An equiangular triangle can never be a right triangle.

Tessellations

A tessellation is a design in which congruent copies of a figure are arranged


to fill the plane in such a way that no figures overlap and there are no gaps.

14
Examples:

Try to make a tessellation for each figure below. Which figure could not
be used for tessellation?

You can also combine figures to make a tessellation:

To make an original pattern for a tessellation, start with a parallelogram. Make


congruent changes on one or both pairs of sides:

15
Let’s summarize
1. The basic parts of a triangle are sides, angles and vertices while the
secondary parts are median, altitude and angle bisector.

2. A median is a segment whose endpoints are a vertex of a triangle and the


midpoint of the opposite side.

3. An altitude is a segment from a vertex of the triangle perpendicular to the


opposite side or to the line containing the opposite side.

4. Triangles are classified according to sides and angles.


a. Scalene – all sides have different lengths
b. Isosceles – 2 sides have the same length
c. Equilateral – all 3 sides have the same length
d. Acute ∆ - all 3 angles are acute
e. Right ∆ - one angle is right
f. Obtuse ∆ - one angle is obtuse

What have you learned


Select the correct answer for each question:

1. Which of the following is a polygon?

a b c d

2. A heptagon is a polygon with

a. 4 b. 5 c. 6 d. 7 sides

3. One of the following is NOT a convex polygon. Which one is it?

a b c d

4. In ∆ABC, A, C and B are called

a. sides b. interior angles c. vertices d. exterior angles

16
5. Using the figure at the right, BD is a /an B

a. median b. altitude
c. ∠ bisector d. ⊥ bisector A D C

6. In ∆DEF, m∠E = 90. ∆DEF is

a. acute b. right c. obtuse d. equilateral triangle

7. It is a triangle with all angles congruent

a. scalene b. isosceles c. obtuse d. equiangular

8. ∠CBA is a
B A
a. inscribed angle b. central angle C O
c. obtuse angle d. vertex angle
D
9. The longest side in a right triangle is known as

a. leg b. hypotenuse c. vertex d. base

10. In isosceles triangle ABC, AB = BC. AB and BC are called

a. bases b. vertices c. hypotenuse d. legs

11. A polygon with no given number of sides can be named as

a. dodecagon b. undecagon c. n-gon d. gon

12. The sides of a polygon is made up of

a. segments b. rays c. lines d. planes

13. A line segment from a vertex of a triangle to the midpoint of the opposite
side.

a. altitude b. perpendicular bisector c. median d. angle bisector

14. A right triangle with congruent legs is

a. equilateral b. isosceles c. scalene d. isosceles right triangle

15. An isosceles triangle is also equilateral. The statement is

a. always true b. sometimes true c. never true d. can’t be determined

17
Answer Key
How much do you know

1. C
2. B
3. A
4. A
5. B
6. C
7. D
8. B
9. B
10. D
11. C
12. A
13. C
14. D
15. A

Try this out

Lesson 2

A.
1. Polygon
2. not
3. not
4. not
5. Polygon
6. Polygon
7. not

B.
1. non-convex
2. convex
3. non-convex
4. convex
5. non-convex
6. non-convex
7. non-convex
8. convex
9. convex
10. non-convex

18
C.
1. Nonagon
2. Heptagon
3. Quadrilateral
4. Octagon
5. Heptagon
6. Pentagon
7. Hexagon
8. Quadrilateral

D.
1. Draw a square

2. Any 5-sided polygon that is not convex

3.

4. – 5. Refer to the drawing tip

Try this out

Lesson 4

A.
1. Sides
2. Vertex / vertices
3. altitude
4. angle bisector
5. median

B.
1. isosceles right triangle
2. scalene right triangle
3. isosceles acute triangle
4. equilateral / equiangular
5. scalene obtuse triangle

C.
1. true
2. false

19
3. true
4. false
5. true
6. false
7. true
8. false
9. true
10. true

What have you learned

1. B
2. D
3. C
4. C
5. B
6. B
7. D
8. D
9. B
10. D
11. C
12. A
13. C
14. D
15. C

20
Module 1
Circles

What this module is about


This module will discuss in detail the characteristics of a circle as well as the
segments and lines associated with it. Here, you will gain deeper understanding of the
angles formed in circles, how to get their measures and how they are related to one
another. Furthermore, this module will also give meaning to the circle being composed of
arcs and how each arc is related to the angles formed in circles.

What you are expected to learn


This module is written for you to

1. define a circle.
2. define and show examples of the lines and segments associated with circles.
3. describe the relationship of lines and segments that are peculiar to circles.
4. define, identify and give examples of the kinds of arcs that compose a circle.
5. identify central angle and inscribed angle.
6. discover the relationship between the measures of central angle and inscribed angle
and their intercepted arcs.

How much do you know


Answer the following as indicated. S

1. Given a circle with center O. Name the following :


T
a. the circle
M
b. a diameter
c. two radii · OO
N
d. two chords which are not diameters
e. a secant
R
f. a tangent

2. If a radius is perpendicular to a chord then it ________ the chord.


P
3. In the given circle A, PT is a diameter, therefore A
MT is a ________ and ●
4. PTM is a _________. M●
T

5. Radius AB ⊥ CE. If CE = 8 cm, then


CX = ________. A
E
6. Using the same figure, if AX = 3 cm, X
What is the length of radius AC? C
B
B

7. In circle O, m∠BOC = 93 . What is mBC? A


O
8. What is m∠BAC ?

C
P R
9. In the figure, PR ║ ST . Using the given
Find mPT and m∠RPS .

T S
10. A quadrilateral PQRS is inscribed in a circle.
If m∠P = 103 , what is m∠R ?

What you will do


Lesson 1

Identifying a circle, the lines, segments and angles associated with it.

A circle is defined as the set of all points that are at the same distance from a given
point in the plane. The fixed given point is called the center. The circle is named after its
center. Hence in the figure, given is a circle O. A
The set of points on the plane containing the circle is
divided into 3, (1) the circle, (2) the set of points outside O
the circle and (3) the set of points inside the circle.
OC , OB and OA are segment whose endpoints are C
D
the center of the circle and a point on the circle.
These three segments are called radii of the circle. B

Radius of a circle is a segment whose endpoints are the center and a point on the
circle. In the figure, AD is a segment whose endpoints are points on the circle. AD is
called chord of the circle. AB is a segment whose endpoints are points on the circle and it

2
passes through the center. AB is called diameter of a circle. Diameter of a circle is a chord
that passes through the center.

Lines on the plane containing the circle may intersect the circle at one point or at two
points or not at all.
b

O· O R O
· ·
X
S
a c

Fig. 1. line a does not Fig. 2. line b intersect Fig. 3 line c intersect circle
intersect circle O. circle O at point X at two points R and S.

In figure 2, line b is tangent to the circle, and in figure 3, line c is a secant. Hence, we can
use the following definitions.

Tangent is a line that intersect a circle at one points. Secant is a line that intersect a circle
at two points.

Some theorems in circle show relationship between chord and radius. One of them
is this theorem:

Theorem: If a radius is perpendicular to a chord, then it bisects the chord.

Proof: Consider the given circle. If radius OA ⊥ BC at D, then OA


bisects BC or BD = DC. One way of proving segments or angles o
congruent is by showing that they are corresponding parts of ·
B C
congruent triangles. Here, we must prove that BD and DC are D
corresponding sides of congruent triangles. If O and B are
joined and O and C are also joined, we have ∆OBD and ∆OCD. A
Both of these triangles are right since OA ⊥ BC and thus ∠ODB and
∠ODC are both right angles. Since OB and OC are both radii of the same circle,
hence they are congruent. And finally OD ≅ OB by reflexive property. Therefore, by
the HyL Congruency for right triangles, ∆OBD ≅ ∆OCD . Since the two triangles are
congruent, then the remaining corresponding parts such as BD and DC are also
congruent.

We have just proven the theorem here, only this time, instead of using the two
column form we use the paragraph form.

3
Our conclusion therefore is that a radius that is perpendicular to a chord bisects the
chord. The most important considerations here were the perpendicularity and the word to
bisect.

Examples:
1. OB ⊥ DE at T, DT = 3x -7 , TE = x + 15

Solution:
Since OB ⊥ DE , then DT = TE
Hence,
3x – 7 = x + 15
2x = 15 + 7 O
2x = 22
E
x = 11 D T
T
Substituting the value of x, we get
B
DT = 3(11) – 7 = 33 – 7
= 26
TE = 11 + 15 = 26
DE = DT + TE
DE = 26 + 26 = 52

There are other theorems whose main idea is taken from the previously proven
theorem. The next theorem serves as the converse of the first theorem and it states that: If
a radius of a circle bisects a chord that is not a diameter, then it is perpendicular to the
chord.

If the previous theorem was proven using the HyL congruence for right triangle, the
converse is proven using the reverse process, that is two angles must be proven part of
congruent triangles and they are congruent and supplementary.

You can prove the theorem as part of your exercise. Examples on how to use these
two theorems are given below.

2. Given: AB bisects chord CD at E.


CD = 6, AE = 4
Find the length of the radius of the circle.

Solution: Based on the theorem, AB ⊥ CD , thus A


∆ACE ≅ ∆ADE and both are right triangles.
By the Pythagorean theorem, we can solve C E D
for the length of the radius.
B

In ∆ACE, AC 2 = AE 2 + CE 2
But CE = ½ CD so

4
CE = ½ (6)
CE = 3

AC 2 = AE 2 + CE 2
AC 2 = 42 + 32
AC 2 = 16 + 9 = 25
AC = 25
AC = 5

Lesson on circle is very rich with theorems and definitions, principles and postulates.
Some of those theorems and definitions will be introduced as we plod along with this
module.

Definitions:

• Congruent circles are circles that have congruent radii.

• Concentric circles are coplanar circles having the same center

Illustrations:
a) b)

A B
·

X Y

Circle A is congruent to circle B if and These two circles are


only if AX ≅ BY concentric circles

Theorem:

If chords of a circle or of congruent circles are equidistant from the center(s), then the
chords are congruent

Illustration of the theorem.


O
Circle O ≅ circle P O P
OX = PY C D
A B Y
Then, AB ≅ CD X
X
N M

5
Try this out

A. Using the given figure, name A


B

1. the circle
2. 2 diameters O
3. 2 chords which are not diameters F
4. 2 secants E
5. a tangent C
D

B. Given: AB ⊥ CD at E

CD is 10 cm long. How far is CD


from the center if the length of the
A
radius is
1. 13 cm 5. 12 cm
2. 7 cm 6. 10 cm C D
E
3. 14 cm 7. 5 2 cm
B
4. 8 cm 8. 3 6 cm

C. Given: CD is 20 cm long. How long is the radius of the


circle if the distance of CD from the center is

1. 7 cm 3. 13 cm
2. 10 cm 4. 8 cm
5. 5 cm 7. 5 5 cm
6. 21 cm 8. 4 6 cm

D. AC is 12 cm long. How long is chord CD if its distance from the center is

1. 10 cm 5. 9 cm
2. 6 cm 6. 23 cm
3. 8 cm 7. 2 11 cm
4. 5 cm 8. 4 5 cm

E. Solve the following problems.


1. ON ⊥ MP O
ME = 7x + 5
M P
PE = 4x – 20 E
Solve for ME , PE and MP N

6
2. In a circle are two chords whose lengths are 10 cm and 24 cm respectively. If the
radius of the circle is 13 cm, what is the maximum distance of the two chords?
What is their minimum distance?

Lesson 2

Arcs and Central Angles

A•
A part of a circle between any two points is an arc. In the •B
figure, the set of points from A to B is an arc. A circle is in itself

an arc. Arc of a circle is measured in terms of degrees.

The whole arc making up the circle measures 360°.

Any arc of a circle can belong to any of these three groups.


a. minor arc – an arc whose measure is between 0 and 180°.
b. semicircle – an arc whose measure is exactly 180°
c. major arc – an arc whose measure is between 180° and 360°
A
In the given figure, AB is a diameter, hence AB represents
a semicircle, AC is minor arc and ABC is a major arc.
Aside from AC, another minor arc in the figure is BC.
O● C
ACB also represents a semicircle.

Angles in a circle are formed by radii, chords, secants and


tangents. Determination of the measures of the angles formed by B
these lines depends upon the measure of the intercepted arcs
of the given angles.

Examples: A

B
In circle some angles formed by chords and
radii are shown. Each of the angles intercepts O
an arc defined by the endpoints contained on the
sides of the angle. C
E

∠ AEB intercepts AB. D


∠ BOC intercepts BC
∠ COD intercepts CD
∠ EOD intercepts ED
∠ AEB intercepts AB.
∠ AEB intercepts AB.
∠ AEB intercepts AB.

7
At this point we will discuss in detail the kinds of angles formed in a circle, their
characteristics and how to get their measures from the measures of the intercepted arcs.
We will start with the angle formed by two radii. A

Central angle is an angle formed by two radii B


and the vertex is the center of the circle. In the
figure, ∠ AOB, ∠ BOD and ∠ DOC are all O
examples of central angles. Each of these angles D
has its own intercepted arc. ∠ AOB intercepts AB,
∠ BOD intercepts BD and ∠ DOC intercepts DC.
C
The measure of a central angle is numerically B
equal to its intercepted arc.

In the figure, ∠ BAC is a central angle and A 83°


∠ BAC intercepts BC. Since mBC = 83, then D

m ∠ BAC = 83, mBDC = 277°. C

In the study of geometry, every new topic or concept is always associated with study
of postulates, theorems and definitions. In the study of arcs and angles in a circle, we will
discuss many theorems that will help us solve problems involving the said concepts. We
will start with the simplest postulate in the chapter.

Like any measure, measure of an arc is also a unique real number and as such, we
can perform the four fundamental operations on those measure. So the first postulate is the
Arc Addition Postulate: The measure of an arc formed by two adjacent non-overlapping
arcs is the sum of the measures of the two arcs. B

A● •C
In the given circle, m AC = m AB + m BC
·
Examples: D
·
1. DG is a diameter. Find the measure of the · E
following arcs. 60°
O●
DG, DE, DF, GE, DGF · F
70°
Solution: ·G
Since DG is a diameter, then DG is a semicircle.

Therefore,
m DG = 180
m DE = 180 – (60 + 70)
= 180 – 130
= 50

8
m DF = m DE + m EF
= 50 + 60
= 110

m GE = m GF + m FE
= 70 + 60
= 130

m DGF = m DG + m GF
= 180 + 70
= 250
Definitions:

In the same circle or in congruent circles, arcs which have the same measure are
congruent.
D
60°
Example: 1. In the figure, m DC = 60, m BC = 60
m AB = 60 . ●C
Therefore, DC ≅ BC ≅ AB 60°

2. Since every semicircle measures 180°, ●B


then all semicircles are congruent. 60°
A
Theorem:

If two minor arcs of a circle or of congruent circles are congruent, then the
corresponding chords are congruent. B

Examples: 1. Given: AB ≅ BC A
Since AB subtends AB and · C
BC subtends BC then
AB ≅ BC A
X

2. Circle O ≅ circle M M•
O• B
If AB ≅ XY, then AB ≅ XY
Y
Theorem:

If two chords of a circle or of congruent circles are congruent, then the corresponding
minor arcs are congruent.

This is the converse of the previous theorem. Basically if you prove these two
theorems, the steps will be just the reverse of the other. Instead of proving them, showing
examples will be more beneficial to you.

9
P
R
In circle A, if RS ≅ PQ
then RS ≅ PQ A •

Q
Theorem: S

If two central angles of a circle or of congruent circles are congruent, then the
corresponding minor arcs are congruent. M
P
Example: In circle O, ∠ MNO ≅ ∠ BOA
O
Therefore, MP ≅ AB
B
A
Theorem:

If two minor arcs of a circle or of congruent circles are congruent, then the
corresponding central angles are congruent.
E
Example:
A
In circle A, BC ≅ DE B
Therefore ∠BAC ≅ ∠DAE ` D

C
Theorem:

If two central angles of a circle or of congruent circles are congruent, then the
corresponding chords are congruent.
B

Given: In circle O, ∠XOY ≅ ∠AOB O


X
Prove: XY ≅ AB
A

Y
Proof
Statements Reasons
1. In circle O, ∠XOY ≅ ∠AOB 1. Given
2. OX ≅ OB , OY ≅ OA 2. Radii of the same or congruent circles are
congruent
3. ∆XOY ≅ ∆BOA 3. SAS congruency Postulate
4. Corresponding parts of congruent
4. XY ≅ AB
triangles are congruent..

10
Theorem:

If two chords of a circle or of congruent circles are congruent circles are congruent,
then the corresponding central angles are congruent. S

Given: In circle A, PR ≅ ST A
Prove: ∠PAR ≅ ∠SAT T

Proof: R

Statements Reasons
1. In circle A, PR ≅ ST 1. Given
2. Radii of the same circle are
2. AP ≅ AS
congruent.
AR ≅ AT
3. ∆PAR ≅ ∆SAT 3. SSS Congruency Postulate
4. ∠PAR ≅ ∠SAT 4. Corresponding parts of congruent
triangles are congruent

Examples:
A
Given: AB and CD are diameters of circle E. C
1. What is true about ∠AED and ∠BEC ? Why?
2. What kind of angles are they?
E
3. Give as many conclusions as you can D
based on the previously discussed theorems.
B
Answers:

1. ∠AED ≅ ∠BEC . They are vertical angles and vertical angles are congruent.
2. In the circle they are central angles. Central angles are angles whose vertex is the
center of the circle.
3. a. AD ≅ BC. If two central angles of a circle or of congruent circles are congruent,
then the corresponding arcs are congruent.
b. AD ≅ BC

Likewise
1. ∠AEC ≅ ∠BED
2. AC ≅ DB
3. AC ≅ DB

Try this out

A. AB is a diameter of circle O.
m∠AOE = 82 .

11
A

Find the measures of: E

1. AB 4. ABE O
2. AE 5. BAE
3. BE

B
B. GE and FD are diameters of circle A. If DA = 73º, find the measures of
D
1. ∠ DAE 5. GF
2. ∠ GAF 6. DG
3. ∠ EAF 7. FDE E
4. ∠ DAG A

C. Given circle A. If m BTY = 116, B T F


and m = 3n, find
n m
1. m 5. m ∠ BAT
2. n 6. m ∠ TAY Y
A
3. BT 7. m ∠ BAY
4. TY
A

D. Given circle O. AB ≅ BC. If mAB = 56, what is


m∠AOB ? What is mABC ? O
Which chords are congruent?
B

C
A

E. A. B and C are three points on the circle.


IF AC ≅ AB ≅ BC, what is the measure of each arc?
What is true about the chords AC , AB, and BC ? ·
If ABC is 16 more than three times AC, C
B
find mAC, mABC.

F. P, Q and R are three points on a circle. If the


ratio PQ:QR:PR = 3:4:5, find the measures of
PR, QR and PS.

12
P
G. Using the figure and the given in it, find the Q
measures of:
1. PQ 5. ∠POQ x
2. QR 6. ∠ QOR 2x + 13
O
3. SR 7. ∠ SOR 2x 4x + 5
4. PS 8. ∠ POS S

R
B
H. BD and EC are diameters of circle A.
If m∠C = 35 , find the measures of

1. ∠B 5. ∠ EAD E A 35º
2. ∠E 6. BC C
3. ∠D 7. CD
4. ∠ BAC

D
Lesson 3

Arcs and Inscribed Angles

Another angle in a circle that is very important in the study of circle is the inscribed
angle.

Definition:

An inscribed angle is an angle whose vertex lies on the circle and the sides contain
chords of the circle.
B
D
P

F
T ●
● C

A
S
E

Fig. 1 Fig. 2 Fig 3

Each of the angle shown above is an example of an inscribed angle. Three cases
are represented here relative to the position of the sides in relation to the center of the
circle.

13
Case 1. the center of the circle is on one side of the inscribed angle.
Case 2, the center of the circle is in the interior of the inscribed angle.
Case 3, the center of the circle is on the exterior of the inscribed angle.

In the study of the angles in a circle and in determining their measures, it is important
to determine the intercepted arc(s) of the given angle. To understand better, let us see
some examples.

In the figure, the arc in the interior Intercepted arc C


of the angle is the intercepted arc of
the angle.

The intercepted arc of ∠BAC is
B A
the minor arc AC.

In the given examples of inscribed angles above the following holds:

a) In figure 1, ∠ DEF is an inscribed angle


∠ DEF intercepts arc DF

b) In figure 2, ∠ PST is an inscribed angle,


∠ PST intercepts arc PT

c) In figure 3, ∠ BAC is an inscribed angle


∠ BAC intercepts arc BC

Every angle whether in a circle on in any plane is associated with a unique number
defined as its measure. If the measure of a central angle is equal to the measure of its
intercepted arc, the next theorem will tell us how to find the measure of the inscribed angle.

Theorem: Inscribed angle Theorem

The measure of an inscribed angle is equal to one half the measure of its intercepted
arc.
D

It means that in the given figure,


F

m∠DEF = 12 mDF O●

14
Since there are three cases by which an inscribed angle can be drawn in a circle, then we
have to prove each of those cases.
D

Case 1 (One side of the angle is the diameter of the circle) F


1 y
Given: Circle O with inscribed angle ∠DEF O●
Use the notation in the figure for
clarity O
x

1
Prove: m∠DEF = (mDF ) E
2

Proof:
Statements Reasons
1. Circle O with inscribed angle ∠DEF 1. Given
2. Draw OF to form ∆FOE 2. Line determination postulate
3. ∠ 1 is an exterior angle of ∆FOE 3. Definition of exterior angle
4. m ∠ 1 = x + y 4. Exterior angle theorem
5. Radii of the same circle are congruent
5. OF ≅ OE
6. ∆FOE is an isosceles triangle 6. Definition of isosceles triangle
7. x = y 7. Base angles of isosceles triangle are
congruent
8. m ∠ 1 = x + x = 2x 8. Substitution (Steps 4 and 7)
9. 2x = m ∠ 1, x = ½ m ∠ 1 9. Multiplication property of equality
10. But ∠ 1 is a central angle 10. Definition of central angle
11. m ∠ 1 = m DF 11. Measure of a central angle equals its
1 intercepted arc.
12. x = m∠DEF = (mDF ) 12. Substitution (Steps 9 and 11)
2

So, we have proven case 1. Let us now prove case 2 of the inscribed angle theorem.

Case 2. (The center of the circles lies in the interior of the inscribed angle)
P
Given : Circle O with inscribed ∠PQR

1 S
Prove: m ∠PQR = m PR a ●
2 O
Q b

15
Proof:
Statements Reasons
1. Circle O with inscribed ∠PQR . Use the 1. Given
given notation in the figure.
2. Draw diameter QS 2. Line determination Postulate
3. Angle Addition Postulate
3. m ∠PQR = a + b
4. a = 12 mPS 4. Inscribed angle theorem (Case 1)
b = 12 mSR
5. a + b = 12 mPS + 12 mSR = 12 (mPS + mSR) 5. Addition Property of Equality
6. mPR = mPS + mPR 6. Arc Addition Postulate
1
7. m ∠PQR` = (mPS + mPR) 7. Transitive Property of Equality
2
8. m ∠PQR` = 2 mPR
1 8. Transitive Property of Equality

Case 3. (The center is in the exterior of the inscribed angle)

Given: ∠BAC is an inscribed angle in circle O O


Use the additional notation in the figure A ● D
a
x
Prove: m∠BAC = 12 mBC
B

Proof:
Statements Reasons
1. Draw diameter AD 1. Line determination Postulate
2. m∠DAC = m∠DAB + m∠BAC 2. Angle Addition Postulate
3. m∠BAC = m∠DAC − m∠DAB 3. Subtraction Property of Equality
4. m∠DAC = 12 mDC 4. Inscribed angle Theorem (Case 1)
m∠DAB = 12 mDB
5. m∠BAC = 12 mDC − 12 mDB = 12 (mDC-mDB) 5. Substitution
6. mDC = mDB + mBC 6. Arc Addition Postulate
7. mBC = mDC – mDB 7. Subtraction Property of Equality
8. m∠BAC = 12 mBC 8. Substitution

From the proofs that were given, we can therefore conclude that wherever in the
circle the inscribed angle is located, it is always true that its measure is one-half its
intercepted arc.

16
Examples. Use the figure at the right.
B
1. Given: circle O. m∠BOD = 80
O
Find: mBD, m∠BAD
A D
Solution:

Since m∠BOD = 80 , then

a. mBD = 80

b. m∠BAD = 1
2
BD

= 1
2
(80)
= 40

2. Given: circle O. m∠BAD = 37

Find: mBD , m∠BOD

Solution:
m∠BAD = 37 = 12 mBD
mBD = 2(37) = 74
m∠BOD = mBD
m∠BOD = 74

Like in the study of central angles and its measure, discussing inscribed angles and
its measure also involves many theorems. Each previous theorem studied is always a tool
in proving the next theorem.

The following theorem is one of the most useful theorem in solving problems which
involve inscribed angles.
A
Theorem: Angle in a semicircle theorem.

An angle inscribed in a semicircle is a right angle.


B ● C
Given: Circle O. BAC is a semicircle. O

Prove: ∠BAC is a right angle. ( m∠BAC = 90

17
Proof:
Statements Reasons
1. Draw BC passing through center O. 1. Definition of diameter
2. ∠ ABC, ∠ ACB, and ∠ BAC are all inscribed 2. Definition of inscribed angles
angles.
3. m∠ABC = 12 AC , m∠ACB = 12 AB 3. Inscribed Angle Theorem
4. mBAC = mAC + mAB 4. Arc Addition Postulate
5. BAC is a semicircle 5. Given
6. The measure of a semicircle is 180
6. mBAC = 180
7. Transitive Property of Equality
7. mAC + mAB = 180
8. m∠ABC + m∠ACB = 12 AC + 12 AB = 12 ( AC + AB) 8. Addition Property of Equality (Step 3)
9. m∠ABC + m∠ACB = 12 (180) = 90 9. Substitution (Steps 7 and 8)
10. m∠ABC + m∠ACB + m∠BAC = 180 10. The sum of the angles of a triangle
m∠ABC + m∠ACB = 90 is 180.
11. m∠BAC = 90 11. Subtraction Property of Equality
(Step 10 – step 9)
12. ∠BAC is right angle 12. Definition of a right angle

From this point onward, you can use this very important theorem in proving or in
exercises.

There are other theorems on inscribed angle that are also important as the previous
theorem. Of those theorems, we will prove two and the rest, you can answer as exercises.

Theorem:
M
Inscribed angles subtended by the same arc are congruent.
T
Given: Circle O. MN subtends both ∠ T and ∠ P ●O
∠ T and ∠ P are inscribed angles
P N
Prove: ∠ T ≅ ∠ P

Proof:
Statements Reasons

1. In circle O, MN subtends both ∠ T and 1. Given


∠ P. ∠ T and ∠ P are inscribed angles.
2. m∠T = 12 mMN 2. Inscribed Angle Theorem
m∠P = 12 mMN
3. m∠T = m∠P 3. Transitive Property of Equality
4. ∠T ≅ ∠P 4. Definition of congruent angles

18
The next theorem is about polygon inscribed in a circle.

Definition:

A polygon inscribed in a circle is polygon whose vertices lie on the circle.

Examples: The figures below show examples of inscribed polygon.

● ● ●

Inscribed triangle Inscribed Inscribed Inscribed


Quadrilateral Pentagon Hexagon

Theorem: Opposite angles of a inscribed quadrilateral are supplementary.


R

Given: Circle A. PRST is an inscribed quadrilateral. S


A
P ●
Prove: ∠ P and ∠ S are supplementary
∠ R and ∠ T are supplementary

T
Proof:
Statements Reasons
1. Circle A. PRST is an inscribed quadrilateral. 1. Given
2. m∠ P = ½ mRST 2. Inscribed angle theorem
m∠ S = ½ mRPT
m∠ R = ½ mPTS
m∠ T = ½ mPRS
3. m∠ P + m∠ S = ½ mRST + ½ mRPT 3. Addition property of equality
4. m∠ P + m∠ S = ½( mRST + mRPT) 4. Factoring
5. mRST + mRPT = 360 5. The arc of the whole circle is 360º
6. m∠ P + m∠ S = ½(360) 6. Substitution (Steps 4 and 5)
7. m∠ P + m∠ S = 180 7. Algebraic process (step 6)
8. ∠ P and ∠ S are supplementary 8. Definition of supplementary angles
9. m∠ R + m∠ S = ½ mPTS + ½ mPRS 9. Addition property of equality
10. m∠ R + m∠ S = ½ (mPTS + mPRS) 10. Factoring
11. mPTS + mPRS = 360 11. The arc of the whole circle is 360º
12. m∠ R + m∠ S = ½(360) 12. Substitution (Steps 4 and 5)
13. m∠ R + m∠ S = 180 13. Algebraic process (step 6)
14. ∠ R and ∠ T are supplementary 14. Definition of supplementary angles

19
Examples:

1. Given: XY is a diameter.
a. What kind of angle is ∠ Z?
b. If m∠ X = 35, what is m∠ Y? A
c. If m∠ Y = 73, what is mXZ? What is mYZ? X ● Y

Answers:
a. Since XY is a diameter, then XZY is Z
a semicircle and ∠ Z is inscribed in a semicircle.
Therefore, ∠ Z is a right angle.

b. m ∠ X + m ∠ Y = 90.
m ∠ Y = 90 - m ∠ X
m ∠ Y = 90 – 35
m ∠ Y = 65

c. ∠ Y intercepts XZ.
m XZ = 2(75) = 150
m YZ = 180 – 150
m YZ = 30

2. MNOP is inscribed in circle E. If m ∠M = 94,


what is m ∠ O?
N
Answer: O
∠ M and ∠ O are supplementary.
m ∠ M +m ∠ O = 180 E
m ∠ O = 180 - m ∠ M M ●
= 180 – 94
= 86
P
3. Given: Circle O. AB is a diameter
m∠ 1 = 36 and m∠ 3 = 61. C
Find: m∠ 2, m∠ 4, m∠ CBD,
m∠ ADB, m∠ ACB, mCBD, O
m∠ CAD, mAD A ● 1 B
3 2

Solution: 4
m∠ 1 = 36, mAC = 2(36) = 72 D
m∠ 3 = 61, mBD = 2(61) = 122
m∠ 2 = ½ AD
mAD = 180 – BD
= 180 – 122 = 58
m∠ 2 = ½ (58)
= 29

20
m∠ 4 = ½ CB
mCB = 180 – mAC
= 180 – 72
= 108
m∠ 4 = ½ (108)
= 54

m∠ CBD = ½(m AC + mAD)


= ½(72 + 58)
= ½(130)
= 65

m∠ ADB = 90 (Angle in a semicircle)


m∠ ACB = 90 (angle in a semicircle)

mCBD = mCB + mBD


= 108 + 122
= 230

m∠ CAD = ½(mCBD)
= ½ (230)
= 115

Try this out A

A. Given: AB is a diameter of circle O. C


mAC = 79 .
Find: O

1. m∠ AOC
2. m∠ ABC
3. m∠ COB B

B. Given: Circle A., XY and BE are diameters


m∠ XAE = 104. B
Find:
X
4. m XE
5. m BX A
6. m∠ E
7. m∠ B Y
8. m∠ BXY
9. m∠ YXE E

21
C. Using the given figure, find: M Q

10. x
11. m∠ MNQ O
12. m∠ MOQ 3x
13. m∠ POQ
14. m∠ M
15. m∠ MON N P

D. BD is a diameter of circle A.
If m BC = 78, and m DE = 132,
find: C
7
16. m CD 23 m∠ 6 8
17. m BE 24. m∠ 7 1 9 10 ● 6
B D
18. m∠ 1 25. m∠ 8 2 A 5
19. m∠ 2 26. m∠ 9
3 4
20. m∠ 3 27. m∠ 10
21. m∠ 4 E
22. m∠ 5
P

E. PRST is inscribed in circle A.


If m∠ T = (5x – 4)º and m∠ R = (4x + 13)º A R
find: T ●

28. x
29. m∠ T
30. m∠ R S

F. ∆XYZ is inscribed in the circle.


If XY ≅ XZ , prove that m∠ X = b - a

a b
X
Z

22
Let’s Summarize
1. A circle is the set of all points that are at the same distance from a given point in the
plane.

2. Some of the lines associated with circle are the following:

a. Radius
b. Chord
c. Diameter
d. Secant
e. Tangent

3. If a radius is perpendicular to a chord, then it bisects the chord.

4. If a radius of a circle bisects a chord that is not a diameter, then it is perpendicular to


the chord.

5. Congruent circles are circles that have congruent radii. Concentric circles are circles
having the same center.

6. A circle is made up of arcs classified as minor arc, semicircle and major arc.

7. A central angle is an angle on the circle whose vertex is the center of the circle.

8. The measure of the central is numerically equal to its intercepted arc.

9. If two minor arcs of a circle or of congruent circle are congruent, then,

a. the corresponding central angles are congruent,


b. the corresponding subtended chords are congruent

10. An inscribed angle is an angle on the circle whose vertex is a point on the circle.

11. The measure of an inscribed angle is equal to one-half its intercepted arc.

12. An angle inscribed in a semicircle is a right angle.

13. Inscribed angle subtended by the same arc are congruent.

14. The opposite angles of an inscribed quadrilateral are supplementary.

23
What have you learned
Answer as indicated.

1. If the diameter of a circle is 15 cm, what is the length of the radius?


2. A line that intersects a circle at one point is called _________.
3. If a radius bisects a chord which is not a diameter, then its is _________ to the
chord. D

4. CD is a diameter of circle A. CED is a ______________.


5. CE is a _____________. A●
6. CDE is a ______________ E

7. ON ⊥ PT at E. If OE = 6 cm, and the radius


of the circle is 10 cm, what is the length of PT ?. O

P E T

N
8. AC is a diameter of circle O. Using the given in the
given figure, find A
a. m∠ A
b. m∠ C A●
c. m AB
d. m BC
C B
P
9. PT is a diameter of circle Q. Find
a. m∠ PQR
b. m∠ RQT 2x
Q R
3x

T A
10. ∆ABC is inscribed in circle O.
If the ratio of m∠ A: m∠ B: m∠ C =
2:3:5, Find ●O
a. m BC C
b. m AC
c. m AB
B

24
Answer Key
How much do you know

1. a. circle O
b. MN
c. MO , ON
d. MT , MR
e. MR
f. MS
2. bisects
3. minor arc
4. major arc
5. 4 cm
6. 5 cm
7. 93°
8. 46.5°
9. 98°, 49°
10. 77°

Try this out


Lesson 1

A. 1. circle O
2. AC , BD
3. AD, BC
4. EC , BC
5. CF

B. 1. 12 cm 5. 119 cm
2. 2 6 cm 6. 5 3 cm
3. 171 cm 7. 5 cm
4. 43 cm 8. 29 cm

C. 1. 149 cm 5. 5 5 cm
2. 10 2 cm 6. 11 cm
3. 269 cm 7. 15 cm
4. 2 41 cm 8. 14 cm

25
D. 1. 4 11 cm 5. 6 7 cm
2. 12 3 cm 6. 22 cm
3. 8 5 cm 7. 20 cm
4. 2 19 cm 8. 16

Problem Solving:
1. ME = 52, PE = 52, MP = 104
2. maximum distance is 12 cm
minimum distance is 2 cm

Lesson 2

A. 1. 180º 3. 98º
2. 82º 4. 278º
5. 262º

B. 1. 73º 5. 73º
2. 73º 6. 107º
3. 107º 7. 253º
4. 107º

C. 1. 29º 5. 29º
2. 87º 6. 87º
3. 29º 7. 116º
4. 87º

D. 1. 56
2. 56
3. AB and CD

E. 1. Each arc measures 120


2. AC = 86º
3. ABC = 274º

F. PQ = 90
QR = 120
PR = 150

G. 1. 38º 5. 38º
2. 157º 6. 157º
3. 76º 7. 76º
4. 89º 8. 89º

26
H. 1. 35º 5. 110º
2. 35º 6. 110º
3. 35º 7. 17º
4. 110º

Lesson 3
A. 1. 79 3. 101
2. 39.5

B. 4. 104 7. 52
5. 76 8. 52
6. 38 9. 38

C. 10. 36 13. 108


11. 36 14. 36
12. 72 15. 108

D. 16. 102 22. 24


17. 48 23. 39
18. 51 24. 66
19. 66 25. 24
20. 39 26. 105
21. 51 27. 75

E. 28. 19
29. 91
30. 89

F. Proof:

1. ∠ X + ∠ Y + a = 180 1. Sum of the measures of the angles of a


triangle is 180.
2. b = m ∠ X + m ∠ Y 2. Exterior angle theorem
3. m ∠ Y = a 3. Angles opposite equal sides in the same
triangle are congruent
4. b = m ∠ X + a 4. Substitution
5. m ∠ X = b – a 5. Subtraction Property of Equality

What have you learned

1. 7.5 cm 8. a. 22.5 10. a. 72


2. tangent b. 67.5 b. 108
3. perpendicular c. 135 c. 180
4. semicircle d. 45
5. minor arc 9. a. 72
6. major arc b. 108
7. 16 cm

27
Basic Statistics


   !
01234351607829
"#$#%&&$'"()*(+*&&
), -"#.#/-$0.0"#$
&$%"#&$&'"'(("#,".#(
(0"%*$&$(.#0)012
"#$234 53 36 7 37 35 54 36 55 83 57 7
&239 39 38 3: 36 36 5; 37 5< 54 58 38
5: 5; 37 5< 39 56 7 8< 53 58 5< 39
=.0&$>('")#01(*?
@ABCDEFGFABCHIJGKFL
P ="-&%.*'."(.)&$"&".#(0&$(0"%'"(("#?
P Q"10"-&%1%.(&$%.#)#.#P'-&1$?
P =1"-&%*R*(>1$"("&R"*&)?

] a bcoidogy^z{\|jkl} ÒÐÎÊÎÆÑ


` \
_ wsx „ˆ€™ àÒÊ
\]^kl]mn^okpk‘g€qˆ’lfk—rs„t†u˜šv‹†Œ™†‚…  „‚ž‰…¡˜‹ˆ–‚ƒ… ËÉÒß


efghij “Ž‚…”…‹–†‡ˆ‰…
• åâã
€”
›™ˆ…”‚“‚ˆƒˆ’
ó

3
åãã$:%/3
óôõö÷ñøï

”†‡ˆ‰… 9 ;! )#4% Ñ


JKMNËÆÎÆÒË
…œ‹Œ
„‚ƒ„ ./* +'
'0,-1#2% äâã ÎÅ
~ €…‚†ƒ‡ˆ‰… &'($
LÆËÎÅMÊÒÍÎÆÅçâ çä

Š‹Œ ~†‰„ˆ€™ 6,/ #)% äãã)#"%'
 ,!
ò

! 1#7%
€€„Ÿ…‚˜ˆ‚ƒ…
OMÈçá Ñ
ƒ
…†˜…ˆ‚†‡ˆ‰…
ñ

3 â
â
 ã
çé çê ÑLÉËÊ
! 
'
ð

„ 5,/,$#1%  ' 


 ƒ Œ‹ŒÆÑ
ï

˜
…‘˜ ‹– ‘€†’„… ÎÏÐÆ ÑÒÐ ‚ 
ÅÆnj ÈɎÊÉË ',($#$%!" 8,+/!9 +$"â#4ã%ã
î

…†‡žˆ‹‰•ÌÆÍ çè
#$%
 u çã oyhuuu
éáä

6

– 
áâã ææ æç >oj{=kA
ëì

ù
§¤¤¤
¢£¤ °±±°³¬«3¯·°µúûüýüþ 4#)%
1«­ ÿ¥
¢¤¤
1©45£6µ·°01·2
£
uu
¹º»¼½¾º¿
I{=ioyj
èáä

kD

s u
BH{émæf
éè
A

££¤
FpkéGægpE
mi
Bgppk= æã
@mAkzmi çã

@mAEêkålz
C{jzDâê

£¤¤
suu
i
y>gglzm

7
¥£¤ §¤¤¤
¥£ hv üþþ°¢«±ú
?=o|o=gy

8 hv9
¥¤ h8u h8s h8
h8t h8 u
<xk=opm

h8 h8
ÀÁÂûº»Ä ¦£ h8v h88

u
h89 h9u
¨©ª«¬­®¬¬¯°±²©³°±´°µ¶°±·¸ÓÔÂÕ×Û ÓÔÂÕ×Ü ÓÔÂÕ×Ý ÓÔÂÕ×Þ
h9s h9

h9t h9 h9
h9
ÓÔÂÕÖ× ÓÔÂÕ×Ø ÓÔÂÕ×Ù ÓÔÂÕ×Ú

MFIFBEFBNE.(*"-0"&&0."#O"P#.(."#O%.(&$O#&$(.(#%.#."#"'%,
Q#$P"-((-0((0""&(O*-(.#(((#%P"R#)#%)#(0"&&0.#'")."#,
.#'")."#.(-(%"%).#10#P(#%%O"10#P(
R*#)%R*#(-00(('-&,
S"R#)#((").)(0"#%-0NTCEKE.#1.0$P%'")&&"'#."#>(
"-&."#,.(.#'")."#.(-(%"&)+%0.(."#(1.01.&&''0-(.#'--,
U"V)&OP"R#)#)-(0"#(.%"1)-0)"#$#%("*"R.%%'"&
0.#$(%*0-(#-)*"'&%&$"&.(.#0(.#P,
(-&("'0"&&0."##%.#."#"'%%.(&$%-(.#PDWIXAEOFIJYTE#%
ZBIDWI[E,
7898

9 87   989  
 

y mnopqrmnstpupvqnwxusm
9!"#!$%$"&'
()*+,-./)0' 9#!12!%!3!45&!!13! #! #&!11&"2$6
7-8*,9' 82!%&!"!:"%;<!!4 %$!$6
=-0>)8?-8*,9' 8 11&$!"<!"!!45& %&&"&
"<!!34"21&$6
@0A9B90C9' 8&!"&1%!"<!% :4$!"<!%%;<!";2!"6
D!E 1F%!#&!"$%&%;<!"!# %&&!&!1%$"<!%&!!1
69GHGIJKLMHN%$"<!%&!!16
8OKPGJQ&!"4<1&"2RS%$""$! 3! &!!1!1168"MNTQUQNVQ
24 !%$"&!&!11!"&#:6
WXYZ[Z\]Z[]^X_
`"2!;" "&!%C90?+?a%;<!""*)*+,-./)0!:
69!&;<E";"$:1!!36
"$F2!;" " <&!"$%&?+Bb9c!3?-8*,9!3!%1!"6!"
%1!3%;<<&1!3#!1!%1!"69!"%F 1
%4"$! 1<&!"F"$12&&16
0123422156
d 7&%#<'
e &1!"2 "%3&%#!%1$1:!:"!#4!$< % "
!3!1"$33"22!%
f  "2!3<!%&!!1&""#!%1$
4"$"<"$a%"
!33!!$<!%
g <!%&!!1:&!$!3#
4!%24< &!!1!%1!"
!%2!%<
h !!1!2  "$ "
 %F"311F"$!&
% % "!%2!%
&!%"<"$!%2!%#!1$6
i D!&!3%!"1$"a%!"dF$&%!#"3! !"&!%1$4
&!11&$6
j 7&%!#<!%#!%1$2$"&!33!11!#"2%!"'
e k!%#! "%3&%!"$#"!:"!#!# "<!3&l!
:6
012 34546788965
7 4 3 9 99 545
7 61
 4!" #$%%%&'&&
 &(
) * ! "&"# "'+!," -,!+! "
(
. * !#&,!,   %#/0&(
1 7',#$'#&" %''
%#+#'!" %%( 3!# %""
%#&&,%#!(6 &&#&%#"2#'2
$''2,&+-,&"! ,,(
343567839:;
9 !''" &,##& %<
= >00,!&  +& %%,$%
) ?0!, & . " ,%#9'&&'!'
4,' ,,'#,!!'',@
A B "" !&,!,#&&<
= $ ! %/$
  %&4 C
)  %!#+D2E2?2>2/ F
. ,%#$ %/E&',@
3`=abcd9 efghijklm
G# & %>00!,2,##& %F0!,
"&,(D?"%!, +&!( * !#!
= B "#!,!&@ $"
 B "#!,#&@ ,
+
%%
"
) H% %#&+&!@ !&
. 9#"#!&+&!( ,#&(
= 5>00!,!&(
 5F0!,#&(
) D? ! %F0!,#&+&!
 % %#&"+&! IKJLM
. IKJL %>00 NIKJL %!&+&!O
P IKJLQ>00
RST
UVWXYWVZ[\] D? ^ F0 Q >00 _
2-#&ST!, &+&!(
0121345563274818096
96 68212748 3  
  !""""#$%""&&'(& &)*%+
, )& -&#*
. /,# )(0
1 /,2,# #0
3 4 2  # &&#0
5 6#( )6 &7&#*
8 9",&#&  :9", && (2(2  
,*4  9"$%%#(&  (;& *
. 4 ,# )(#(< &(2 0
1 /,2,# #0
3 4 2  #&(&=>?(; 0
5 6#( )6 , &(&(; *

_ UVWXYZ[\UV]^VWV
@ABCDEFGHAIJABA(#&, ( 6&(2(#*
'$#)#,#&#(#(&& )  2(2
#*
4)# &9K&$LK6)$MK2$4 K, ($4K )# 
)#& & ((*
2  ,(2#)#,6#-&(# 9"#N
LM449 4M494 44LLM 4M949 444ML
LLMM4 4444M 44L44 9449L 44L49
/-(2&)2(&$, (#EFDAOGPC((2)#*4& (#)#(2
(  Q JEBRIEB 
Q BAIÌAOJaFCbcCOH`BAdIC
42(#(  & #)#(&((#)#*'N
2 SEJC(# #T))(22*
012 34546788965
7 4 3 9 99 545
7 61

4 !" #$%&#'(#$)*$(+&'(,$-$,-(.)/0
3,),$,1-/$(,',((#-##))*-(2*3456789",:;56<9"0
4 (,),1,'',),/(#$$,&- tuvwxuyz{|}uz~|uzu€{v€u|uv}{z{
),-%$#(A

pnsnim
g
6'=#$,-(,1,''',((#- f
(0 e
52,(#$”&#•$D #$2,$1–/-)* cd
,''/#-%'%,*0 hijklmnopqmr
_ `a `_ ba
FQBRSTUP VWXYZ[\]^
4'$$#2$)$,1(#-=$&',((,.2#-/#,(0
7>?@@)/$+ >$,*2#)=+ 6>',)+
>#'(0
5(&$A 76
76
7
767 7776 767 7767 7
7

B 3&(,),,1(0 C D#$2,(E
B ‹{|Žu}vwy¤Ž C 52,(#$?@@)/$0
Šm‡h£
 
Ÿ
ž
g
_ `a `_ oml¡ilhp¢
FGFHIJKFLMNOL
L 5(,),$,&$$,$#—$1,$/(-$#- ‹ŒuŽwxŽ
„
hijklmnor‰iŠlh‰r

%(™0 ‚
B ˜,&2-*$/(-$#-%(™#$ 
$',,)E b
C ˜,&2-*.$,$#—š,2,E _ ‚ ƒ „ … `a```b`†r‡ˆl
œ D'-%.$,$#—$›,2,E
‹€Œuu|uv€ŒŽzv{
P 5(,),$,&$-/23$,1$/(-$
hijklmnor‰iŠlh‰r

„
)*#-% .#,/$ #-$/2-$ #-  $',,) ‚
,'$0 
B ˜,&2-*)*$#-%(#-$/2-$E b
C ˜,&2-*$/(-$#-,'$E ‡hplsnkqrrosi‰lm‡hl‰j‘l‰knhl“nhlmijr
œ #-(2,(,1(0 n
‡s
 ksl psq ‰mi mnj’n‘ Š
Šni ‰ rq
0121345563274818096
96 68212748 3  
 1 6!"#"  $$ %"
%&2  "' 2() ("$$!) 3(#) *(+#+
 ,(+&
2 "' ,,323 3 , 2 22*, ,,3 , 2,2*3
- 0" . &
/ , $ &
0 1 $  "+  " )
 $$ ,) !1 9) ! 0) " !
2&2 " #$ 1&
2 "' 92 2,00 99,2 9 92 999,
- 0"1 &
/ , $ &

345564789:;<=;7>634?5;@
7 )%A%+"% $&
2 BCDDE    !%&2 FGHIJHKLE$$ %)
!1! $+ !%&
 +FGHIJHKLEMNOBGNPJBNQKBCPDH$%FGHIJHKLEBCPDH&
,  A%+' RSTSUV WXTTY ZV[\U[]^Y
2 % 4  _`______ a
#
 &
2  _`____`____`______ ba
c _`______ a
* _`_______ d
9 _`_____ e
2 FGHIJHKLE!%  $+$ !%&
fg-hijk lmnopqrst
2 +" "1 
%&
2 ("#) * (+%) *(+) 3() 2(
2 '
2 * *23 * 32 * * * 33* * 2 **23
* * *23 3* 333 2 2 *23
- 0"A%+! &
/ , $ &
012 34546788965
7 4 3 9 99 545
7 61
  !"#$ "$$% &!'()*% > 5?<@27A3B7
+,- ./... 0 273C<4<3367
12343, ./...... 5 ?<7DE6:,4F
167 ./..... 8
6 ./..../.... 99
52: .... ;
5<, =0
GHGIJKLGMNOPQ
M 576,7<D76R4<D43<,<62:3,77<DS54T76@:7UV
11,W:1,W: 111,W:W 1W111, 11,1WW:
11,1W:W 11,W:
U 1XY<U:Z 1,XY,6Z W:XC:Z :@ WXC4F
 6<?,DE6:34Y,D<@F
> [2:@?<@<D@F
Q 6@:72:732:33,77<Y2:@D<,,<U2:C,R,7<D32R?:V
3664466361666316666914661616
 6<?,DE6:34@272Y62<:Y,D<@Y<RF
> \74<6Y,<D2:@V
]:6?Y<D76@:7U<<Y2:@6
]]D32<:<D76@:7U<<Y2:@1F
^ +27?<@<D@_
` 5<6277742:C2:324<,U76R4@<D2:@
<6U4<6CY<67R23Y4<,
7DDF54U7-@<3<<7 9Xa3,,:Z
WXC<<@ZX727D3<4Z<\X6:727D3<4F
576,7UV
9WW9 \WW W\WW 9W\W 9WW
 6<?,DE6:34Y,D<@F
> +27?<@<D@_
^ 6CC77<:U42776R4U<6,@Y32@<6F

ˆ I€‚Lƒ„J€…GƒIKJ€†‡€…€
6C<23,@27<D:@27,4@672:Cbcdefghijklm:@knobljipmF
qrstuvwxrxyvwzzsrr{|}~wzx
6,23-<:23<:<,<@72723,3-CU233:Y67@<@U ‰Š‹ŠŒ‰ŠŒˆ‰
R24<D72723,C7F ‹ˆŽ‹
6:C<@2DD:CY43,23-2:C<:@2DD:23<:F<U74227
<3:C,Y,7<:a7:@2,<DCF
0121345563274818096
96 68212748 3  
29$01 5155++3 
$0416$10/$H010551+1,7
 !"
# $ %&
' ()
*+, -.
/0 -&
1  --

$0144/ 314 '$05+,'$0/$5 Y


51+,60/ Y0'3$/ 7
23 314#13'3 11',531614 3$3$5+/ 3$0 6$0$0/
14 $35 781,50+31,3 $3314#$0093$3$5+1:591,69*
;,$'1'17
<=>?@ABCDEFG
~€
‚ƒ„…†‡ˆ‰Š‹Œ5103$31450/,+51+,60314;,+#$' 72 
$/ 145 51+,60303  4;,05914 5/197
`oJpqrst uvwxyz{|}
2 / /$H03 1#3 9314'$0I ª«¬«­­®°̄«±²³̄´­
,5 3'*93,'03533$67
Ÿ”˜¨š˜–œ©

’Ž
J K $3 +31,+'$0IL ‘Ž
M K $3 61'14 'L Ž
N O1#6093,'03'$0I10/:,$5L Ž
P K 50/ 143,'03'$0I Ž ˜
5 151+6$+IL œ ¢ ¥¢ ˜˜  ©µ˜
—˜™š› Ÿ ¡”›– • ˜¦› ¡œžŸŸ
“”•– ž ¤žœž¥ §œ˜
£
J 75'5144 Q3 1351+,60R
M S 14'$0IT$3 61'7
N ()3,'03'$0I10/:,$57
P 2 1+0,6*143,'03,5 3$0/'$0I3U()V%WV-&V-XU.X
1Y 50/143,'03'$0I$0/5 151+6$+I$3 Z\[]^-XX_U(X_
`a`bcde`fgchf
f 2 3,+3143,H914951+1, i"! j1#0 j+, k0 k9
$05+3314(&9l3,'03#n !" -- ) l m
J 7++,3 33,+3,3$0/ 0'
'#051+,60/ 7
M K $3 6134;,0+9155,$0/951+1,L
N K 50/14 3,'03 H*+,93L
012 34546788965
7 4 3 9 99 545
7 6
 ]Y LMNOPQRSTUTVWOX 5 !"!#$%&
'&%(#)*+'#!'&
}~€w|xyz{|
\Y ,&-
[Y . /! !&$0#
ZY $#$-
Y 1 2&3$&!!4
debf

qrs_`
ghijcklm_n
^_à_b_c

o^p
5 2$!%#'
tuvwxyz{| '&%4'&4
6 7#$&'!!$&!$(!&!! ‡TPQVˆ‰QPTŠ‹‡‹PŒVXOO‰
(!&#&!' - ƒ ŽŽx}xyxŽŽ{|
89 &%: ]
. &"#!"!$&' \
[
1 "'&*++; Z
5 ""!%$&!!% ‚Z [YY\[YY][YYƒ[YY„ [YY… [YY†
;<#&'4 ‚[ uw{|

=>?@ABCDE
4ÂÃÄÅÆÇÈÉ&!!$"#$'&!#&
"0&#&!'&%&''-4$& ‘’“”•–
‘—•˜
!!"0&#-
2'&%&'&& ÊÄÅÉËÈÊ
"'!!"##- ’t¥w| ‘w{Žt¥
«wyw}zw ›¦ œ¬
89F&!&' ”vŽ¡xu{€xw~}|t¢£›¤
!"!"('$#&! ¨}~žxt||wŸ}—œ›  ”~z{tx}
'&!&('- {Ž—t{}zw£
§~Ž ›©
— 

t{
 ›š
’t¥w|“x™w}|z}wtx
t~|wœ›£ —
} œ w}t
‘x~}Ÿœ›œ–w|™zwš|›{œt™w ª~€Žz«w€tš›¦

t
¨}tw|wt
¦› 

·¸¹º» ·¸¼½¹¼¾¿À 5(!&!"!!!"!!$#°


7 )J !'!"!3'®2&!#$%&$&4¯-
4 *) 5 ;+&G!FK!K!&!&'
HI $
G &-HI$J;+ &!;F!"
Á )+ !!&:
& ³
 µ )J±;KK²³°KK $!
5 ;+ *)±;K²)*K; $!
)+±;K²;+K $(! ][­ƒ]­…†­
³±;K²´*K $&! „Y­Z[„­
µ±; ²¶´ $!-
5'&&!!"!&'-
0121345563274818096
96 68212748 3  
IY!Z[\]^ _`abcdefg
2   3 "#)"2-) hijklmnopqrkmo
4.5-¤"+2 "31
 S" ¢¥¦>B§¨©CADª>«CAD§B¬¨­CDBU£ {|x}€}z~
   stuvw y‚†‚zƒv„……
!  "# xyz
$  #"# ‡‚……|wv„……
xˆz ‰„Š|ƒv„……
%  "*)"3 )2"
*1*## ‹€z
&  "*)"3 )2"+
! 2 #''# "#( )*##+
$ ,##-*## #"#  ##'#+
% ./0)"*1*## )2"+
( "*)"3 )2"*1*##
.7/80)4.5
6 9::;4.56<5 =>?@A?>BCDE ./ F 455 ; 4.5 G
& 2 "*)"3 )2"
49/80)4.5
6 9::;4.564H =>?@A?>BCDE 4/ F 455 ; 4.5 G

IJIKLMNIOPLQR
O ŒklqpkŽiopmlqp 2  #'##" 
š…|„}~}u )3S")2" " #"+
€z ! T3 " 3"U
‘„’“|} $ T3 " #")
™„t}“’w ””z 3"U
x‹z
% 7)  " #"V551##)
•–‚—|’~}u 3"'"#( 
y˜z S"-)3")
W 3' WW#'+
R 2  #' 3 ' ›Žkonœqnžpq
)3T3 3X
)# '+
! T3 X#-3+ •~Ÿ|x‹ •~yŸ€|x
z
”
$ 1'").553)  yˆz
+6 3-3"#3
X45# '+ •~Ÿ|x 
xˆz
•~Ÿ|xˆ ¡ƒ–|’xˆz
x€z
011 2343567785496
3
2 8 8 8
43496
5
 4! klmnkopqklrspqtlqusovwxvylkxw
%# "% Ž{ˆˆ ‚‹˜Ž š‹ |š€„„š{–ˆ„‹
!*$ Ž‚‹€„„š{–ˆ„‹ Š{š‚}‡Ÿ|~‹„ “
“‰žˆ ¡‚{ˆ„‹
*!& žž‰œˆ †ˆ ……‰žˆ Ÿ|‹„˜Š„Œ„|ƒ„
)%&* ‡‰ …ž’ˆ
!%&*!* …‡‰
*+%& z„—„|‹‚{˜™„|š‹›
’œ‰†ˆ
'
•|‘…~‘–ˆ{‹
‘‘‰ z{€|}|~|
Ž‚‹||ƒ‘„”ˆ„
‰ ‹‘
’“ˆ ‚|ƒ…}„…Š‰‹{…ˆŒ
†‡ˆ|‹„
 !"#$"%&'
( 5%))*%*&&* %+*$"%!*$ *)%&* %#*'
, -!*$%%&**$ *%*.
/ !%**0 //5%))&**5&&2+%)*1
2 0%#)&!)%* *%*.
/8*+%*)* //52+%)*1
3 6*#+!$)%&**1
40%#)&! **%1

j _`abcdefghfif
56789:;<=><?<#!**&)"%)'

&)!!*"¥¦§¨©ª«¬­&*$L?87M<N>M=8<OP=Q?%?<==®<N>O98¯68N;®?<°=8'

&)!&&*$!";Q=67N±9<P²'
@ABCDEFGDHBEIJHKA@
3L?87M<N>M=8<OP=Q?!*"&%# %*%'
R%S)T#$*U$%)!&.
[\[TZ[TY¢TY£TX¤TZ¢TWXTYVTXZTYXTXYTYZTWYT
Y\T[\XTX\TYVTW£TZVTXWTYXTX£TWYTVZTY¤TXX
R%!+&T&*$& =8<OT*$"%)*
L?87%*#!!'
% )"  VTWTXTYTZT*[\T*#*&*%**%*
!**$%'
-*%#%%U!+&*&*'4+&[\['9 )"[\*
['-!%[%$% )"[\'4*S+&Z['9 )
"Z*['-!%[%$% )"Z'] *$)%
#!%**&*% )^*^%'
45657899
76 8 5 4


656 8 7 
!! "#!$%& %'!&( )"*'&+ >!! "#!$%& %'!&( )"*'&+
(,!)-.# &#& (,!)-.# &#&
/0156 7 8 78
0 2313 0 1233 2<
234 2 983:012 2 :910238
3 ;1728:729 3 :9;177228 4?
2<4=4 8 6;7 8 6;7
6: 62 6: 62
64@2A032<0ABB1010230C10DACEA?A2< 0F2313
BB10F1233E 612D1@20ACC2<G0CG4?4AE
H#)I!#.&#J
K 0F1233 202?A/23A2010 L 20
L 30C03m

:9;177228
K 64@243C10230C10DACE
K 6234/0156442/243@4/A? 027 81233

:910238
34?E ?234B2k/015:6 ml0 3 :910238

1233
mLmLmLm 8 :9;177228
0F1233BA/4C2001020303

6;7
6;7

62
8
K A10DC1D<LB22A? 6: 62 702386:
3A4/<0DE
O[\]^_`a bcdefghij
5?12<03ACCA4B1DA?44
2303/D3CYE7A02/3
2<G0CG4?4A?AC2AB0=4AB
10NDZ
66L60L:2L97L;8L90L61L62L63L;6
;6L97L1;L;9L97L68L60L:3L;1L96
M#!$%& %'!&( )"*'&+("I.&**!I&#I.
: 23
6 6100238
9 67770
; 66918
/0156ND 1 ;
OPOQRSTOUVWXU
U 6B1D2A?91?AA=444Y2B3ACCA
02ND2?A44AB2Z
29 0; 8: 2: 02 26 38 01 8; 30
00 23 27 38 3: 86 36 29 32 29
30 31 31 32
7A02/32<G0CG4?4AAC124Y12CE
012 34546788965
7 4 3 9 99 545
7 6
 5 !"#$%$&%%"#$$'
() *! ! ! *+ (, ! - *) . * )( /
*, - +! *! -- * */ */ -/ / (/ ) +( *, ,
6"0&12"%2$$%$3%4
5 51"0#"631$07$7
"%%&%%&#8"6$'
/ *- , (* -( ! .
*) () */ * *- ( *+
6"0&12"%2$$%$3%4

9 51"0#"6318$""&"8"6$'
(:! *:- !:, (:* :. :! !:/
!:) (:) *:/ !:* *:- (: !:+
6"0&12"%2$$%$3%7"&$0%4
;<=>?@A;?BC@DEF=?GHIJHBH
cdLefghi jklmnopqr
4"K&"$&#3$3&$$""016$8*+1&
%' - .(+ )-**- .!+(, /*.. ) .*(!
L 7"%"$$3"%M0"&36$4
N O%"&$01"4
P 7"1"3&&"%%$3&+1$"1&Q
R 7"&"&&"%%$3&-1$"1&Q
L STUVW XUVVY Z[\]^\_`Y N y …†‡ˆ‰Š‹†ŒŽŒŒ
! a ( }~€‚ƒ„
( aa * x
* aaaa - w
aaa
- aaa v
+ aa *
. abaaa + u
/ a ( t
) aa *
, a ( s s t u v w x y z { |ts
(! a ( ‘’“”
0121345563274818096
96 68212748 3  
 ! " #!$%&'% %(%)!%*
%$$!%*!+
%, $*( -../
 01-../
2./
343567839:;<=
9 > 3% )?@A*$B&!CA%*C)?% *A C%?$ )&**D.? )!E
.,',2,,,D,,',.,,,,,.,,D,,',,,,',D,D,.,',,,,'
F G!B%AC)%?*& E
H*A C%?? )!I I%$ )&*
HH?$%*%?? )!I  $ )&*+
= J%))%I*( (!%?$ )&*BE
,,K,,',D,,,,D,,',,',,.,,,D,'
> 4(*! &*))B*&?@A*$BC)+
F #%I *B$ )&**&& BL
 #%I *B%? $ )&*I(&%DL
" M $*(ID% %B!%)&L
N 0!)B &%*$%)A *( +
O 2 (P*( ! %I! *A C%?%*! \W abcdecfghijklmnojppqrstnkurvmk
!$%&CBC!QC))%P2.R $ %&+ [
> M %*!$%%$$A& %!?@A*)BL Z
F 4* %I *B%$$!%*!I.% % Y
%*!!$%&L
 7*I $*(%? $ !I?I XW W \ X ] Y ^ Z _ [ `\W\\\X\]
 *%*!!$%&L wxyfz{
S 2 *A C!%?(%)!Q$Q&CB?%%C)))B$ $ ?% ..V!!%*IE
D.'.DDDD.'D
> 3% )?@A*$BC)?% (P*&+
F G! C)%?*& *A C%?( !I  )BQ$Q&E
HT$)BD(%)! HH)!D(%)!+
U 1$%&I!Q%? *A C%?(%)!!$%&CB(%)
! %%**C))( !&A*( !!%*+2 !A)!IE
.KVVK
,,,,,,. ,.2
,,KK
,,,,K ,,2VVK
,,,,,
> 3% )?@A*$BC)?% &C%P+
F 0I$%)A *( %? &+
 J*& *A C%?( !*
I $  ! %%!$%&E
HT$)BV(%)! HH)!V(%)!+
  !"#$%
& '((!)*+,-./*0!( !1!"2!-*+!+('(3456!+74/*0!(8!"!(,9!):
!;*99*8'+2+<,/!"(*;,-!(8!"!-*<+!)=
7>:3?:7@:34:3?:3A:34:7>:7@:3?:34:3B:7>:7@:3?:34:3A:7@:34:3A:
3?:7>:34:7@:34:3?:3A:34:7@:7>:3A:34:3?:7@:34:34:7>:3B:3A:7@
C "!"!;"!D<!+-E /9!;*"'()5
F G*8,+E/*0!()!0-9E34,-!(H
I G*8,+E/*0!()34*",*"!,-!(H
J 6;"-'*+*;/*0!()9!((+34,-!(H
K *E*< '+L !,+<;-<"!"M(-9','(19')H

e ]^_`ab_c^b_d^
!NOPQ*"PROSPTO*;(!*;+<,/!"('(+',*"+ ,!(<"!*;!'",'))9!56!9L
/*< 1!"2!(99!',!5U*"!0,9!=
V !1!"2!(!!)*;-"
V 1!"2!!'2 +)8!'2
V !1!"2!(-*"!;*"!(
V !1!"2!82!*"'+-*,!5
!NOPQ*"PROSPTO'(!*9*;99(-*"!()'1')!)/E !+<,/!"*;(-*"!(5
U*"!0,9!:!,!+*;A:B:B:3:W+)??'(
AXBXBX3XWX??
W Z!"!"!W(-*"!([
YWB4
Y3
0123422156 35789
16 09 9
'(-<((*81!"2!(-+/!<(!) *-*,"!)';;!"!+ (!(*;)5\*<
,E8'( *-*+(')!"!(!(!,!+(=
V+ !9(6*"9)<:f"g'9(-*"!)+
1!"2!*;AB
l 2*9(!",-5h!",+E
(-*"!)+1!"2!*;?3
l 2*9(!",-5
V!i>,*)!9"1!9(??W l L,!"9'"!
*;;<!9:8!"!(!jB "1!9(?Akl L,
!"9'"!5
V+,!"'-+U**/99:8E'(!1!"2!
!'2 +) 8!'2 *; ! 9E!"(
',*"+H
0121345563274818096
96 68212748 3  
,aIbcde` fghijklmn 2 9
  !!!"#!!!$%&'( 98 6
9979(
)* +!!+!"+#+!$!+!$+%&+'+
* $$$
*!!
,-,./01,23,
2   ! % & 4 " #(
5 36786  979  : ' & ' # ' !!!(
; 2 <= 9=8<>87?6=9@ #'= #"= #%= != #$= #$=
 A=> <=  87?6=9(
B 79?C8=759C89 66<=979@ !!' !!% !!:
!'& !%%
36786 A=6= 59D9?C89(
E 7>9?>66D969!%=999 97%& !: &# %$ &: 44 !: "%
4 %' "' 4%9(F <9 997A 9G
H U9>669 97?7  <9>9>667 9(2 99 
7 9?69 8@
@!&' !&" !&! !!$ !%" 97?@ !&% !%" !&: !%!!%$
I  A=9 7 9 <>J7 >9>66(
K F  9 99A=7 =9G
3 3 7<=89989 96 79
8!'?9(
LMNOPQR  # # : # $  " 4  LMNOPSR $ #  # : !' & $ $ : $
I 36786 7 =8(
K 2 !'989TMNOPQ!!TMNOPS(U7898V868>9
7 =8987 9(2869G
W F 7 =8> 9G
` 2 =A9 <9 =6997>J=69 XYZ[ LNY\] LYZ[]
 676>66997( 66JU< %&: $
I  8>=697 =6 ^ 9 %&" !'
9 ( ^_ %%: $
K F 7 =69  9 >9A=

 %!$ 
7G 5F= %' :

66J5J7 !$$ 
3<6J !$ #
   !"#$%&'(
01234325 0678839
8202382310
6 3 2
)#*+!,#*-*#../01*223%/1 1 $!#
/-$%!456!."$%*/6*)!,/)#%/22$6
$7"$%*8$)#9:/01*220+$;3%!*)+/<
1 $!#*)$!, /<="2/#+9
>?@ABCCDEEFG =+!0,$%+/%,/<<$$H!%2*6+I,/##/)
1//2I4=3%!*)+/<1 $!#I8$!+0%$I
$.$6%/""$%I6*20#$62*J0*6<$%#*2*+$%9
KLMNNOPOQ
R !.$%#$,/##/)1//2#%$$J0!%#$%+/<#$1!.0"$!, 2*69
2!,$;3%!*)+/<1 $!#!#$J0!26*+#!),$+!"!%#*)$!, 2*69
S !T$2#$2*6+!+"2/#+5I4IU!)6=9!#0%!#$$!, "2/#1*# 5V8/<1!#$%9
W -$%!U1$$X"$%*/6I"$%</%8#$</22/1*)3Y
Z )"2/#5+J0$$[$46%/"+/<1!#$%/)#/$!, 3%!*)/<1 $!#$-$%.1$$X6!.9
Z )"2/#4+J0$$[$46%/"+/<1!#$%/)#/$!, 3%!*)/<1 $!#$-$%.\/)6!.I
]$6)$+6!.!)6^%*6!.9
Z )"2/#U+J0$$[$46%/"+/<1!#$%!)656%/"/<6*20#$6<$%#*2*+$%/)#/$!,
3%!*)$-$%.\/)6!.I]$6)$+6!.!)6^%*6!.9
Z )"2/#=+J0$$[$46%/"+/<1!#$%!)656%/"/<6*20#$6<$%#*2*+$%/)#/$!,
3%!*)$-$%.1$$X6!.9
_ 2!,$!22#$"2/#+*)#$+!8$+!<$I+$2#$%$6"2!,$1*# "2$)#./<2*3#9
` -$%.\/)6!.I]$6)$+6!.!)6^%*6!.I%$,/%6#$8$!) $*3#/<!).3$%8*)!#*)3
+$$6+</%$!, "2/#9-/*6 !)62*)3!).+//#+9\!X$!#!T2$#/+088!%*+$./0%
%$+02#+9
a b+$3%!"+!)6#$2!)30!3$/<+#!#*+#*,+#/,/88$)#/)./0%%$+02#+9

d 5e7 f8g8 f32h381h062


Z !-$%!3$ Z T!%3%!" Z ,!#$3/%*,!26!#!
Z 6/#"2/# Z <%$J0$),. Z <%$J0$),.#!T2$
Z *)<$%$),$ Z 8$!) Z 8/6$
Z )08$%*,!26!#! Z "*$,!%# Z "/"02!#*/)
Z %!)6/8+!8"2$ Z +!8"2$ Z +#$8c!)6c2$!<"2/#
Z #!22.!)6<%$J0$),.#!T2$

  
    !"
012314516789
# $%&'()*)%+,'*)+*-./&))01*2-+))*$%-
*2,%345%2-%-*2,%64*$)''*2'
/,%(+/+)360
7'-))'(*+*%&/8*$9+&)+*%&20
: . ; < = = < 36 3> ? 3 36 : ; =
. . ? 3> ; . 3 < = < ? @ . 33 ?
A &&-*$BC2*/-%&')'(+)$0
D *)(B'&&'(+*9C2)+'*)E
FG'(,*-)2$*)(+*/&))H
FFG'(,*-9+&)(+*/&))H
FFFIB/+'*'B/&))()%'*+* +&H
FJ I/*9'B/&))()%'*+*K/H
L *$+M+*9/',++'*4&&-N)B+*&$+M()($$B'&&'(+*9)/')E
@O@<O3@O<<O><O6@O:@O@
A 5+*$,*'B.)/')9+M*0
D B+9)*$&'())/')(&B'24(('2&$%M9'B
=,+*+*9)/')H
P /'&2,*9)*)M&2'B ~€‚ƒ„…„‚€†‡‡ˆ‰
'*,'*N))&)*N)2M+*9) ]W idjfhklmbnhdgohbpqhr
'0
A FI 9''$) )*  \[WW
+9)M&2'B&/+/&+,) ZW
)'&$H
FFQ+M(')'*)(-+),- YXWW
M*$0
D I()'&M&2'B9''$) VW
)'&$H
y_a` d`m_gfh
jt_l̀ fgh
vfpa_fgduuj

|}|_bghflh
_gws bah

{fjfehzhlfsh
ibng _bgfah

R B6>>),&&8+/*&+*/)&+8 ah
ujdzf
xdhm wfadl
isdj cdefbe

8&)*$'))()'&$4(
bgo_l

_h
m

()+M9+/H
o
^_`ab

S Š)+/'*)(B'&&'(+*9C2)+'*)E
A I(),T'$+))/2)+*9 ©€…‚ª‡ƒ€…‚€‰‚ˆ‡†ˆ«„¬­
$))2&'B),'8+*9H ‹ŒŽ‘’Ž‘Œ“”
D I69'2)'B$+))),$2 §—•“–”—•”— •–—˜™Œš”—˜•
›œ
=>U'B&&),'8+*9&$$)H ›¨ ž“Œ—
R B6>>>>'&$+$+*'*-) ‘‘Ÿ •“Žš
)2&'B),'8+*94)+,'( –”¡—¢•”—”
,*-$+$B',E
F$+)) FF&2*9/*/ ž“
ž“Œ—‘£•‘—”
FFF'/*/)H ¤Ÿ¥‘•‘—
›¦ œ
Œ—

     !


012314516789
" #$%&'($$)*'+$,&'%-&++&)+%')&+./)#&&)+0
1#&'($23)')%--,%('*'+$,&'&+/.#0
43)'($$,)'/)+$*'5+$63),)&$*'0
&'$*%)(3)'($$)/+(&++0,&'3),)#%,
(',),&'.#5'/)+$3)(',)'/)+$.#5+$'))+0
74726724266247246724
8 )+'9/,:9+/-*%')(&+.3)'($$)(&++&+/
.#0
; (/)%9#+.)')(,:9+/-),3)'0
< +)(#+-.#'$&$(&+++)/&33)'=
> ?('(&++@')%3)=
A +(/+.),.#'$&$(&++/&33)'=
B?('#++9#*),3)'(&++/&3$.#=
C .&3+&/'+''%),D677777
()),.))$'*-')'')$9&+.&'E*9- [\]^_
'%0 `ab_]`
cde
8 F,)(&/')')%$*'= nbb]p\gg
; ?3%9),.))$',)(')'+$ ice fbgh
++&'('')%$= d ie
< '#/G+.),.))$'('')%$&+ nl]k^`` j^kkl̀
')'')@'H#&%%&)+9)I+$),J%@0 doe cme
K?3%9),,&+''.('')%$=
KK?(')%#)9+),++&'+$
(')''%'=
L $)%)')('-'),'$ zy}€}ƒ„ ‚q
…†‡ˆ…‰„……Š‡‹‡…
3)+)+$-0
8 M)(#+-'($)+&'$-=
; E&+$#)$),$0
< ?/+.),'$(
,&'=
qrs tuv wxyt zx{|us|}y
~}ss~}
N O&%%@'OP-$3&''+(3&-&+&'+.),'&'0$&%-'%'),
+(3&-Q 4254R56S54656T56U56R56V56S56656R5645W5VX
E&+$#+),$&%-'%'0
Y &#&+#&+9'P+,)/9')#''9+)/&3&#%'&'
.&3+*%)(Q
6R 4S 26 S 44 6S 2R 4U 6R V 4S 6T 46 47 62
8 )+'9/'#Z+$Z%,%))$&'%-&'$0
; E&+$3.&#,)/9')#')/&3&#%'0
Lecture 2 – Grouped Data
Calculation
1. Mean, Median and Mode
2. First Quantile, third Quantile and
Interquantile Range.
Mean – Grouped Data
Example: The following table gives the frequency distribution of the number
of orders received each day during the past 50 days at the office of a mail-order
company. Calculate the mean. Number f
of order
10 – 12 4
13 – 15 12
16 – 18 20
19 – 21 14
n = 50
Solution:
X is the midpoint of the
Number f x fx class. It is adding the class
of order limits and divide by 2.
10 – 12 4 11 44
13 – 15 12 14 168
x=
∑ fx = 8 3 2 = 1 6 .6 4
16 – 18 20 17 340 n 50
19 – 21 14 20 280
n = 50 = 832
Median and Interquartile Range
– Grouped Data
Step 1: Construct the cumulative frequency distribution.
Step 2: Decide the class that contain the median.
Class Median is the first class with the value of cumulative
frequency equal at least n/2.
Step 3: Find the median by using the following formula:
⎛ n ⎞
⎜ 2 - F ⎟
M e d ia n = L m + ⎜ ⎟ i
⎜ f m ⎟
⎝ ⎠
Where:
n = the total frequency
F = the cumulative frequency before class median
f = the frequency of the class median
m

i = the class width


Lm = the lower boundary of the class median
Example: Based on the grouped data below, find the median:
Time to travel to work Frequency
1 – 10 8
11 – 20 14
21 – 30 12
31 – 40 9
41 – 50 7
Solution:

1st Step: Construct the cumulative frequency distribution


Time to travel Frequency Cumulative
to work Frequency
1 – 10 8 8
11 – 20 14 22
21 – 30 12 34
31 – 40 9 43
41 – 50 7 50

n 50
= = 25 class median is the 3rd class
2 2
So, F = 22, fm = 12, L = 20.5 and i = 10
m
Therefore,
⎛n ⎞
⎜ - F ⎟
Median = Lm + ⎜ 2 ⎟i
f
⎜ m ⎟
⎝ ⎠
⎛ 25 - 22 ⎞
= 21.5 + ⎜ ⎟ 10
⎝ 12 ⎠
= 24

Thus, 25 persons take less than 24 minutes to travel to work and another 25 persons
take more than 24 minutes to travel to work.
Quartiles
Using the same method of calculation as in the Median,
we can get Q1 and Q3 equation as follows:
⎛n ⎞ ⎛ 3n ⎞
⎜4-F ⎟ ⎜ 4 -F ⎟
Q1 = LQ1 + ⎜ ⎟i Q3 = LQ3 + ⎜ ⎟i
⎜ f Q1 ⎟ ⎜ f Q3 ⎟
⎝ ⎠ ⎝ ⎠

Example: Based on the grouped data below, find the Interquartile Range

Time to travel to work Frequency


1 – 10 8
11 – 20 14
21 – 30 12
31 – 40 9
41 – 50 7
Solution:
1st Step: Construct the cumulative frequency distribution

Time to travel Frequency Cumulative


to work Frequency
1 – 10 8 8
11 – 20 14 22
21 – 30 12 34
31 – 40 9 43
41 – 50 7 50
2nd Step: Determine the Q1 and Q3

n 50 ⎛n ⎞
Class Q 1 = = = 12 . 5
4 4 ⎜ 4 -F ⎟
Q1 = LQ1 + ⎜ ⎟i
⎜ fQ1
Class Q1 is the 2nd class

Therefore, ⎝ ⎠
⎛ 12.5 - 8 ⎞
= 10.5 + ⎜ ⎟ 10
⎝ 14 ⎠
= 13.7143
⎛n ⎞
3n 3 ( 50 ) ⎜ 4 -F ⎟
Class Q 3 = = = 37 .5 Q3 = LQ3 + ⎜ ⎟i
⎜ fQ3
4 4 ⎟
⎝ ⎠
⎛ 37.5 - 34 ⎞
Class Q3 is the 4th class = 30.5 + ⎜ ⎟ 10
Therefore, ⎝ 9 ⎠
= 34.3889

Interquartile Range
IQR = Q3 – Q1

IQR = Q3 – Q1
calculate the IQ
IQR = Q3 – Q1 = 34.3889 – 13.7143 = 20.6746
Mode – Grouped Data
Mode
•Mode is the value that has the highest frequency in a data set.
•For grouped data, class mode (or, modal class) is the class with the highest frequency.
•To find mode for grouped data, use the following formula:

⎛ Δ1 ⎞
M o d e = Lmo + ⎜ ⎟i
Δ
⎝ 1 + Δ 2 ⎠

Where:

i is the class width


Δ1 is the difference between the frequency of class mode and the frequency
of the class after the class mode
Δ 2 is the difference between the frequency of class mode
and the frequency of the class before the class mode

Lmo is the lower boundary of class mode


Calculation of Grouped Data - Mode
Example: Based on the grouped data below, find the mode

Time to travel to work Frequency


1 – 10 8
11 – 20 14
21 – 30 12
31 – 40 9
41 – 50 7
Solution:
Based on the table,

Lmo = 10.5, Δ1 = (14 – 8) = 6, Δ 2 = (14 – 12) = 2 and


i = 10

⎛ 6 ⎞
M o d e = 1 0 .5 + ⎜ ⎟ 1 0 = 1 7 .5
⎝ 6 + 2 ⎠
Mode can also be obtained from a histogram.
Step 1: Identify the modal class and the bar representing it
Step 2: Draw two cross lines as shown in the diagram.
Step 3: Drop a perpendicular from the intersection of the two lines
until it touch the horizontal axis.
Step 4: Read the mode from the horizontal axis
Variance and Standard Deviation
-Grouped Data
( ∑ fx )
2

Population Variance: ∑ fx 2

N
σ2 =
N

( ∑ fx )
2

∑ fx 2

n
Variance for sample data: s =
2

n −1

Standard Deviation:
Population: σ2 = σ2

Sample: s2 = s2
Example: Find the variance and standard deviation for the following data:

No. of order f
10 – 12 4
13 – 15 12
16 – 18 20
19 – 21 14
Total n = 50

Solution:

No. of order f x fx fx2

10 – 12 4 11 44 484
13 – 15 12 14 168 2352
16 – 18 20 17 340 5780
19 – 21 14 20 280 5600
Total n = 50 832 14216
( ∑ fx )
2

∑ fx 2

n
Variance, s2 =
n −1
(832 )
2

14216 −
= 50
50 − 1
= 7.5820

Standard Deviation, s = s = 7.5820 = 2.75


2

Thus, the standard deviation of the number of orders received at


the office of this mail-order company during the past 50 days is 2.75.

You might also like